You are on page 1of 67

LABOR 1 | JMM | CASE DIGESTS 2020 – TOPICS 1-3

DISCLAIMER: Please don’t copy-paste the digest provided here as your assignment in Labor. Unless, your work. The
following digest shall be use as guide for the digest assignments and online recitations. Lastly, please do not submit
the group digest as your individual output for Labor, as a precaution.

TOPIC 2: BASIC PRINCIPLES

1. Sonza vs. ABS-CBN, G.R. No. 138051, June 10, 2004

PETITIONER: RESPONDENT:
Jose Y. Sonza (Lose) ABS-CBN Broadcasting Corporation (Win)

SPECIAL DETAILS:
Services Sonza would render to ABS-CBN:
1. Co-host for Mel & Jay radio program, 8:00-10:00 am, Mondays to Fridays
2. Co-host for Mel & Jay television program, 5:30-7:30 pm, Sundays
Talent fee of Sonza:
- P310,000 monthly for the first year and P317,000 for the 2nd and 3rd year to be paid on the 10th and 25th
days of the month

LAW & PRINCIPLES:


Four-Fold Test of Employer-Employee Relationship
1. Selection and engagement of the employee
2. Payment of wages
3. Power of dismissal
4. Employer’s power to control the employee on the means and methods by which the work accomplished
(Control Test)

FACTS:
- ABS-CBN signed an Agreement with the Mel and Jay Management and Development Corporation
(MJMDC), represented by petitioner, as President and General Manager, and Carmela Tiangco, as EVP and
Treasurer. In the Agreement, MJMDC, as “Agent,” agreed to provide petitioner’s services to respondent
exclusively.
- petitioner wrote a letter to respondent which served as notice of rescission of their Agreement. It contained
the resignation of petitioner due to “recent events concerning his programs and career” which petitioner
claims as acts of the station which are violative and a breach of their Agreement.
- petitioner filed a complaint against respondent claiming that respondent did not pay his salaries, separation
pay, service incentive leave pay, 13th month pay, signing bonus, travel allowance and amounts due under
the Employees Stock Option Plan (“ESOP”)
- respondent filed a motion to dismiss on the ground that no employer-employee relationship existed
between the parties
- ABS-CBN continued to remit petitioner’s monthly talent fees through PICBank then opened a new account
and deposited petitioner’s talent fees and other payments due him under the Agreement
- The Labor Arbiter dismissed the complaint for lack of jurisdiction since no employer-employee relationship
existed between petitioner and respondent. Labor Arbiter affirmed the argument of respondent that all the
benefits received by petitioner were merely talent fees and other contractual benefits and should not be
deemed as salaries, wages, and/or other remuneration given to an employee.

ISSUES:
Whether or not an employer-employee relationship existed between Sonza and ABS-CBN

RULING:
No employer-employee relationship.

A. Selection and engagement of the employee


- Independent contractors often present themselves to possess unique skills to distinguish them
from ordinary employees. ABS-CBN engaging in Sonza’s services because of his peculiar skills,

Page 1 of 67
LABOR 1 | JMM | CASE DIGESTS 2020 – TOPICS 1-3

talent, and celebrity status indicates, but does not conclude, that what existed between them was
an independent contractual relationship.
B. Payment of wages
- Talent fees and benefits paid to petitioner were the result of negotiations that led to the
Agreement, not because of an employee-employer relationship because if it was so, there would
be no need to stipulate benefits such as 13th month pay and Medicare since the law automatically
incorporates this into every employer-employee contract.
- Petitioner’s talent fees are so huge and out of the ordinary that such power to bargain fees indicate
more an independent contractual relationship rather than an employer-employee relationship.
C. Power of Dismissal
- petitioner failed to show that ABS-CBN could terminate his services on grounds other than breach
of contract.
- Even after ABS-CBN ceased broadcasting Sonza’s programs, ABS-CBN still paid him his talent
fees since ABS-CBN remained obligated to pay Sonza’s talent fees during the life of the Agreement
D. Power of Control (Control Test)
- no local precedent on whether a radio and television program host is an employee or an
independent contractor so SC referred to foreign case law
- A broadcast specialist who works under minimal supervision is an independent contractor. No
records show that ABS-CBN exercised any supervision and control over how Sonza utilized his
skills and talent in his shows
- One can still be an independent contractor although the hirer reserved certain supervision to
insure the attainment of the desired result. The hirer must not however deprive the one hired from
performing his services according to his own initiative
- Being an exclusive talent does not necessarily mean being under the control of the hirer. Even an
independent contractor can validly provide his services exclusively to the hiring party

NOTES:
Control Test - based on the extent of control the hirer exercises over a worker. The greater the supervision and
control the hirer exercises, the more likely the worker is deemed an employee. The less control the hirer exercises,
the more likely the worker is considered an independent

2. Lazaro vs. Social Security Commission, 435 SCRA 472 [2004]

PETITIONER: RESPONDENT:
ANGELITO L. LAZARO, Proprietor of Royal Star SOCIAL SECURITY COMMISSION, ROSALINA LAUDATO,
Marketing, petitioner, SOCIAL SECURITY SYSTEM and THE HONORABLE
proprietor of Royal Star Marketing ("Royal Star"), which is COURT OF APPEALS
engaged in the business of selling home appliances.

Argument:
Laudato was not qualified for social security coverage, There is an employer-employee relationship.
as she was not an employee of Royal Star, her income
dependent on a generation of sales and based on
commissions. It is argued that Royal Star had no control
over Laudato's activities, and that under the so-called
"control test," Laudato could not be deemed an
employee.

SPECIAL DETAILS:
● Lazaro's arguments are nothing more but a mere reiteration of arguments unsuccessfully posed before
two bodies: the SSC and the Court of Appeals. The determination of an employer-employee relationship
depends heavily on the particular factual circumstances attending the professional interaction of the
parties. The Court is not a trier of facts and accords great weight to the factual findings of lower courts or
agencies whose function is to resolve factual matters.

Page 2 of 67
LABOR 1 | JMM | CASE DIGESTS 2020 – TOPICS 1-3

Evidence found that Laudato was an employee of Royal Star:

1. Cash vouchers issued by Royal Star to Laudato


2. Calling cards of Royal Star denominating Laudato as a "Sales Supervisor"
3. Certificates of Appreciation issued by Royal Star to Laudato in recognition of her unselfish and loyal efforts
in promoting the company.
4. Memorandum dated 3 May 1980 of Teresita Lazaro, General Manager of Royal Star, directing that no
commissions were to be given on all "main office" sales from walk-in customers and enjoining salesmen
and sales supervisors to observe this new policy which evinced control over their work.

LAW & PRINCIPLES:


CONTROL TEST DOCTRINE
It is an accepted doctrine that for the purposes of coverage under the Social Security Act, the determination of
employer-employee relationship warrants the application of the "control test," that is, whether the employer controls
or has reserved the right to control the employee, not only as to the result of the work done, but also as to the means
and methods by which the same is accomplished.

● Laudato filed a petition before the SSC for social security coverage and remittance of unpaid monthly social
security contributions against her three (3) employers. Respondent is herein petitioner.
○ Laudato alleged: That despite her employment as sales supervisor of the sales agents for 7 years,
Lazaro had failed to report her to the SSC for compulsory coverage or remit Laudato's social
security contributions.
● Lazaro denied that Laudato was a sales supervisor of Royal Star. His defense:
○ She was a mere sales agent whom he paid purely on commission basis.
○ She was not subjected to definite hours and conditions of work. Because of this, she cannot be
deemed an employee of Royal Star.
● SSC applied the control test. Held:
○ Laudato was an employee of Royal Star
○ Order for Royal Star for unremitted SSS contributions, plus penalty and damages against Laudato.
● After Lazaro’s Motion for Reconsideration before the SSC was denied. He filed a petition before the CA,
reiterating his defense.
● CA decision:
○ Lazaro did not go into the specific of his petition that and merely reiterated the arguments he
presented in the lower court, failing to support substantial evidence. Thus, CA affirmed SSC
decision.

ISSUES: WON Laudato was an employee of Royal Star?

No error on the two decisions.


Lazaro's arguments may be dispensed with by applying precedents. Suffice it to say, the fact that Laudato was paid
by way of commission does not preclude the establishment of an employer-employee relationship. (Grepalife v.
Judico). Neither does it follow that a person who does not observe normal hours of work cannot be deemed an
employee. (Cosmopolitan Funeral Homes v. Maalat). Noting also, that Laudato was a sales supervisor and not a
mere agent. As such, Laudato oversaw and supervised the sales agents of the company, and thus was subject to
the control of management as to how she implements its policies and its end results. Such findings was supported
by substantial evidence. Through the examination of the cash vouchers, calling cards, certificates of appreciation,
issued by Royal Star.

NOTES:
Grepalife v. Judico
the Court upheld the existence of an employer-employee relationship between the insurance company and its
agents, despite the fact that the compensation that the agents on commission received was not paid by the company
but by the investor or the person insured. The relevant factor remains, as stated earlier, whether the "employer"
controls or has reserved the right to control the "employee" not only as to the result of the work to be done but also
as to the means and methods by which the same is to be accomplished.

Page 3 of 67
LABOR 1 | JMM | CASE DIGESTS 2020 – TOPICS 1-3

Cosmopolitan Funeral Homes, Inc. v. Maalat


the employer similarly denied the existence of an employer-employee relationship, as the claimant according to it,
was a "supervisor on commission basis" who did not observe normal hours of work. This Court declared that there
was an employer-employee relationship, noting that "[the] supervisor, although compensated on commission basis,
[is] exempt from the observance of normal hours of work for his compensation is measured by the number of sales
he makes."

3. Phil. Global Communication vs. De Vera, 459 SCRA 260 [2005]

PETITIONER: Philippine Global Communications, Inc. RESPONDENT: Ricardo De Vera is a physician by


(PhilCom), is a corporation engaged in the business of profession whom petitioner enlisted to attend to the
communication services and allied activities medical needs of its employees.

SPECIAL DETAILS:

Proposed plan of works by De Vera:

1. Application of preventive medicine including periodic check-up of employees; 2. Holding of clinic hours in the
morning and afternoon for a total of five (5) hours daily for consultation services to employees; 3. Management and
treatment of employees that may necessitate hospitalization including emergency cases and accidents; 4. Conduct
pre-employment physical check-up of prospective employees with no additional medical fee; 5. Conduct home visits
whenever necessary; 6. Attend to certain medical administrative function such as accomplishing medical forms,
evaluating conditions of employees applying for sick leave of absence and subsequently issuing proper certification,
and all matters referred which are medical in nature.

LAW & PRINCIPLES:


Article 280 of the Labor code (sic) provides:
'The provisions of written agreement to the contrary notwithstanding and regardless of the oral agreements of the
parties, an employment shall be deemed to be regular where the employee has been engaged to perform in the usual
business or trade of the employer, except where the employment has been fixed for a specific project or undertaking
the completion or termination of which has been determined at the time of the engagement of the employee or
where the work or services to be performed is seasonal in nature and the employment is for the duration of the
season.'
An employment shall be deemed to be casual if it is not covered by the preceding paragraph: Provided, That, any
employee who has rendered at least one (1) year of service, whether such is continuous or broken, shall be
considered a regular with respect to the activity in which he is employed and his employment shall continue while
such activity exists.'

ART. 157. Emergency medical and dental services. - It shall be the duty of every employer to furnish his employees
in any locality with free medical and dental attendance and facilities consisting of:
(a) The services of a full-time registered nurse when the number of employees exceeds fifty (50) but not more than
two hundred (200) except when the employer does not maintain hazardous workplaces, in which case the services
of a graduate first-aider shall be provided for the protection of the workers, where no registered nurse is available.
The Secretary of Labor shall provide by appropriate regulations the services that shall be required where the number
of employees does not exceed fifty (50) and shall determine by appropriate order hazardous workplaces for
purposes of this Article;
(b) The services of a full-time registered nurse, a part-time physician and dentist, and an emergency clinic, when the
number of employees exceeds two hundred (200) but not more than three hundred (300); andcr
(c) The services of a full-time physician, dentist and full-time registered nurse as well as a dental clinic, and an
infirmary or emergency hospital with one bed capacity for every one hundred (100) employees when the number of
employees exceeds three hundred (300).

Page 4 of 67
LABOR 1 | JMM | CASE DIGESTS 2020 – TOPICS 1-3

In cases of hazardous workplaces, no employer shall engage the services of a physician or dentist who cannot stay
in the premises of the establishment for at least two (2) hours, in the case of those engaged on part-time basis, and
not less than eight (8) hours in the case of those employed on full-time basis. Where the undertaking is
nonhazardous in nature, the physician and dentist may be engaged on retained basis, subject to such regulations as
the Secretary of Labor may prescribe to insure immediate availability of medical and dental treatment and
attendance in case of emergency.

FACTS:
● De Vera, via a letter offered his services to proposing his plan of works required of a practitioner in industrial
medicine.
● The parties agreed and formalized respondent's proposal in a document denominated as RETAINERSHIP
CONTRACT.
● Background about the retainership contract:
○ Period of 1 year subject to renewal. Renewed yearly.
○ respondent will cover "the retainership the Company previously had with Dr. K. Eulau"
○ respondent's "retainer fee" will be at P4,000.00 a month.
○ Said contract was renewed yearly.
○ Went on from 1981 to 1994 with changes in the retainer's fee.1995 and 1996, renewal of the
contract was only made verbally.
● Turning point: Philcom, thru a letter bearing "TERMINATION - RETAINERSHIP CONTRACT", informed De
Vera of its decision to discontinue the latter's "retainer's contract with the Company effective at the close
of business hours of December 31, 1996" because management has decided that it would be more
practical to provide medical services to its employees through accredited hospitals near the company
premises.
● De Vera filed a complaint for illegal dismissal before the National Labor Relations Commission.
Allegations:
○ He had been actually employed by Philcom as its company physician since 1981 and was
dismissed without due process
○ He was designated as a "company physician on retainer basis" for reasons allegedly known only
to Philcom.
○ Since he was not conversant with labor laws, he did not give much attention to the designation as
anyway he worked on a full-time basis and was paid a basic monthly salary plus fringe benefits,
like any other regular employees of Philcom.
● NLRC Decision: Petition dismissed for lack of merit.
○ Held: De vera as a "retained physician" under a valid contract mutually agreed upon by the parties,
De Vera was an "independent contractor" and that he "was not dismissed but rather his contract
with [PHILCOM] ended when said contract was not renewed after December 31, 1996.
● NLRC Appeal: Decision reversed.
○ Held: De Vera is Philcom's "regular employee" and accordingly directed the company to reinstate
him to his former position without loss of seniority rights and privileges and with full backwages
from the date of his dismissal until actual reinstatement.
● With its motion for reconsideration having been denied by the NLRC, Philcom then went to the Court of
Appeals on a Petition for Certiorari, imputing grave abuse of discretion amounting to lack or excess of
jurisdiction on the part of the NLRC when it reversed the findings of the labor arbiter and awarded thirteenth
month pay and traveling allowance to De Vera even as such award had no basis in fact and in law.
● CA Decision: Deleted the travel allowance and ordered the payment of separation to De Vera in lieu of
reinstatement.
● Philcom filed a motion for reconsideration but was denied.

ISSUES:
1. Whether an employer-employee relationship exists between petitioner and respondent
2. Whether respondent is a regular employee as contemplated in Art. 280 of the Labor Code

RULING:
1st Issue:
The court in determining the existence of an employer-employee relationship, has invariably adhered to the four-fold
test, to wit: [1] the selection and engagement of the employee; [2] the payment of wages; [3] the power of dismissal;

Page 5 of 67
LABOR 1 | JMM | CASE DIGESTS 2020 – TOPICS 1-3

and [4] the power to control the employee's conduct, or the so-called "control test", considered to be the most
important element.

Findings based on the elements:


1. It was respondent himself who sets the parameters of what his duties would be in offering his services to
petitioner which was borne by his May 1891 letter which was the substantial basis of the labor arbiter’s
findings that there existed no employer-employee relationship between petitioner and respondent.
2. An April 1982 letter by De Vera, indicated that the complainant was proposing to extend his time with the
respondent and seeking additional compensation for said extension. This shows that the respondent
PHILCOM did not have control over the schedule of the complainant as it [is] the complainant who is
proposing his own schedule and asking to be paid for the same. Also, a proof that the complainant
understood that his relationship with the respondent PHILCOM was a retained physician and not as an
employee. If he were an employee he could not negotiate as to his hours of work.
3. The complainant being a Doctor of Medicine, is presumed to be a well-educated person. Yet, he claims to
be not conversant with the law. But, admits to the services of the contract.
4. That the contract clearly is a retainership contract. The retainer fee is indicated thereon and the duration
of the contract for one year is also clearly indicated in paragraph 5 of the Retainership Contract. Thus, he
cant claim unawareness.
5. Undisputed by Dr. De Vera on the fact that he was never included in the payroll, was never deducted any
contribution for remittance to the Social Security System (SSS); and was subjected to ten (10%) percent
withholding tax for his professional fee, in accordance with the National Internal Revenue Code, matters
which are simply inconsistent with an employer-employee relationship.
6. Absence of the element of control, whereby the employer has reserved the right to control the employee
not only as to the result of the work done but also as to the means and methods by which the same is to
be accomplished. Petitioner had no control over the means and methods by which the respondent went
about performing his work at the company premises. He could even embark in the private practice of his
profession, not to mention the fact that respondent's work hours and the additional compensation therefore
were negotiated upon by the parties. In fine, the parties themselves practically agreed on every terms and
conditions of the respondent's engagement, which thereby negates the element of control in their
relationship.

The elements of an employer-employee relationship are wanting in this case. The records are replete with evidence
showing that respondent had to bill petitioner for his monthly professional fees. It simply runs against the grain of
common experience to imagine that an ordinary employee has yet to bill his employer to receive his salary. The
power to terminate the parties' relationship was mutually vested on both. Either may terminate the arrangement at
will, with or without cause.

2nd issue:
Brief background: It was found that there was no employer-employee relationship, absent the elements. Despite
that, the NLRC and CA ruled that petitioner is a regular employee at the time of his separation.
CA Rationale: Based their reasoning on Art. 280 of the Labor Code that if the employee is engaged in the usual
business or trade of the employer, more so, that he rendered service for at least one year, such employee shall be
considered as a regular employee. That private respondent was with petitioner since 1981 and his employment was
not for a specific project or undertaking, the period of which was pre-determined and neither the work or service of
private respondent seasonal.

SC Decision: SC does not agree. The appellate court's premise that regular employees are those who perform
activities which are desirable and necessary for the business of the employer is only true in the case of an
independent contractorship as well as in an agency agreement. The provision merely distinguishes between two (2)
kinds of employees, i.e., regular and casual. It does not apply where, as here, the very existence of an employment
relationship is in dispute.

Respondent invokes Art. 157 of the Labor Code, arguing he satisfies all the requirements. However, SC said he
should have read carefully the statutory provision he invoked because he would have noticed that in non-hazardous
workplaces, the employer may engage the services of a physician "on retained basis." While it is true that the
provision requires employers to engage the services of medical practitioners in certain establishments depending
on the number of their employees, nothing is there in the law which says that medical practitioners so engaged be

Page 6 of 67
LABOR 1 | JMM | CASE DIGESTS 2020 – TOPICS 1-3

actually hired as employees, adding that the law, only requires the employer "to retain", not employ, a part-time
physician who needed to stay in the premises of the non-hazardous workplace for two (2) hours. As it is, Article 157
of the Labor Code clearly and unequivocally allows employers in non-hazardous establishments to engage "on
retained basis" the service of a dentist or physician. Nowhere does the law provide that the physician or dentist so
engaged thereby becomes a regular employee. The very phrase that they may be engaged "on retained basis", revolts
against the idea that this engagement gives rise to an employer-employee relationship.

4. ABS-CBN vs. Nazareno, G.R. No. 164156, Sept. 26, 2006

PETITIONER: ABS-CBN Broadcasting Corp. (Lose) RESPONDENT: Marilyn Nazareno, Merlous


● Nazareno et.al. are NOT REGULAR EMPLOYEESS but Gerzon, Jennifer Deiparine, Josephine Lerasan
are PAs who basically assist in the conduct of a (Win)
particular program ran by an anchor or talent and that ● Nazareno et.al. should be recognized
their duties include monitoring and receiving incoming as regular employees since they
calls from listeners and field reporters and calls of news belonged to a "work pool" from which
sources; ABS-CBN chose persons to be given
● Nazareno et.al. are considered in the industry as specific assignments at its discretion,
"program employees" in that, as distinguished from and were thus under its direct
regular or station employees, they are basically supervision and control regardless of
engaged by the station for a particular or specific nomenclature.
program broadcasted by the radio station.
● PAs, reporters, anchors and talents occasionally
"sideline" for other programs they produce, such as
drama talents in other productions.
● As program employees, a PA's engagement is
coterminous with the completion of the program, and
may be extended/renewed provided that the program is
on-going; a PA may also be assigned to new programs
upon the cancellation of one program and the
commencement of another. As such program
employees, their compensation is computed on a
program basis, a fixed amount for performance
services irrespective of the time consumed. At any rate,
petitioner claimed, as the payroll will show, respondents
were paid all salaries and benefits due them under the
law.

● NLRC commited a grave abuse of its discretion in giving
Art. 223 of the Labor Code a liberal application to
prevent the miscarriage of justice.

SPECIAL DETAILS:
● ABS_CBN made Nazareno et.al. perform the following tasks and duties:
a) Prepare, arrange airing of commercial broadcasting based on the daily operations log and digicart of
respondent ABS-CBN;
b) Coordinate, arrange personalities for air interviews;
c) Coordinate, prepare schedule of reporters for scheduled news reporting and lead-in or incoming reports;
d) Facilitate, prepare and arrange airtime schedule for public service announcement and complaints;
e) Assist, anchor program interview, etc; and
f) Record, log clerical reports, man based control radio.
● Nazareno et.al. were under the control and supervision of Assistant Station Manager Dante J. Luzon, and
News Manager Leo Lastimosa.
● ABS-CBN’s revised schedule for Nazareno et.al. are the following:

Page 7 of 67
LABOR 1 | JMM | CASE DIGESTS 2020 – TOPICS 1-3

The Monday-Saturday
4:30 A.M.-8:00 A.M.-Marlene Nazareno.
Miss Nazareno will then be assigned at the Research Dept.
From 8:00 A.M. to 12:00
4:30 P.M.-12:00 MN-Jennifer Deiparine
Sunday
5:00 A.M.-1:00 P.M.-Jennifer Deiparine
1:00 P.M.-10:00 P.M.-Joy Sanchez

LAW & PRINCIPLES:


● There are two kinds of regular employees under the law:
(1) those engaged to perform activities which are necessary or desirable in the usual business or trade of
the employer; and
(2) those casual employees who have rendered at least one year of service, whether continuous or broken,
with respect to the activities in which they are employed.
● Primary standard in determining regular employment:
(1) nature of work performed and its relation to the scheme of the particular business or trade in its
entirety.
(2) Also, if the employee has been performing the job for at least a year, even if the performance is not
continuous and merely intermittent, the law deems repeated and continuing need for its performance as
sufficient evidence of the necessity if not indispensability of that activity to the business.

FACTS:
● ABS-CBN Broadcasting Corporation (ABS-CBN) a Philippine broadcasting business that owns a network of
television and radio stations, and whose operations revolve around the broadcast, transmission, and relay
of telecommunication signals, decided to hire Nazareno, Gerzon, Deiparine, and Lerasan as production
assistants (PAs) on different dates.
● Nazareno et.al. were assigned at the news and public affairs, for various radio programs in the Cebu
Broadcasting Station, with a monthly compensation of P4,000. They were issued ABS-CBN employees'
identification cards and were required to work for a minimum of eight hours a day, including Sundays and
holidays.
● ABS-CBN decided to exclude Nazareno et.al. from the Collective Bargaining Agreement (CBA) it executed
with the ABS-CBN Rank-and-File Employees.
● ABS-CBN then decided to assign Nazareno, et.al. to non-drama programs, and that the DYAB studio
operations would be handled by the studio technician effective August 1, 2000 and revised their schedule
and other assignments
● Nazareno et.al. filed a Complaint for Recognition of Regular Employment Status, Underpayment of
Overtime Pay, Holiday Pay, Premium Pay, Service Incentive Pay, Sick Leave Pay, and 13th Month Pay with
Damages against ABS-CBN before the NLRC.
● Labor Arbiter Jose G. Gutierrez dismissed the complaint without prejudice for lack of interest on April 30,
2001.
● Instead of re-filing their complaint with the NLRC within 10 days from May 16, 2001, Nazareno et.al. filed
an Earnest Motion to Refile Complaint with Motion to Admit Position Paper and Motion to Submit Case For
Resolution on June 11, 2001.
● NLRC granted their request.
● Labor Arbiter ruled in favor of Nazareno et.al. and declared them as regular employees. Labor Arbiter
subsequently denied the appeal of ABS-CBN.
● NLRC affirmed and slightly modified the Labor Arbiter’s decision, it ruled that Nazareno were regular
employees and were entitled to the CBA. Subsequent motion of reconsideration was denied.
● CA dismissed the ABS-CBN’s petition and their subsequent motion for reconsideration
● The Case is elevated to the SC.

ISSUES:
Whether or not Nazareno et.al. are regular employees of ABS-CBN

RULING:
● The SC denied the petition and affirmed the CA’s decision.

Page 8 of 67
LABOR 1 | JMM | CASE DIGESTS 2020 – TOPICS 1-3

● The Court rejected ABS-CBN’s that Nazareno et.al. are considered as its talents, hence, not regular
employees of the broadcasting company and that the functions performed by the Nazareno et.al. are not
at all necessary, desirable, or even vital to its trade or business is belied by the evidence on record.

Regular employee distinguished from project and season employees:


● Where a person has rendered at least one year of service, regardless of the nature of the activity performed,
or where the work is continuous or intermittent, the employment is considered regular as long as the activity
exists, the reason being that a customary appointment is not indispensable before one may be formally
declared as having attained regular status.
● The Court cited several rulings including Universal Robina Corp. v. Catapang, where it ruled that the primary
standard in determining regular employment is the reasonable connection between the particular activity
performed by the employee in relation to the usual trade or business of the employer. The connection can
be determined by considering the:
(1) nature of work performed and its relation to the scheme of the particular business or trade in its
entirety.
(2) Also, if the employee has been performing the job for at least a year, even if the performance is not
continuous and merely intermittent, the law deems repeated and continuing need for its performance as
sufficient evidence of the necessity if not indispensability of that activity to the business.
.
● On the other hand, "project employees," are those whose completion or termination of which is more or
less determinable at the time of employment, such as those employed in connection with a particular
construction project, and "seasonal employees" whose employment by its nature is only desirable for a
limited period of time. Even then, any employee who has rendered at least one year of service, whether
continuous or intermittent, is deemed regular with respect to the activity performed and while such activity
actually exists.

Two Kinds of Regular Employees:


● The SC also noted that there are two kinds of regular employees under the law:
(1) those engaged to perform activities which are necessary or desirable in the usual business or trade of
the employer; and
(2) those casual employees who have rendered at least one year of service, whether continuous or broken,
with respect to the activities in which they are employed.

Test in determining if employee is regular:


● What determines whether a certain employment is regular or otherwise is not the will or word of the
employer, to which the worker oftentimes acquiesces, much less the procedure of hiring the employee or
the manner of paying the salary or the actual time spent at work. It is the character of the activities
performed in relation to the particular trade or business taking into account all the circumstances, and in
some cases the length of time of its performance and its continued existence. It is obvious that one year
after they were employed by petitioner, respondents became regular employees by operation of law.

Differentiating the present case from the SONZA case:


● Unlike the case of Sonza v. ABS-CBN Broadcasting Corporation, the employer-employee relationship in the
present case between ABS-CBN and the respondents has been proven.
(1) In the selection and engagement of respondents, no peculiar or unique skill, talent or celebrity status
was required from them because they were merely hired through petitioner's personnel department just
like any ordinary employee.
(2)The so-called "talent fees" of respondents correspond to wages given as a result of an employer-
employee relationship. Respondents did not have the power to bargain for huge talent fees, a circumstance
negating independent contractual relationship.
(3) Petitioner could always discharge respondents should it find their work unsatisfactory, and respondents
are highly dependent on the petitioner for continued work.
(4) The degree of control and supervision exercised by petitioner over respondents through its supervisors
negates the allegation that respondents are independent contractors.
● The presumption is that when the work done is an integral part of the regular business of the employer and
when the worker, relative to the employer, does not furnish an independent business or professional
service, such work is a regular employment of such employee and not an independent contractor.

Page 9 of 67
LABOR 1 | JMM | CASE DIGESTS 2020 – TOPICS 1-3

● Hence, the SC found Nazareno et.al. as entitled to the benefits provided for in the existing CBA between
petitioner and its rank-and-file employees.

NOTES:
● While Nazareno et.al. failed to perfect their appeal from the decision of the Labor Arbiter within the
reglementary period therefore, it was ABS-CBN who perfected its appeal within the period, and since it had
filed a timely appeal, the NLRC acquired jurisdiction over the case to give due course to its appeal and
render the decision of November 14, 2002.

On Labor Arbiter’s alleged abuse of discretion:


● Labor Arbiter is mandated by law to use every reasonable means to ascertain the facts in each case
speedily and objectively, without technicalities of law or procedure, all in the interest of due process.

On the definition of project and the test to determine if it’s a project:


● The SC noted that there were two definitions of “project”:
(1) a project may refer to a particular job or undertaking that is within the regular or usual business of the
employer, but which is distinct and separate, and identifiable as such, from the other undertakings of the
company. Such job or undertaking begins and ends at determined or determinable times.
(2) the term project may also refer to a particular job or undertaking that is not within the regular business
of the employer. Such a job or undertaking must also be identifiably separate and distinct from the
ordinary or regular business operations of the employer. The job or undertaking also begins and ends at
determined or determinable times.

● The principal test is whether or not the project employees were assigned to carry out a specific project or
undertaking, the duration and scope of which were specified at the time the employees were engaged for
that project.
● The SC further pointed out that the assigned tasks given to Nazareno et.al. are necessary or desirable in
the usual business or trade of the petitioner. The persisting need for their services is sufficient evidence of
the necessity and indispensability of such services to petitioner's business or trade.
● While length of time may not be a sole controlling test for project employment, it can be a strong factor to
determine whether the employee was hired for a specific undertaking or in fact tasked to perform functions
which are vital, necessary and indispensable to the usual trade or business of the employer. Additionally,
ABS-CBN did not report the termination of Nazareno’s et.al.' employment in the particular "project" to the
Department of Labor and Employment Regional Office having jurisdiction over the workplace within 30 days
following the date of their separation from work, using the prescribed form on employees'
termination/dismissals/suspensions.

5. Francisco vs. NLRC, 500 SCRA 690 [2006]

PETITIONER: Angelina Francisco (Win) RESPONDENT: NLRC, Kasei Corporation,


● Francisco is an employee of Kasei Corp, Acting Seichiro Takashi, Timoteo Acedo, Delfin Liza,
Manager, handled the recruitment of all employees and Irene Ballestros, Trinidad Liza, and Ramon
performed management administration functions; and Escueta (Lose)
she represented the company in all dealings with ● Francisco is not an employee of Kasei
government agencies, especially with the Bureau of Corporation, she was hired in 1995 as
Internal Revenue (BIR), Social Security System (SSS) one of its technical consultants on
and in the city government of Makati; and to administer accounting matters and act
all other matters pertaining to the operation of Kasei concurrently as Corporate Secretary.
Restaurant which is owned and operated by Kasei ● Francisco’s esignation as technical
Corporation consultant depended solely upon the
will of management, hence, her
consultancy may be terminated any
time considering that her services were

Page 10 of 67
LABOR 1 | JMM | CASE DIGESTS 2020 – TOPICS 1-3

only temporary in nature and dependent


on the needs of the corporation.

SPECIAL DETAILS:
● In her stay with the company, Francisco was never entrusted with the corporate documents and neither
did she attend any board meeting nor required to do so.
● Francisco, as Acting Manager, handled the recruitment of all employees and performed management
administration functions; and she represented the company in all dealings with government agencies,
especially with the Bureau of Internal Revenue (BIR), Social Security System (SSS) and in the city
government of Makati; and to administer all other matters pertaining to the operation of Kasei Restaurant
which is owned and operated by Kasei Corporation.
● Francisco became the acting manager in 1996.
● Kasei Corp. et.al. presented to the Labor Arbiter a list of employees for the years 1999 and 2000 duly
received by the BIR showing that Francisco was not among the employees reported to the BIR, as well as
a list of payees subject to expanded withholding tax which included petitioner. SSS
● records were also submitted showing that petitioner's latest employer was Seiji Corporation.

LAW & PRINCIPLES:


Two-tiered test in determining employer-employee relationship:
(1) right of control test where the person for whom the services are performed reserves a right to control not
only the end to be achieved but also the means to be used in reaching such end. It is the putative employer's
power to control the employee with respect to the means and methods by which the work is to be
accomplished; and the
(2) underlying economic realities of the activity or relationship or existing economic conditions prevailing
between the parties, like the inclusion of the employee in the payrolls, can help in determining the existence
of an employer-employee relationship.

To determine the underlying economic realities of the between employer and employee depends the
following must be present:
(1) the extent to which the services performed are an integral part of the employer's business;
(2) the extent of the worker's investment in equipment and facilities;
(3) the nature and degree of control exercised by the employer;
(4) the worker's opportunity for profit and loss;
(5) the amount of initiative, skill, judgment or foresight required for the success of the claimed independent
enterprise;
(6) the permanency and duration of the relationship between the worker and the employer; and
(7) the degree of dependency of the worker upon the employer for his continued employment in that line of
business.

FACTS:
● Kasei Corporation during its incorporation stage in 1995 hired Angelina Francisco as its Accountant and
Corporate Secretary. Ms. Francisco was assigned to handle all the accounting needs of the company and
was also designated as Liaison Officer to the City of Makati to secure business permits, construction
permits and other licenses for the initial operation of the company.
● Even though Francisco worked as the corporate sec., she never prepared any legal document nor was she
able to represent the company as its Corporate Secretary. However, on some occasions, she was prevailed
upon to sign documentation for the company.
● Francisco became acting manager until Liza R. Fuentes replaced her in 2001.
● Francisco alleged that she was required to sign a prepared resolution for her replacement but she was
assured that she would still be connected with Kasei Corporation.
● Kasei Corporation reduced Francisco’s salary by P2,500.00 a month (Jan-Sept 2001), and
did not give her a mid-year bonus allegedly because the company was not earning well.
● Francisco later found out from Acedo that she was no longer connected with the Kasei Corp so she decided
not to report for work and filed an action for constructive dismissal before the labor arbiter.
● The Labor Arbiter found Francisco to have been illegally dismissed.
● The NLRC affirm the Labor Arbiter’s decision.

Page 11 of 67
LABOR 1 | JMM | CASE DIGESTS 2020 – TOPICS 1-3

● CA reversed the NLRC’s decision and denied the subsequent motion for reconsideration.
● Case is elevated to the SC.

ISSUES:
Whether or not there was an employer-employee relationship between Francisco and Kasei Corporation

RULING:
The SC granted the petition and reinstated the NLRC’s decision.

On determining the employer-employee relationship:


The SC noted that courts have relied on the a two-tiered test:
(3) right of control test where the person for whom the services are performed reserves a right to control not
only the end to be achieved but also the means to be used in reaching such end. It is the putative employer's
power to control the employee with respect to the means and methods by which the work is to be
accomplished; and the
(4) underlying economic realities of the activity or relationship or existing economic conditions prevailing
between the parties, like the inclusion of the employee in the payrolls, can help in determining the existence
of an employer-employee relationship.

By applying the control test, there is no doubt that Francisco is an employee of Kasei Corporation because she was
under the direct control and supervision of Seiji Kamura, the corporation's Technical Consultant. She reported for
work regularly and served in various capacities as Accountant, Liaison Officer, Technical Consultant, Acting Manager
and Corporate Secretary, with substantially the same job functions, that is, rendering accounting and tax services to
the company and performing functions necessary and desirable for the proper operation of the corporation such as
securing business permits and other licenses over an indefinite period of engagement.

On economic realities of the activity:


The determination of the relationship between employer and employee depends upon the circumstances of the
whole economic activity, such as:
(1) the extent to which the services performed are an integral part of the employer's business;
(2) the extent of the worker's investment in equipment and facilities;
(3) the nature and degree of control exercised by the employer;
(4) the worker's opportunity for profit and loss;
(5) the amount of initiative, skill, judgment or foresight required for the success of the claimed independent
enterprise;
(6) the permanency and duration of the relationship between the worker and the employer; and
(7) the degree of dependency of the worker upon the employer for his continued employment in that line of business.

The broader economic reality test, likewise proves that Francisco was an employee of the corporation because she
had served the company for six years before her dismissal, receiving check vouchers indicating her salaries/wages,
benefits, 13th month pay, bonuses and allowances, as well as deductions and Social Security contributions from
August 1, 1999 to December 18, 2000. In addition there were also several evidences shown such as the corporation’s
report to the SSS signed by Irene Ballesteros and the Kasei Corporation president’s signature in the SSS specimen
signature card and the inclusion of Francisco’s name in the on-line inquiry system of the SSS.

The two-tiered test takes into consideration the totality of circumstances surrounding the true nature of the
relationship between the parties. This is especially appropriate in this case where there is no written agreement or
terms of reference to base the relationship on; and due to the
complexity of the relationship based on the various positions and responsibilities given to
the worker over the period of the latter's employment.

When Francisco was designated as the General Manager, Kansei corporation made a report to the SSS signed by
Irene Ballesteros and the Kasei Corporation president’s signature in the SSS specimen signature card and the
inclusion of her name in the on-line inquiry system of the SSS evinces the existence of an employer-employee
relationship between petitioner and respondent corporation

Page 12 of 67
LABOR 1 | JMM | CASE DIGESTS 2020 – TOPICS 1-3

NOTES:

On the diminution of payment to the employee:


A diminution of pay is prejudicial to the employee and amounts to constructive dismissal. Constructive dismissal is
an involuntary resignation resulting in cessation of work resorted to when continued employment becomes
impossible, unreasonable or unlikely; when there is a demotion in rank or a diminution in pay; or when a clear
discrimination, insensibility or disdain by an employer becomes unbearable to an employee.

The SC cited its ruling in Globe Telecom, Inc. v. Florendo-Flores, where it held that where an employee ceases to
work due to a demotion of rank or a diminution of pay, an unreasonable situation arises which creates an adverse
working environment rendering it impossible for such employee to continue working for her employer. Hence, her
severance from the company was not of her own making and therefore amounted to an illegal termination of
employment.

The Court further noted that in affording full protection to labor, this Court must ensure equal work opportunities
regardless of sex, race or creed. Even as we, in every case, attempt to carefully balance the fragile relationship
between employees and employers, we are mindful of the fact that the policy of the law is to apply the Labor Code
to a greater number of employees. This would enable employees to avail of the benefits accorded to them by law,
in line with the constitutional mandate giving maximum aid and protection to labor, promoting their welfare and
reaffirming it as a primary social economic force in furtherance of social justice and national development.

6. Nogales et al., vs. Capitol Medical Center et al., G.R. No. 142625, December 19, 2006

PETITIONER: Rogelio Nograles, and on behalf RESPONDENT: Capitol Medical Center; Dr. Oscar Estrada. Dr. Ely
of Roger Anthony, Angelica, Nancy and Villaflor, Dr. Rosa Uy,Dr. Joel Enriquez, Dr. Perpetua Lacson, Dr.
Michael Christopher (Win) Noe Espinola, Nurse J. Dumlao (Lose)
● CMC is vicariously liable for Dr. ● Dr. Estrada was a mere visiting physician and that it
Estrada's negligence based on admitted Corazon because her physical condition then
Article 2180 in relation to Art. 2176 of was classified an emergency obstetrics case.
the Civil Code ● Dr. Estrada is an independent contractor "for whose
● CMC, in allowing Dr. Estrada to actuations CMC would be a total stranger." CMC
practice and admit patients at CMC, maintains that it had no control or supervision over Dr.
should be liable for Dr. Estrada’s Estrada in the exercise of his medical profession.
malpractice.

SPECIAL DETAILS:
● The RTC did not find any legal justification to implead the other physicians and hospital personnel, besides
Dr. Estrada, as civilly liable.
● Upon appeal, petitioners claimed that aside from Dr. Estrada, the remaining respondents should be held
equally liable for negligence, pointing out the extent of each respondent’s alleged liability.
● Dr. Estrada did not appeal the decision of the Court of Appeals affirming the decision of the RTC

LAW & PRINCIPLES:


● Borrowed servant- provides that once the surgeon enters the operating room and takes charge of the
proceedings, the acts or omissions of operating room personnel, and any negligence associated with such
acts or omissions, are imputable to the surgeon. While the assisting physicians and nurses may be
employed by the hospital, or engaged by the patient, they normally become the temporary servants or
agents of the surgeon in charge while the operation is in progress, and liability may be imposed upon the
surgeon for their negligent acts under the doctrine of respondeat superior.

FACTS:
● Corazon Nogales (Corazon) was pregnant of her 4th child and was under the exclusive prenatal care of Dr.
Oscar Estrada (Dr. Estrada).

Page 13 of 67
LABOR 1 | JMM | CASE DIGESTS 2020 – TOPICS 1-3

● On her last trimester of pregnancy, Dr. Estrada noted an increase in Corazon’s blood pressure and
development of leg edema which may lead to a dangerous complication of pregnancy. When Corazon
started experiencing mild labor pains, she and her husband Rogelio opted to see Dr. Estrada for
examination, and the latter advised them to admit Corazon to the Capitol Medical Center (CMC).
● Shortly, Corazon’s bag of water ruptured and started to experience convulsions.
● Dr. Estrada and another physician in the name of Dr. Villaflor began extracting the baby, which allegedly
tore a piece of cervical tissue of the patient. After the baby was taken out of the womb, Corazon began to
manifest moderate vaginal bleeding which rapidly became profuse.
● Corazon died of “hemorrhage, post-partum.” even though there was an effort from the doctors to revive
her.
● Rogelio Nogales, et al. sued CMC, Dr. Estrada, and the other involved medical personnel of the hospital (Dr.
Villaflor, Dr. Uy, Dr. Enriquez, Dr. Lacson, Dr. Espinola, and a certain Nurse J. Dumlao) for the death of
Corazon, for damages and charged CMC with negligence in the selection and supervision of Corazon’s
physicians and hospital staff.
● The RTC found Dr. Estrada as solely liable for damages.
● The CA affirmed the RTC’s decision and applied the “borrowed servant” doctrine considering that Dr.
Estrada was an independent contractor who was merely exercising hospital privileges.
● Case is elevated to the SC (CA’s decision on affirming RTC decision convicting Dr. Estrada became final
because he did not appeal ito to the SC).

ISSUES:
Whether or not CMC is vicariously liable for the negligence of Dr. Estrada.
Whether or not there is employer-employee relationship between Dr. Estrada and CMC.

RULING:
SC ruled that Capitol Medical Center vicariously liable for the negligence of Dr. Oscar Estrada

On determining the relationship between a hospital and a consultant/visiting physician:


● SC cited its ruling in Ramos v CA, where it held that hospitals exercise significant control in the hiring and
firing of consultants and in the conduct of their work within the hospital premises. Doctors who apply for
"consultant" slots, visiting or attending, are required to submit proof of completion of residency, their
educational qualifications; generally, evidence of accreditation by the appropriate board (diplomate),
evidence of fellowship in most cases, and references. It was held in the said case that an employer-
employee relationship in effect exists between hospitals and their attending and visiting physicians.
● While "consultants" are not, technically employees, a point which respondent hospital asserts in
● denying all responsibility for the patient's condition, the control exercised, the hiring, and the right to
terminate consultants all fulfill the important hallmarks of an employer-employee relationship, with the
exception of the payment of wages. In assessing whether such a relationship in fact exists, the control test
is determining.

The employer (or the hospital) must have the right to control both the means and the
details of the process by which the employee (or the physician) is to accomplish his task.

On the liability of hospital for a physician's negligence


● In general, a hospital is not liable for the negligence of an independent contractor-physician. There is,
however, an exception to this principle, the doctrine of apparent authority.
● Doctrine of apparent authority- states that the hospital may be liable if the physician is the "ostensible"
agent of the hospital. SC cited a U.S. ruling, Gilbert v. Sycamore Municipal
● Hospital, where it ruled that for a hospital to be liable under the doctrine of apparent authority, a plaintiff
must show that:
(1) the hospital, or its agent, acted in a manner that would lead a reasonable person to conclude that the
individual who was alleged to be negligent was an employee or agent of the hospital;
(2) where the acts of the agent create the appearance of authority, the plaintiff must also prove that the
hospital had knowledge of and acquiesced in them; and
(3) the plaintiff acted in reliance upon the conduct of the hospital or its agent, consistent with ordinary care
and prudence
● The doctrine of apparent authority essentially involves two factors to determine the liability of an
independent-contractor physician. The first factor focuses on the hospital's manifestations and is

Page 14 of 67
LABOR 1 | JMM | CASE DIGESTS 2020 – TOPICS 1-3

sometimes described as an inquiry whether the hospital acted in a manner which would lead a reasonable
person to conclude that the individual who was alleged to be negligent was an employee or agent of the
hospital. In this regard, the hospital need not make express representations to the patient that the treating
physician is an employee of the hospital; rather a representation may be general and implied.

● The Court found no single evidence pointing to CMC's exercise of control over Dr. Estrada's treatment and
management of Corazon's condition.
● There was no showing that CMC had a part in diagnosing Corazon's condition. While Dr. Estrada enjoyed
staff privileges at CMC, such fact alone did not make him an employee of CMC. CMC merely allowed Dr.
Estrada to use its facilities when Corazon was about to give birth, which CMC considered an emergency.

NOTES:
On signature:
● Without any indication in these consent forms that Dr. Estrada was an independent contractor-physician,
the Spouses Nogales could not have known that Dr. Estrada was an independent contractor. Significantly,
no one from CMC informed the Spouses Nogales that Dr. Estrada was an independent contractor.

(1) CMC granted staff privileges to Dr. Estrada. CMC extended its medical staff and facilities to Dr. Estrada.
Upon Dr. Estrada's request for Corazon's admission, CMC, through its personnel, readily accommodated
Corazon and updated Dr. Estrada of her condition.
(2) CMC made Rogelio sign consent forms printed on CMC letterhead. Prior to Corazon's admission and
supposed hysterectomy, CMC asked Rogelio to sign release forms, the contents of which reinforced
Rogelio's belief that Dr. Estrada was a member of CMC's medical staff.
● Without any indication in these consent forms that Dr. Estrada was an independent contractor-physician,
the Spouses Nogales could not have known that Dr. Estrada was an independent contractor. Significantly,
no one from CMC informed the Spouses Nogales that Dr. Estrada was an independent contractor. On the
contrary, Dr. Atencio, who was then a member of CMC Board of Directors, testified that Dr. Estrada was
part of CMC's surgical staff. 53
● Estrada's referral of Corazon's profuse vaginal bleeding to Dr. Espinola, who was then the Head of the
Obstetrics and Gynecology Department of CMC, gave the impression that Dr. Estrada as a member of
CMC's medical staff was collaborating with other CMC-employed specialists in treating Corazon.

7. Coca-Cola Bottlers Phils., vs. Dr. Climaco, G.R. No. 146881, February 15, 2007

PETITIONER: Coca-cola Bottlers Philippines RESPONDENT: Dr. Dean Climaco

SPECIAL DETAILS: The Court agrees with the Labor Arbiter and the NLRC that there is nothing wrong with the
employment of respondent as a retained physician of petitioner company and upholds the validity of the Retainership
Agreement which clearly stated that no employer-employee relationship... existed between the parties.
Considering that there is no employer-employee relationship between the parties, the termination of the Retainership
Agreement, which is in accordance with the provisions of the Agreement, does not constitute illegal dismissal of
respondent.

FACTS: Dr. Dean Climaco (respondent), a medical doctor, was hired by Coca-cola Bottlers Phil. (petitioner) by virtue
of a Retainer Agreement. The terms and conditions are as follows:
1. That the agreement shall only for 1 year beginning Jan. 1, 1988 to Dec. 31, 1988. Either party
may terminate the contract upon giving a 30-day written notice to the other;
2. That petitioner shall compensate respondent a retainer fee of P3,800/month. The DOCTOR may
charge professional fee for hospital services rendered in line with his specialization;
3. That in consideration of the retainer’s fee, the DOCTOR agrees to perform the duties and
obligations in the COMPREHENSIVE MEDICAL PLAN, made an integral part of this retainer agreement;
4. That the DOCTOR shall observe clinic hours at the company’s premises from Monday to
Saturday of a minimum of two (2) hours each day or a maximum of TWO (2) hours each day or treatment
from 7:30 a.m. to 8:30 a.m. and 3:00pm to 4:00pm. It is further understood that the DOCTOR shall be on
call at all times during the other work shifts to attend to emergency case(s);
5. That no employee-employer relationship shall exist between the company and the DOCTOR.

Page 15 of 67
LABOR 1 | JMM | CASE DIGESTS 2020 – TOPICS 1-3

The retainer agreement expired after 1 year. However, despite the non-renewal of the agreement, respondent
continued to perform his functions as company doctor to petitioner until he received a letter dated March 9, 1995
from the company ending their retainership agreement. Respondent thereafter filed a complaint before the NLRC
seeking recognition as a regular employee of petitioner and thus prayed from payment of all the benefits of a regular
employee including 13th month pay, COLA, holiday pay, service incentive leave, and Christmas bonus. Also,
respondent filed another complaint for illegal dismissal against petitioner.

In the Decisions dated Nov. 28, 1996 & Feb. 24, 1997, both the instant complaint was dismissed by the Labor Arbiters
and subsequently affirmed by the NLRC on the ground that no employer-employee relationship existed between
petitioner-company and respondent. However when it was elevated to CA for review, the latter ruled that employer-
employee relationship existed between the parties after applying the four-fold test: (1) power to hire employee (2)
payment of wages (3) power to dismissal (4) and power to control over the employee with respect to the means and
methods by which the work is to be accomplished.

ISSUES: Whether or not there exists an employer-employee relationship between the parties

RULING: The Court agrees with the finding of the Labor Arbiter and the NLRC. The Court held that the Labor Arbiter
and the NLRC correctly found that Petitioner Company lacked the power of control over the performance by
respondent of his duties.

The Court citing the case of Neri vs. NLRC said, petitioner, through the Comprehensive Medical Plan, provided
guidelines merely to ensure that the end result was achieved. In other words, what was sought to be controlled by
the petitioner company was actually the end result of the task. The guidelines or the Comprehensive Medical Plan
were laid down merely to ensure that the desired end result was achieved but did not control the means and methods
by which respondent performed his assigned tasks.

The Supreme Court further held that, an employee is required to stay in the employer’s workplace or proximately
close thereto that he cannot utilize his time effectively and gainfully for his own purpose. Such is not the prevailing
situation here. The respondent does not dispute that fact that outside of the two (2) hours that he is required to be
at petitioner company’s premises, he is not at all further required to just sit around in the premises and wait for an
emergency to occur so as to enable him from using such hours for his own benefit and advantage. In fact,
respondent maintains his own private clinic attending his private practice in the city, where he services his patients
and bills them accordingly.

The Court finds that the requirement to be on call for emergency cases does not amount to such control, but are
necessary incidents to the Retainership Agreement. The Supreme Court also notes that the Agreement granted to
both parties the power to terminate their relationship upon giving a 30-day notice. Hence, petitioner did not wield the
sole power of dismissal or termination. Therefore, the petition was GRANTED.

8. Calamba Medical Center vs. NLRC et al., G.R. No. 176484, Nov. 25, 2008

PETITIONER: Calamba Medical Center RESPONDENT: Nat’l Labor Relations Commission,


Ronaldo Lanzanas and Merceditha Lanzanas

SPECIAL DETAILS:

LAW & PRINCIPLES:

FACTS: Calamba Medical Center, engaged the services of medical doctors-spouses Dr. Ronaldo and Dr. Merceditha
Lanzanas as part of its team of resident physicians. Reporting at the hospital twice-a-week on twenty-four-hour
shifts, respondents were paid a monthly "retainer" of P4,800.00 each. Also resident physicians were also given a
percentage share out of fees charged for out-patient treatments, operating room assistance and discharge billings,
in addition to their fixed monthly retainer.

Page 16 of 67
LABOR 1 | JMM | CASE DIGESTS 2020 – TOPICS 1-3

The work schedules of the members of the team of resident physicians were fixed by petitioner's medical director
Dr. Desipeda, and they were issued ID, enrolled in the SSS and withheld tax from them.

After an incident where Dr. Trinidad overheard a phone conversation between Dr. Ronaldo and a fellow employee
Diosdado Miscala, the former was given a preventive suspension and his wife Dr. Merceditha was not given any
schedule after sending the Memorandum. On March 1998, Dr. Ronaldo filed a complaint for illegal suspension and
Dr. Merceditha for illegal dismissal.

ISSUES: Whether or not there exists an employer-employee relationship between petitioner and the spouses-
respondents.

RULING: Drs. Lanzanas are declared employee by the petitioner hospital. Under the "control test," an employment
relationship exists between a physician and a hospital if the hospital controls both the means and the details of the
process by which the physician is to accomplish his task.

That petitioner exercised control over respondents gains light from the undisputed fact that in the emergency room,
the operating room, or any department or ward for that matter, respondents' work is monitored through its nursing
supervisors, charge nurses and orderlies. Without the approval or consent of petitioner or its medical director, no
operations can be undertaken in those areas. For control test to apply, it is not essential for the employer to actually
supervise the performance of duties of the employee, it being enough that it has the right to wield the power.
With respect to respondents' sharing in some hospital fees, this scheme does not sever the employment tie between
them and petitioner as this merely mirrors additional form or another form of compensation or incentive similar to
what commission-based employees receive as contemplated in Article 97 (f) of the Labor Code.

Moreover, respondents were made subject to petitioner-hospital's Code of Ethics - the provisions of which cover
administrative and disciplinary measures on negligence of duties, personnel conduct and behavior, and offenses
against persons, property and the hospital's interest.

More importantly, petitioner itself provided incontrovertible proof of the employment status of respondents, namely,
the identification cards it issued them, the pay slips and BIR W-2 (now 2316) Forms which reflect their status as
employees, and the classification as "salary" of their remuneration. Moreover, it enrolled respondents in the SSS and
Medicare (Philhealth) program. It bears noting at this juncture that mandatory coverage under the SSS Law is
premised on the existence of an employer-employee relationship, except in cases of compulsory coverage of the
self-employed.

NOTES:

9. Escasinas et al., vs. Shangri-las Mactan Island Resort et al., G.R. No. 178827, March 4, 2009

PETITIONER: Jerome D. Escasinas, Evan Rigor RESPONDENT: Shangri-la Mactan Island Resort and Dr. Jessia
Singco J.R. Pepito

SPECIAL DETAILS:

LAW & PRINCIPLES:

FACTS: Registered nurses Jeromie D. Escasinas and Evan Rigor Singco (petitioners) were engaged in 1999 and
1996, respectively, by Dr. Jessica Joyce R. Pepito (respondent doctor) to work in her clinic at respondent Shangri-
la’sMactan Island Resort (Shangri-la) in Cebu of which she was a retained physician.
In late 2002, petitioners filed with the National Labor Relations Commission (NLRC) a complaint for regularization,
underpayment of wages, non-payment of holiday pay, night shift differential and 13th month pay differential against
respondents, claiming that they are regular employees of Shangri-la.

Page 17 of 67
LABOR 1 | JMM | CASE DIGESTS 2020 – TOPICS 1-3

Shangri-la claimed, however, that petitioners were not its employees but of respondent doctor, that Article 157 of
the Labor Code, as amended, does not make it mandatory for a covered establishment to employ health personnel,
that the services of nurses is not germane nor indispensable to its operations, and that respondent doctor is a
legitimate individual contractor who has the power to hire, fire and supervise the work of nurses under her.

ISSUES: Whether or not there was an employee-employer relationship between Shangri-La and the petitioners. 2.)
Whether or not Dr. Pepito is an independent contractor

RULING: 1st. Issue: The resolution of the case hinges, in the main, on the correct interpretation of Art. 157 vis a vis
Art. 280 and the provisions on permissible job contracting of the Labor Code, as amended. Under the foregoing
provision, Shangri-la, which employs more than 200 workers, is mandated to “furnish” its employees with the
services of a full-time registered nurse, a part-time physician and dentist, and an emergency clinic which means that
it should provide or make available such medical and allied services to its employees, not necessarily to hire or
employ a service provider. The term “full-time” in Art. 157 cannot be construed as referring to the type of employment
of the person engaged to provide the services, for Article 157 must not be read alongside Art. 280[9] in order to vest
employer-employee relationship on the employer and the person so engaged. The phrase “services of a full-time
registered nurse” should thus be taken to refer to the kind of services that the nurse will render in the company’s
premises and to its employees, not the manner of his engagement.

2nd Issue: The existence of an independent and permissible contractor relationship is generally established by
considering the following determinants: whether the contractor is carrying on an independent business; the nature
and extent of the work; the skill required; the term and duration of the relationship; the right to assign the
performance of a specified piece of work; the control and supervision of the work to another; the employer's power
with respect to the hiring, firing and payment of the contractor's workers; the control of the premises; the duty to
supply the premises, tools, appliances, materials and labor; and the mode, manner and terms of payment.
Against the above-listed determinants, the Court holds that respondent doctor is a legitimate independent
contractor. That Shangri-la provides the clinic premises and medical supplies for use of its employees and guests
do not necessarily prove that respondent doctor lacks substantial capital and investment. Besides, the maintenance
of a clinic and provision of medical services to its employees is required under Art. 157, which are not directly related
to Shangri-la’s principal business – operation of hotels and restaurants.

NOTES:

10. Tongko vs. Manufacturer Life Insurance Co. (Phils), Inc., et al., G.R. No. 167622, January 25,
2011, En Banc, see June 29, 2010 Main Decision

PETITIONER: Gregorio v. Tongko RESPONDENT: The Manufacturers Life Insurance Co. (Phils.), Inc. and
(lost) Renato A. Vergel de Dios (won)

SPECIAL DETAILS: Evidence shows that Tongko’s role as an insurance agent never changed during his relationship
with Manulife. If changes occurred at all, the changes did not appear to be in the nature of their core relationship.
Tongko essentially remained an agent, but moved up in this role through Manulife’s recognition that he could use
other agents approved by Manulife, but operating under his guidance and in whose commissions he had a share.

Tongko lacks evidence on record showing that Manulife ever exercised means-and-manner control, even to a limited
extent, over Tongko during his ascent in Manulife’s sales ladder.

LAW & PRINCIPLES:

INSURANCE CODE

The Insurance Code imposes obligations on both the insurance company and its agents in the performance of their
respective obligations under the Code, particularly on licenses and their renewals, on the representations to be made
to potential customers, the collection of premiums, the delivery of insurance policies, the matter of representation,
and the measures to ensure ethical business practices in the industry.

Page 18 of 67
LABOR 1 | JMM | CASE DIGESTS 2020 – TOPICS 1-3

GENERAL LAW ON AGENCY

The general law on agency expressly allows the principal an element of control over the agent in a manner consistent
with an agency relationship. In this sense, these control measures cannot be read as indicative of labor law control.

Foremost among these are the directives that the principal may impose on the agent to achieve the assigned tasks,
to the extent that they do not involve the means and manner of undertaking these tasks. The law likewise obligates
the agent to render an account; in this sense, the principal may impose on the agent specific instructions on how an
account shall be made; particularly on the manner of expenses and reimbursements. To these extents, control can
be imposed through rules and regulations without intruding into the labor law concept of control for purposes of
employment.

FACTS: Petitioner Tongko was initially an insurance agent of respondent Manulife who was promoted to the role of
a manager. The contractual relationship between Tongko and Manulife had two basic phases.

The first phase under a Career Agent’s Agreement provided that it is understood and agreed that the Agent is an
independent contractor and nothing contained therein shall be construed or interpreted as creating an employer-
employee relationship between the Company and the Agent. As an agent, Tongko’s tasks were to canvas for
applications for insurance products and to collect money due to the company. The second phase was when Tongko
was named Unit Manager. Later on, he became a Branch Manager, and then subsequently a Regional Sales Manager.

Manulife instituted manpower development programs at the regional sales management level where the first step
was to transform Manulife into a “big league player” and to increase the number of agents to at least one thousand
(1,000) for a strong start.

It was later found out that Tongko’s region was the lowest performing region in terms of recruiting and he received
several letters with respect to this concern. However, Tongko still failed to align his directions with the
Management’s avowed agency growth police. Hence, he was terminated. Tongko then filed an illegal dismissal
complaint alleging that he was an employee of Manulife, thus, an employer-employee relationship existed.

The Labor Arbiter ruled in favor of Manulife while the National Labor Relations Commission (NLRC) ruled in favor
of Tongko (that there, in fact, existed an employer-employee relationship). On the other hand, the Court of Appeals
upheld the decision of the Labor Arbiter.

ISSUES: Whether or not there was an employer-employee relationship.

RULING: By the agreement’s express terms, Tongko served as an “insurance agent” for Manulife, not as an
employee. From this perspective, the provisions of the Insurance Code cannot be disregarded as this Code expressly
envisions a principal-agent relationship between the insurance company and the insurance agent in the sale of
insurance to the public. For this reason, judicial notice may be taken that as a matter of Insurance Code-based
business practice, an agency relationship prevails in the insurance industry for the purpose of selling insurance.

From jurisprudence, an important lesson that can be gleaned from the first Insular Life case is that a commitment
to abide by the rules and regulations of an insurance company does not ipso facto make the insurance agent an
employee. Neither do guidelines somehow restrictive of the insurance agent’s conduct necessarily indicate “control”
as this term is defined in jurisprudence.

Guidelines indicative of labor law “control,” as is mentioned in the first Insular Life case, should not merely relate to
the mutually desirable result intended by the contractual relationship; they must have the nature of dictating the
means or methods to be employed in attaining the result, or of fixing the methodology and of binding or restricting
the party hired to the use of these means.

The hallmarks of an employer-employee relationship in the management system established are: exclusivity of
service, control of assignments and removal of agents under the private respondent’s unit, and furnishing of
company facilities and materials as well as capital described as a Unit Development Fund. All of these are obviously
absent in the present case.

Page 19 of 67
LABOR 1 | JMM | CASE DIGESTS 2020 – TOPICS 1-3

NOTES:

ELEMENT OF CONTROL

In an agency relationship, the principal is allowed to have an element of control over his agent without intruding into
the labor law concept of control for the purposes of employment.

11. Semblante et al., vs. Court of Appeals, et al., G.R. No. 196426, August 15, 2011

PETITIONER: Marticio Semblante RESPONDENT: Court of Appeals, 19th Division (now Special Former 19th
and Dubrick Pilar (lost) Division), Gallera de Mandaue/Spouses Vicente, and Maria Luisa Loot (won)

SPECIAL DETAILS: According to the existence of employer-employee relationship and application of the four-fold
test, the petitioners are not employees of respondents. This is because the petitioners perform their duties through
their expertise and knowledge free from direction and control of the respondents.

LAW & PRINCIPLES:

FOUR-FOLD TEST OF EMPLOYMENT

The elements of the four-fold test are the following:

1) The selection and engagement of the employee;


2) The payment of wages;
3) The power of dismissal; and
4) The power to control employee’s conduct, which is the most important element.

FACTS: This is a petition for Certiorari under Rule 45 assailing and seeking to set aside the decision and resolution
dated May 29, 2009 and February 23, 2010, respectively, of the Court of Appeals. The Court of Appeals later on
affirmed the October 18, 2006 Resolution of the NLRC.

Petitioners Semblante and Pilar assert that there were hired by respondent-spouses Vicente and Maria Luisa Loot,
the owners of Gallera de Mandaue, which is a cockpit, as the official masiador and sentenciador, respectively, of the
cockpit sometimes in 1993.

Masiador Semblante’s job was to call and take bets from the gamecock owners and bettors, as well as to order the
start of the fight. He also distributed the winnings after deducting the “arriba”, or the commission of the cockpit.

On the other hand, sentenciador Pilar oversaw the proper gaffing of fighting cocks, determined the physical condition
of the cocks, and the capabilities of the cocks to continue fighting. He also declared the results.

For their services as masiador and sentenciador, Semblante received P2,000.00 per week or a total of P8,000.00 per
month, while Pilar received P3,500.00 a week or P14,000.00 per month.

They worked on Tuesday, Wednesday, Saturday, and Sunday of every week, excluding monthly derbies and
cockfights held on special holidays. Their working days started at 1 P.M. and continued until 12 A.M., or until the
early hours of the morning depending on the needs of the cockpit. Petitioners were also issued employees’ ID cards
that they wore while working. They alleged that they had never violated any rules of the cockpit.

On November 14, 2003, the petitioners were denied entry to the cockpit upon the instruction of the respondents, and
were informed of their termination effective on that date. This prompted the petitioners to file a complaint for illegal
dismissal.

Respondents denied that petitioners were their employees and alleged that they were associates of respondents’
independent contractors Tomas Vega. Respondents also claimed that the petitioners did not have regular working

Page 20 of 67
LABOR 1 | JMM | CASE DIGESTS 2020 – TOPICS 1-3

hours, and were free to decide if they were going to report in for work or not. The ID cards were only given to them
to indicate that they were free from the normal entrance fee.

On June 16, 2004, Labor Arbiter Julie Rendoque found petitioners to be regular employees of respondents as they
performed work that was necessary and indispensable to the business of respondents. She also ruled that they were
illegally dismissed and were therefore entitled to backwages and separation pay.

The counsel for the respondents received the LA’s decision on September 14, 2004 and on September 24, 2004,
within the 10-day appeal period, he filed an appeal with the NLRC. However, counsel filed the appeal without posting
a cash or surety bond equivalent to the monetary award. It was only on October 11, 2004 that they filed the appeal
bond. Hence, in a Resolution dated August 25, 2005, the NLRC denied the appeal for non-perfection. However, in a
motion for reconsideration, the NLRC reversed its Resolution on the postulate that their appeal was meritorious and
the filing of an appeal bond, albeit belated, is a substantial compliance with the rules. Thus the NLRC held that there
was no employer-employee relationship.

The Court of Appeals also decided the case in favor of the respondents, as they believed petitioners to be akin to
independent contractors who possess unique skills and talents to distinguish them from ordinary employees.

ISSUES: Whether or not there was an employer-employee relationship.

RULING: No, there was no employer-employee relationship in this case. This is because the relationships of the
employer-employee failed to pass the four-fold test of employment.

As found by both the NLRC and the CA, respondents had no part in petitioners’ selection and management. Their
compensation was also paid out of the arriba, which is a percentage deducted from tbe bets, and not by the
petitioners. Lastly, the petitioners performed their functions as masiador and sentenciador free from the direction
and control of respondents. Petitioners relied mainly on their “expertise that is characteristic of cockfight gambling”.

Respondents, not being petitioners’ employers, could never have dismissed, legally or illegally, petitioners, since
respondents were without power and prerogative to do so.

NOTES:

PROCEDURAL ASPECT OF THE NON-POSTING OF AN APPEAL BOND

The Court held that the procedural aspect of the non-posting of an appeal bond should not defeat the substantive
rights of respondent to be free from the unwarranted burden of answering for an illegal dismissal for which they
were never responsible.

12. Bernarte vs. Phil. Basketball Association et al., G.R. No. 192084, September 14, 2011

PETITIONER: Jose Mel Bernarte (lost) RESPONDENT: Philippine Basketball Association (PBA),
Jose Emmanuel M. Eala, and Perry Martinez (won)

SPECIAL DETAILS: The fact that the PBA repeatedly hired the petitioner does not, by itself, prove that the petitioner
is an employee of the former. For a hired party to be considered an employee, the hiring party must have control
over the means and methods by which the hired party is to perform his work, which is absent in this case.

LAW & PRINCIPLES:

DISTINCTION BETWEEN INDEPENDENT CONTRACTOR AND EMPLOYEE

Repeated hiring does not prove the existence of an employer-employee relationship. What is important is that there
is control over the means and methods by which the hired party must perform his duty.

Page 21 of 67
LABOR 1 | JMM | CASE DIGESTS 2020 – TOPICS 1-3

FACTS: Complainants Jose Mel Bernarte and Renato Guevarra aver that they were invited to join the PBA as referees.
During the leadership of Commissioner Emilio Bernardino, they were made to sign contracts on a year-to-year basis.
During the term of Commissioner Eala, however, changes were made regarding the terms of their employment.

Complainant Bernarte, for instance, was not made to sign a contract during the first conference of the All-Filipino
Cup which was from February 23, 2003 to June 2003. It was only during the second conference when he was made
to sign a one-and-a-half month contract for the period July 1 to August 5, 2003.

On January 15, 2004, Bernarte received a letter from the Office of the Commissioner advising him that his contract
would not be renewed citing his unsatisfactory performance on and off the court. It was a total shock for Bernarte
who was awarded Referee of the Year in 2003. He felt that the dismissal was caused by his refusal to fix a game
upon the order of Ernie De Leon.

On the other hand, complainant Guevarra alleges that he was invited to join the PBA pool of referees in February
2001. On March 1, 2001, he signed a contract as a trainee. Beginning 2002, he signed a yearly contract as a Regular
Class C referee. On May 6, 2003, respondent Martinez issued a Memorandum to Guevarra expressing dissatisfaction
over his question on the assignment of referees officiating out-of-town games. Beginning February 2004, he was no
longer made to sign a contract.

Respondents aver, on the other hand, that complainants entered into two contracts of retainer with the PBA in the
year 2003. The first contract was for the period January 1, 2003 to July 15, 2003; and the second was for September
1 to December 2003. After the lapse of the latter period, PBA decided not to renew their contracts.

The Labor Arbiter and the NLRC declared petitioner an employee whose dismissal was illegal. They also ordered the
reinstatement and the payment of backwages, and of moral and exemplary damages, and attorney’s fees.

However, the Court of Appeals overturned the decisions of the Labor Arbiter and the NLRC as the respondents did
not exercise any form of control over the means and methods by which the petitioner performed his work as a
basketball referee.

ISSUES: Whether or not petitioner is an employee of the respondents.

RULING: No, as the existence of an employer-employee relationship is ultimately a question of fact. As a general
rule, factual issues are beyond the province of the Supreme Court. However, this rule admits of exceptions, one of
which is where there are conflicting findings of fact between the Court of Appeals, on one hand, and the NLRC and
Labor Arbiter on the other, such as in the present case.

To determine the existence of an employer-employee relationship, case law has consistently applied the four-fold
test, to wit: (a) the selection and engagement of the employee; (b) the payment of wages; (c) the power of dismissal;
and (d) the employer’s power to control the employee on the means and methods by which the work is
accomplished. The so-called “control test” is the most important indicator of the presence or absence of an
employer-employee relationship.

The Court further agrees with the respondents that once in the playing court, the referees exercise their own
independent judgment, based on the rules of the game, as to when and how a call or decision is to be made. The
referees decide whether an infraction was committed, and the PBA cannot overrule them once the decision is made
on the playing court. The referees are the only, absolute, and final authority on the playing court. Respondents, or
any of the PBA officers cannot and do not determine which calls to make or not make and cannot control the referee
when he blows the whistle because such authority exclusively belongs to the referees. The very nature of the
petitioner’s job of officiating a professional basketball game undoubtedly calls for freedom of control by the
respondents.

Moreover, the following circumstances indicate that petitioner is an independent contractor: (1) the referees are
required to report for work only when PBA games are scheduled, which is three (3) times a week, spread over an
average of only 105 playing days a year, and they officiate games at an average of two (2) hours per game; and (2)
the only deductions from the fees received by the referees are the withholding taxes. Thus, the non-renewal of the
contract between the parties does not constitute illegal dismissal of the petitioner by the respondents.

Page 22 of 67
LABOR 1 | JMM | CASE DIGESTS 2020 – TOPICS 1-3

NOTES:

REPEATED HIRING IS NOT, BY ITSELF, PROOF OF EMPLOYMENT

The fact that the PBA repeatedly hired the petitioner does not, by itself, prove that the petitioner is an employee of
the former. For a hired party to be considered an employee, the hiring party must have control over the means and
methods by which the hired party is to perform his work, which is absent in this case.

The continuous rehiring by the PBA of petitioner simply signifies the renewal of the contract between the PBA and
the petitioner, and highlights the satisfactory services rendered by the petitioner warranting such contract renewal.

Conversely, if the PBA decided to discontinue the petitioner’s services at the end of the term fixed in the contract,
whether for unsatisfactory services, or for violation of the terms and conditions of the contract, or for whatever other
reason, the same merely results in the non-renewal of the contract, as in the present case.

13. Lirio vs. Genovia, G.r. No. 169757, November 23, 2011

PETITIONER: Cesar S. Lirio, doing business under the RESPONDENT:Wilner D. Genovia (WON)
name and style of CELKOR AD SONIXMIX (LOST)

SPECIAL DETAILS:
● Respondent Genovia worked as a manager of CELKOR AD SONIXMIX and agreed to produce an album with
the Petitioner for his daughter, Celine Mei Lirio (former ABS-CBN Star Records Talent).
● The respondent arranged and composed songs for the petitioner’s daughter.

LAW & PRINCIPLES:


● If doubts exist between the evidence presented by the employer and the employee, the scales of justice
must be tilted in favor of the latter. It is a time-honored rule that in controversies between a laborer and his
master, doubts reasonably arising from the evidence, or in the interpretation of agreements and writing
should be resolved in the former's favor.
● Before a case for illegal dismissal can prosper, it must first be established that an employer-employee
relationship existed between petitioner and respondent.
● The elements to determine the existence of an employment relationship are: (a)the selection and
engagement of the employee; (b) the payment of wages; (c) the power of dismissal; and (d) the employer's
power to control the employee's conduct.
● The most important element is the employer's control of the employee's conduct, not only as to the result
of the work to be done, but also as to the means and methods to accomplish it.

FACTS:
● On July 9, 2002, respondent Wilmer D. Genovia filed a complaint against petitioner Cesar Lirio and/or Celkor
Ad Sonicmix Recording Studio for illegal dismissal, non-payment of commission and award of moral and
exemplary damages.
● Respondent alleged that on August 15, 2001 he was hired as the studio manager of CELKOR by Lirio. He
operates, promotes and sells the recording studio’s services to clients.
● He received a monthly salary of Php7,000 and reported to work from Mondays to Fridays from 9:00am to
6:00PM. On Saturday he was required to work half day only.
● Respondent stated that after a few days of working in the studio, petitioner approached him and asked to
compose and arrange songs for his daughter, Celine Mei Lirio and the petitioner also promised to draft a
contract to assure the respondent of the compensation for such services.
● As agreed upon, the additional services of arranging and composing songs will only be done during
respondent’s spare time since his other work as a studio manager is the priority.
● From December 2001 to January 2002, respondent, in his capacity as studio manager, worked on digital
editing, mixing and sound engineering of the vocal and instrumental audio files. The album of Lirio’s
daughter was done by early February of 2002 and began airing on the radio by the end of February.

Page 23 of 67
LABOR 1 | JMM | CASE DIGESTS 2020 – TOPICS 1-3

● On February 26, 2002 the respondent reminded the petitioner of the contract on his compensation as
composer and arranger of the album. The petitioner told the respondent that he was a nobody and proved
nothing in the industry and did not deserve high compensation. Petitioner also informed the respondent
that he was only entitled 20% of the net profit share of the album and the salaries he received and would
continue to receive as studio manager of Celkor would be deducted from the said 20% net profit share.
● Respondent objected and on March 14, 2002 petitioner verbally terminated respondent’s services and
instructed him not to report to work anymore.
● Respondent alleged that he was illegally dismissed as he was terminated without any valid grounds and no
hearing was conducted before he was terminated in violation of due process.
● Petitioner on the other hand contends that respondent was not his employee in the studio and that the
respondent was his partner. Their relationship was that of an informal partnership under Article 1767 of
NCC.
● Labor Arbiter Renaldo O. Hernandez rendered a decision, finding that an employer-employee relationship
existed between petitioner and respondent, and that respondent was illegally dismissed. Labor Arbiter
stated that petitioner's denial of the employment relationship cannot overcome respondent's positive
assertion and documentary evidence (receipt of monthly compensation shown from payroll corrected by
the petitioner) proving that petitioner hired respondent as his employee.
● The petitioner appealed the decision to NLRC. It set aside the decision of the labor arbiter on the ground
that the respondent failed to prove with substantial evidence that he was the petitioner's employee.
● The Court of Appeals reversed the NLRC’s decision stating that between the documentary evidence
presented by respondent and the mere allegation of petitioner without any proof by way of any document
evincing their alleged partnership agreement, the Court of Appeals agreed with the Labor Arbiter.
● Hence, this petition in the Supreme Court.

ISSUES: 1. Whether the Court of Appeals erred in reversing and setting aside the decision of the NLRC, and
reinstating the decision of the Labor Arbiter with modification that (1) there is an employer-employee relationship
existed between the respondent and the petitioner and that the respondent was illegally dismissed.

RULING: No, the Court of Appeals did not err in reversing and setting aside the decision of the NLRC.

1. There is an employer-employee relationship between the petitioner and the respondent. In this case, the
documentary evidence presented by respondent to prove that he was an employee of petitioner are as
follows: (a) a document denominated as "payroll" (dated July 31, 2001 to March 15, 2002) certified correct
by petitioner , which showed that respondent received a monthly salary of P7,000.00 with the corresponding
deductions due to absences incurred by respondent; and (2) copies of petty cash vouchers showing the
amounts he received and signed for in the payrolls. Petitioner wielded the power to dismiss as respondent
stated that he was verbally dismissed by petitioner, and lastly the petitioner had the power to check on the
progress and work of respondent as he agreed to teach and help the respondent in working with the studio
equipment.
2. The respondent was illegally dismissed. Article 277 (b) of the Labor Code puts the burden of proving that
the dismissal of an employee was for a valid or authorized cause on the employer, without distinction
whether the employer admits or does not admit the dismissal. For an employee's dismissal to be valid, (a)
the dismissal must be for a valid cause, and (b) the employee must be afforded due process. Procedural
due process requires the employer to furnish an employee with two written notices before the latter is
dismissed: (1) the notice to apprise the employee of the particular acts or omissions for which his dismissal
is sought, which is the equivalent of a charge; and (2) the notice informing the employee of his dismissal,
to be issued after the employee has been given reasonable opportunity to answer and to be heard on his
defense. Petitioner failed to comply with these legal requirements; hence, the Court of Appeals correctly
affirmed the Labor Arbiter's finding that respondent was illegally dismissed.

NOTES:
Employer-Employee Relationship (Four Fold Test)
The elements to determine the existence of an employment relationship are: (a) the selection and
engagement of the employee; (b) the payment of wages; (c) the power of dismissal; and (d) the employer's power
to control the employee's conduct. It is settled that no particular form of evidence is required to prove the existence
of an employer-employee relationship. Any competent and relevant evidence to prove the relationship may be
admitted.

Page 24 of 67
LABOR 1 | JMM | CASE DIGESTS 2020 – TOPICS 1-3

Illegal Dismissal
Article 277 (b) of the Labor Code puts the burden of proving that the dismissal of an employee was for a
valid or authorized cause on the employer, without distinction whether the employer admits or does not
admit the dismissal. For an employee's dismissal to be valid, (a) the dismissal must be for a valid cause,
and (b) the employee must be afforded due process. Procedural due process requires the employer to
furnish an employee with two written notices before the latter is dismissed: (1) the notice to apprise the
employee of the particular acts or omissions for which his dismissal is sought, which is the equivalent of a
charge; and (2) the notice informing the employee of his dismissal, to be issued after the employee has
been given reasonable opportunity to answer and to be heard on his defense.

14. Jao vs. BCC Products Sales Inc. G.R. No. 163700, April 18, 2012

PETITIONER: Charlie Jao (LOST) RESPONDENT: BCC Products Sales, Inc. and Terrence
Ty (WON)

SPECIAL DETAILS: Here, the petitioner believes that he was employed by BCC products Sales, Inc. until one day he
was barred by security guards from entering the premises of the said company.

LAW & PRINCIPLES:


● In determining the presence or absence of an employer-employee relationship, the Court has consistently
looked for the following incidents, to wit: (a) the selection and engagement of the employee; (b) the
payment of wages; (c) the power of dismissal; and (d) the employer's power to control the employee on the
means and methods by which the work is accomplished. The last element, the so-called control test, is the
most important element.

FACTS:
● Petitioner in this case maintained that he was an employee of BCC Products Sales, Inc. owned by
respondent Terrence Ty. He was allegedly employed as comptroller starting from September 1995 with a
monthly salary of Php20,000 to handle the financial aspect of BCC’s business.
● His attempts to report to work in November and December 12, 1995 were frustrated because he continued
to be barred from entering the premises of BCC;
● He filed a complaint dated December 28, 1995 for illegal dismissal, reinstatement with full backwages, non-
payment of wages, damages and attorney's fees.
● Respondents countered that petitioner was not their employee but the employee of Sobien Food
Corporation (SFC), the major creditor and supplier of BCC; and that SFC had posted him as its comptroller
in BCC to oversee BCC's nances and business operations and to look after SFC's interests or investments
in BCC.
● The first labor arbiter that handled the case ruled in favor of the petitioner but the NLRC vacated the ruling
and remanded the case for further proceedings. The second labor arbiter, Jovencio Mayor rendered a
decision on September 20, 2001 dismissing the case of the petitioner for want of employer-employee
relationship. The NLRC on the other hand reversed the decision of Mayor and declared that the petitioner
had been illegally dismissed.
● Respondents moved for reconsideration of the NLRC Decision but it was denied. They assailed the decision
of the NLRC on certiorari in the CA.
● The Court of Appeals agreed with the labor arbiter that there is no employer-employee relationship between
the petitioner and the respondent. there is no proof that the services of the private respondent were
engaged to perform the duties of a comptroller in the petitioner company. There is no proof that the private
respondent has undergone a selection procedure as a standard requisite for employment, especially with
such a delicate position in the company. Neither is there any proof of his appointment nor is there any
showing that the parties entered into an employment contract, stipulating thereof that he will receive
P20,000.00/month salary as comptroller, before the private respondent commenced with his work as such.
Second,the private respondent was not included in the petitioner company's payroll during the time of his
alleged employment with the former.

Page 25 of 67
LABOR 1 | JMM | CASE DIGESTS 2020 – TOPICS 1-3

ISSUES: Whether or not the petitioner was the respondent's employee.

RULING: No, the petitioner was not an employee of BCC. Based on the submitted evidence of the petitioner, he was
installed by Sobien Food Corporation (SFC) as a comptroller in BCC to oversee and supervise SFC’s collections and
the account of BCC to protect SFC’s interest. The evidence shown by the petitioner as proof of employment (ID,
checks carrying the signature of TY etc.) does show that petitioner’s presence in the premises of BCC was due to
the arrangement or agreement between BCC and SFC for the latter to appoint a comptroller to oversee the former's
operations. The affidavit shown to the court actually supported the contention that petitioner had really worked in
BCC as SFC's representative. It does seem more natural and more believable that petitioner's affidavit was referring
to his employment by SFC even while he was reporting to BCC as a comptroller on behalf of SFC.

NOTES:

The "control test," under which the person for whom the services are rendered reserves the right to direct not only
the end to be achieved but also the means for reaching such end, is generally
relied on by the courts in determining employer-employee relationship.

15. Legend Hotel (Manila) vs. Realuyo G.R. No. 153511, July 18, 2012

PETITIONER:LEGEND HOTEL (MANILA), owned by RESPONDENT:HERNANI S. REALUYO, also


TITANIUM CORPORATION, known as JOEY ROA
and/or, NELSON NAPUD, in his capacity as the (WON)
President of Petitioner Corporation (LOST)

SPECIAL DETAILS: Hernani S. Realuyo aka Joey Roa was an artist who performs as a pianist in Legend
Hotel’s Madison Coffee Shop/Tanglaw Restaurant.

LAW & PRINCIPLES:


The issue of whether or not an employer-employee relationship existed between petitioner
and respondent is essentially a question of fact. The factors that determine the issue
include who has the power to select the employee, who pays the employee's wages, who
has the power to dismiss the employee, and who exercises control of the methods and
results by which the work of the employee is accomplished.

FACTS:
● Respondent filed a complaint for alleged unfair labor practice, constructive dismissal and underpayment/
non payment of his premium pay for holidays,
● He worked as a pianist at Legend Hotel’s Tanglaw Restaurant from September 1992 until July 1999 when
the management had notified him that he is no longer required to work as a pianist in the restaurant due
to retrenchment. Because of this, he filed a complaint.
● The petitioner denied the existence of employer-employee relationship with respondent. In its defense,
petitioner denied the existence of an employer-employee relationship with respondent, insisting that he had
been only a talent engaged to provide live music at Legend Hotel's Madison Coffee Shop for three
hours/day on two days each week; and
stated that the economic crisis that had hit the country constrained management to dispense with his
services.
● The labor arbiter dismissed the complaint for lack of merit upon finding that the parties has no employer-
employee relationship because the respondent only receives talent fee and not salary.
● Respondent appealed, but the NLRC affirmed the LA on May 31, 2001. Respondent assailed the decision
of the NLRC in the Court of Appeals (CA) on certiorari.

Page 26 of 67
LABOR 1 | JMM | CASE DIGESTS 2020 – TOPICS 1-3

● On February 11, 2002, the CA set aside the decision of the NLRC stating that respondent failed to take into
consideration that in petitioner's line of work, he was supervised and controlled by respondent's restaurant
manager who at certain times would require him to perform only tagalog songs or music, or wear barong
tagalog to conform with Filipiniana motif of the place and the time of his performance is fixed by the
respondents from
7:00 pm to 10:00 pm, three to six times a week. Petitioner could not choose the time of his performance.
As to the status of petitioner, he is considered a regular employee of private respondents since the job of
the petitioner was in furtherance of the restaurant business of respondent hotel.
● The petitioner appealed to the Supreme Court. The petitioner also argued that only talent fees are given to
the respondent and not wage as defined in the Labor Code and that the respondent did not work 8 hours a
day as required number of working hours under the labor code.

ISSUES: Whether or not the respondent is an employee of Legend Hotel.

RULING: Yes, the respondent is an employee of Legend Hotel. A review of the circumstances reveals that respondent
was, indeed, petitioner's employee. He was undeniably employed as a pianist in petitioner's Madison Coffee
Shop/Tanglaw Restaurant from September 1992 until his services were terminated on July 9, 1999.

First of all, petitioner actually wielded the power of selection at the time it entered into the service contract dated
September 1, 1992 with respondent. This is true, notwithstanding petitioner's insistence that respondent had only
offered his services to provide live music at petitioner's Tanglaw Restaurant, and despite petitioner's position that
what had really transpired was a negotiation of his rate and time of availability. The power of selection was firmly
evidenced by, among others, the express written recommendation dated January 12, 1998 by Christine Velazco,
petitioner's restaurant manager, for the increase of his remuneration.

Respondent's remuneration, albeit denominated as talent fees, was still considered as included in the term wage in
the sense and context of the Labor Code, regardless of how petitioner chose to designate the remuneration. Anent
this, Article 97 (f) of the Labor Code clearly states:

. . . wage paid to any employee shall mean the remuneration or earnings, however designated, capable of being
expressed in terms of money, whether fixed or ascertained on a time, task, piece, or commission basis, or other
method of calculating the same, which is payable by an employer to an employee under a written or unwritten
contract of employment for work done or to be done, or for services rendered or to be rendered, and includes the
fair and reasonable value, as determined by the Secretary of Labor, of board, lodging, or other facilities customarily
furnished by the employer to the employee. Clearly, respondent received compensation for the services he rendered
as a pianist in
petitioner's hotel.

That respondent worked for less than eight hours/day was of no consequence and did not detract from the CA's
finding on the existence of the employer-employee relationship. In providing that the "normal hours of work of any
employee shall not exceed eight (8) hours a day," Article 83 of the Labor Code only set a maximum number of hours
as "normal hours of work" but did not prohibit work of less than eight hours.

Respondent performed his work as a pianist under petitioner's supervision and control under the following manner
to wit:
a. He could not choose the time of his performance, which petitioners
had fixed from 7:00 pm to 10:00 pm, three to six times a week; IDEScC
b. He could not choose the place of his performance;
c. The restaurant's manager required him at certain times to perform
only Tagalog songs or music, or to wear barong Tagalog to conform
to the Filipiniana motif; and
d. He was subjected to the rules on employees' representation check
and chits, a privilege granted to other employees.

Page 27 of 67
LABOR 1 | JMM | CASE DIGESTS 2020 – TOPICS 1-3

NOTES:

16. The New Philippine Skylanders, Inc., vs. Dakila, G.R. No. 199547, Sept. 24, 2012

PETITIONER: The New Philippine Skylanders RESPONDENT: Francisco Dakila

Respondent was a consultant and not their regular employee. He The consultancy contract was a scheme to
was not included in petitioners’ payroll and paid a fixed amount deprive him of the benefits of regularization,
under the consultancy contract. claiming to have assumed tasks necessary and
desirable in the trade or business of petitioners
He was not required to observe regular working hours and was and under their direct control and supervision.
free to adopt means and methods to accomplish his task except
as to the results of the work required of him. Hence, no
employer–employee relationship existed between them.
Moreover, respondent terminated his contract in a letter in April
2007, thus, negating his dismissal.

SPECIAL DETAILS:

LAW & PRINCIPLES:


● Art. 279 of Labor Code an employee who is unjustly dismissed from is entitled to reinstatement without
loss of seniority rights and other privileges and to his full backwages computed from time he was illegally
dismissed.
● Rule 45 of the Rules of Court

FACTS:
● Respondent was employed by petitioner corporation as early as 1987 and terminated for cause in 1997
when the corporation was sold. In May 1997, he was rehired as consultant by the petitioners under a
Contract for Consultancy Services. Thereafter, in a letter in April 2007, respondent informed petitioners of
his compulsory retirement effective on May 2 and sought for the payment of his retirement benefits
pursuant to the Collective Bargaining Agreement. His request was not acted upon, and instead was
terminated from service on May 1.
● Consequently, respondent filed a complaint for constructive illegal dismissal, nonpayment of retirement
benefits, under/non–payment of wages and other benefits of a regular employee, and damages against
petitioner corporation, its president/general manager, before the NLRC.

Ruling of Labor Arbiter


● Respondent have been illegally dismissed and ordered his reinstatement with full backwages computed
from the time of his dismissal on May 1 until his actual reinstatement as well as payment of his unpaid
benefits under CBA. Respondent was a regular employee on the basis of the unrebutted documentary
evidence showing that he was under the petitioners’ direct control and supervision and performed tasks
that were either incidental or usually desirable and necessary in the trade or business of petitioner
corporation for 10 years.
Ruling of NLRC
● Sustained LA’s finding that respondent was a regular and his dismissal was illegal. However, since he was
already beyond the retirement age, his reinstatement was no longer feasible. As such, it ordered the
payment of his retirement pay to be computed from 1997 until the date of the decision. Moreover, it found
respondent entitled to reinstatement wages from the time petitioners received a copy of LA’s decision up
to the date of NLRC’s decision.
Ruling of CA
● CA dismissed the petition for failure to show that the NLRC committed grave abuse of discretion in
affirming the LA’s decision. It found the factual findings of LA and NLRC to be supported by substantial
evidence and thus, should be accorded respect and finality.

Page 28 of 67
LABOR 1 | JMM | CASE DIGESTS 2020 – TOPICS 1-3

ISSUES: W/N respondent was regular employee and thus was illegally dismissed, and is entitled to reinstatement,
retirement benefits and full backwages

RULING:
● The issue of illegal dismissal is premised on the existence of an employer–employee relationship between
the parties. It is essentially a question of fact, beyond the ambit of a petition for review on certiorari under
Rule 45 of the Rules unless there is a clear showing of palpable error or arbitrary disregard of evidence
which does not obtain in this case. Records reveal that both LA and NLRC, as affirmed by CA, have found
substantial evidence to show that respondent was a regular employee who was dismissed without cause.
● Art. 279 of Labor Code an employee who is unjustly dismissed from is entitled to reinstatement without
loss of seniority rights and other privileges and to his full backwages computed from time he was illegally
dismissed. However, since respondent was terminated 1 day prior to his compulsory retirement, his
reinstatement is no loger feasible.
● NLRC correctly held him entitled to the payment of his retirement benefits pursuant to CBA. On the other
hand, his backwages should be computed only for days prior to his compulsory retirement which in this
case is only a day.
● There’s no basis to hold the President/General Managerjointly and severally liable with the corporation for
the payment of the monetary awards. The mere lack of authorized or just cause to terminate one’s
employment and the failure to observe due process do not ipso facto mean that a corporate officer acted
with malice or bad faith. There must be independent proof of malice or bad faith which was not established
in this case.
● Perforce, petitioner president cannot be made personally liable for the liabilities of the corporation which,
by legal fiction, has a personality separate and distinct from its officers, stockholders and members.
Moreover, for lack of factual and legal bases, the awards of moral and exemplary damages cannot also be
sustained.

NOTES:
● The president/general manager was absolved from paying respondent’s monetary awards
● Awards for reinstatement wages as well as moral and exemplary damages are deleted
● Computation of backwages was only limited for only 1 day prior to his compulsory retirement

17. Tesoro et al., vs. Metro Manila Retreaders Inc., et al., GR No. 171482, March 12, 2014

PETITIONER: Ashmor M. Tesoro, Pedro Ang, and RESPONDENT: Metro Manila Retreaders, Inc., Northern
Gregorio Sharp Luzon Retreaders, Inc., or Power Tire and Rubber
Corporation
Notwithstanding the execution of the SFAs, they
remained to be Bandag's employees, the SFAs being but Petitioners freely resigned from their employment and
a circumvention of their status as regular employees. decided to avail themselves of the opportunity to be
independent entrepreneurs under the franchise scheme
that Bandag had. Thus, no employer-employee
relationship existed between petitioners and Bandag.

SPECIAL DETAILS:

LAW & PRINCIPLES:

The tests for determining employer-employee relationship are: (a) the selection and engagement of the employee;
(b) the payment of wages; (c) the power of dismissal; and (d) the employer's power to control the employee with
respect to the means and methods by which the work is to be accomplished. The last is called the "control test," the
most important element

FACTS:
● On various dates between 1991 and 1998, petitioners Tesoro, Ang, and Sharp used to work as salesmen
for respondents Metro Manila Retreaders, Inc., Northern Luzon Retreaders, Inc., or Power Tire and Rubber

Page 29 of 67
LABOR 1 | JMM | CASE DIGESTS 2020 – TOPICS 1-3

Corporation, apparently sister companies, collectively called "Bandag." Bandag offered repair and retread
services for used tires. In 1998, however, Bandag developed a franchising scheme that would enable others
to operate tire and retreading businesses using its trade name and service system.

● Petitioners quit their jobs as salesmen and entered into separate Service Franchise Agreements ith Bandag
for the operation of their respective franchises. Under the SFAs, Bandag would provide funding support to
the petitioners subject to a regular or periodic liquidation of their revolving funds. The expenses out of these
funds would be deducted from petitioners' sales to determine their incomes.

● At first, petitioners managed and operated their respective franchises without any problem. After a length
of time, however, they began to default on their obligations to submit periodic liquidations of their
operational expenses in relation to the revolving funds Bandag provided them. Consequently, Bandag
terminated their respective SFA.
● Aggrieved, petitioners filed a complaint for constructive dismissal, non-payment of wages, incentive pay,
13th month pay and damages against Bandag with the NLRC

ISSUES: W/N petitioners remained to be Bandag's salesmen under the franchise scheme it entered into with them.

RULING:
● Franchising is a business method of expansion that allows an individual or group of individuals to market
a product or a service and to use of the patent, trademark, trade name and the systems prescribed by the
owner. In this case, Bandag's SFAs created on their faces an arrangement that gave petitioners the privilege
to operate and maintain Bandag branches in the way of franchises, providing tire repair and retreading
services, with petitioners earning profits based on the performance of their branches.

● The question is: did petitioners remain to be Bandag's employees after they began operating those
branches? When petitioners agreed to operate Bandag's franchise branches in different parts of the
country, they knew that this substantially changed their former relationships. They were to cease working
as Bandag's salesmen, the positions they occupied before they ventured into running separate Bandag
branches. They were to cease receiving salaries or commissions. Their incomes were to depend on the
profits they made. Yet, petitioners did not then complain of constructive dismissal. They took their chances,
ran their branches, Gregorio Sharp in La Union for several months and Ashmor Tesoro in Baguio and Pedro
Ang in Pangasinan for over a year. Clearly, their belated claim of constructive dismissal is quite hollow.

● It is pointed out that Bandag continued, like an employer, to exercise control over petitioners' work. It points
out that Bandag: (a) retained the right to adjust the price rates of products and services; (b) imposed
minimum processed tire requirement (MPR); (c) reviewed and regulated credit applications; and (d)
retained the power to suspend petitioners' services for failure to meet service standards.

● But uniformity in prices, quality of services, and good business practices are the essence of all franchises.
A franchisee will damage the franchisor's business if he sells at different prices, renders different or inferior
services, or engages in bad business practices. These business constraints are needed to maintain
collective responsibility for faultless and reliable service to the same class of customers for the same
prices.

● This is not the "control" contemplated in employer-employee relationships. Control in such relationships
addresses the details of day to day work like assigning the particular task that has to be done, monitoring
the way tasks are done and their results, and determining the time during which the employee must report
for work or accomplish his assigned task.

● Franchising involves the use of an established business expertise, trademark, knowledge, and training. As
such, the franchisee is required to follow a certain established system. Accordingly, the franchisors may
impose guidelines that somehow restrict the petitioners' conduct which do not necessarily indicate
"control." The important factor to consider is still the element of control over how the work itself is done,
not just its end result.

● Petitioners cannot use the revolving funds feature of the SFAs as evidence of their employer-employee
relationship with Bandag. These funds do not represent wages. They are more in the nature of capital

Page 30 of 67
LABOR 1 | JMM | CASE DIGESTS 2020 – TOPICS 1-3

advances for operations that Bandag conceptualized to attract prospective franchisees. Petitioners'
incomes depended on the profits they make, controlled by their individual abilities to increase sales and
reduce operating costs.

● The Labor Arbiter, the NLRC, and the CA, are unanimous that petitioners were no longer "route salesmen,
bringing previously ordered supplies and goods to dealers, taking back returned items, collecting payments,
remitting them, etc. They were themselves then the dealers, getting their own supply and bringing these to
their own customers and sub-dealers, if any."

● The rule in labor cases is that the findings of fact of quasi-judicial bodies, like the NLRC, are to be accorded
with respect, even finality, if supported by substantial evidence. This is particularly true when passed upon
and upheld by the CA.

NOTES:
Tongko v. The Manufacturers Life Insurance Co. (Phils.), Inc., results-wise, the insurance company, as principal,
can impose production quotas upon its independent agents and determine how many individual agents, with specific
territories, such independent agents ought to employ to achieve the company's objectives. These are management
policy decisions that the labor law element of control cannot reach. Petitioners' commitment to abide by Bandag's
policy decisions and implementing rules, as franchisees does not make them its employees.

18. Royale Homes Marketing Corp., vs. Alcantara, GR No. 195190, July 28, 2014

PETITIONER: Royale Homes Marketing Corp RESPONDENT: Alcantara


● Respondent is not an employee. The ● He is a regular employee since he is performing
appointment paper of respondent is clear that tasks that are necessary and desirable to its
it engaged his services as an independent sales business
contractor for a fixed term of 1 year only. ● The acts of the executive officers amounted to
● Respondent never received any salary, 13th his dismissal from work without any valid or just
month pay, overtime pay or holiday pay as he cause and in gross disregard of the proper
was paid purely on commission basis. procedure for dismissing employees.
● Petitioner had no control on how respondent ● He also impleaded the corporate officers who
would accomplish his tasks and effected his dismissal in bad faith and in an
responsibilities as he was free to solicit sales at oppressive manner.
any time and by any manner which he may ● Prayed to be reinstated to his former position
deem appropriate and necessary. without loss of seniority rights and other
privileges, as well as to be paid backwages,
moral and exemplary damages, atty’s fees, and
ownership of a Mitsubishi Adventure be
transferred to his name.

SPECIAL DETAILS:
● In 2003, respondent was given P1.2M for his services. In the first week of November of the same year, the
executive officers of Royal Homes told him that they were wondering why he still had the gall to come to
office and sit at his table.
● According to petitioner, Alcantara decided to leave the company after his wife, who was once connected
with it as a sales agent, had formed a brokerage company that directly competed with its business, and
even recruited some of its sales agents.

LAW & PRINCIPLES:


Four⎼fold Test ⎼ 1. Selection and engagement of employees, 2. Payment of wages, 3. Power of dismissal, 4. Power
to control employees’ conduct and with respect to the means and methods by which the work is to be accomplished.

FACTS:
● In 1994, Royale Homes, a corporation engaged in marketing real estates, appointed respondent as its
Marketing Director for a fixed period of 1 year. Petitioner reappointed him for several consecutive years,
the last of which covered the period of January 1 to December 31, 2003 where he held the position of
Division 5 VP–Sales.

Page 31 of 67
LABOR 1 | JMM | CASE DIGESTS 2020 – TOPICS 1-3

● On Dec. 17 of the same year, respondent filed a complaint for illegal dismissal against petitioner, its
President, Executive VP for Administration and Finance, and Executive VP for Sales, alleging that he is a
regular employee of the corporation since he is performing tasks that are necessary and desirable to its
business.

Ruling of LA
● Respondent is an employee of Royale Homes with a fixed term employment period from January 1 to Dec.
31, 2003 and that the pre–termination of his contract was against the law. Hence, Alcantara is entitled to
an amount which he may earn on the average for the unexpired portion of the contract. With regard to the
impleaded corporate officers, they are absolved from any liability.

Ruling of NLRC
● Respondent is not an employee but a mere independent contractor of Royale Homes. It’s ruling is mainly
based on the contract which does not require respondent to observe regular working hours. He was also
free to adopt the selling methods he deemed most effective and can even recruit sales agents to assist
him in marketing the inventories of Royale Homes. The fact that respondent was not receiving monthly
salary, but was being paid on commission basis as stipulated in the contract.

Ruling of CA
● Granted respondent’s petition and reversed NLRC’s decision. Applying the fourfold and economic reality
tests, it held that respondent is an employee of Royale Homes. Petitioner exercised some degree of control
over respondent since his job is subject to company rules, regulations, and periodic evaluations. He was
also bound by the company code of ethics. Moreover, the exclusivity clause of the contract has made
respondent economically dependent on Royale Homes , supporting the theory that he is an employee of
said company.

● Respondent’s termination was without any valid or just cause, and it was carried out in violation of his right
to procedural due process. Thus, he is entitled to backwages and separation pay, in lieu of reinstatement.
But since CA was not satisfied with the proof adduced to establish the amount of respondent’s annual
salary, it remanded the case to the LA to determine the same and the monetary award he is entitled to. With
regard to corporate officers, they are absolved from any liability for want of clear proof that they assented
to the patently unlawful acts or that they are guilty of bad faith or gross negligence.

ISSUES: W/N the CA has committed an error in its judgment

RULING:
● The petition is impressed with merit. The primary evidence of the nature of the parties’ relationship in this
case is the written contract that they signed and executed in pursuance of their mutual agreement. While
the existence of E⎼E relationship is a matter of law, the characterization made by the parties in their
contract as to the nature of their juridical relationship cannot be simply ignored, particularly in this case
where the parties’ written contract unequivocally states their intention at the time they entered into it. The
contract, duly signed and not disputed by the parties conspicuously provides that “no E⎼E relationship
exists between” Royale Homes and Alcantara, as well, as his sales agents. It is clear that they did not want
to be bound by E⎼E relationship at the time of the singing of the contract.

● In determining the existence of an E⎼E relationship, the court relied on the four⎼fold rest. Among the four,
the most determinative factor in ascertaining the existence of E⎼E relationship is the “right to control test”.
“It is deemed to be such an important factor that the other requisites may even be disregarded.” This holds
true where the issues to be resolved is whether a person who performs work for another is the latter’s
employee or is an independent contractor, as in this case. For where the person for whom the services are
performed reserves the right to control not only the end to be achieved, but also the means by which such
end is reach, E⎼E relationship is deemed to exist.

● The rules, regulations, code of ethics, and periodic evaluation alluded to by Alcantara do not involve control
over the means and methods by which he was to perform his job. Notably, Alcantara was not required to
observe definite working hours. He had full control over the means and methods of accomplishing his tasks
as he can “solicit sales at any time and by any manner which [he may] deem appropriate and necessary.”

Page 32 of 67
LABOR 1 | JMM | CASE DIGESTS 2020 – TOPICS 1-3

● The continuous rehiring of Alcantara simply signifies the renewal of his contract with petitioner, and
highlights his satisfactory services warranting the renewal of such contract. Nor does the exclusivity clause
of contract establish the existence of the labor law concept of control.

● There is no proof that he received fixed monthly salary. No payslip or payroll was ever presented and there
is no proof that petitioner deducted from his supposed salary withholding tax or that it registered him with
the SSS, PhilHealth, or Pag⎼ibig Fund. All these indicated an independent contractual relationship. This
Court is, therefore, convinced that Alcantara is not an employee of Royale Homes, but a mere independent
contractor. The NLRC is correct in concluding that the LA has no jurisdiction over the case and that the
same is cognizable by the regular courts.

NOTES:
Insular Life Assurance Co., Ltd v. NLRC ⎼ the line should be drawn between rules that merely serve as guidelines
towards the achievement of the mutually desired result without dictating the means/methods to be employed in
attaining it, and those that control/fix the methodology and bind or restrict the party hired to the use of such means.
The first, which aim only to promote the result, create no E⎼E relationship unlike the second, which addresses both
the result and the means used to achieve it.

Tongko ⎼ Guidelines indicative of labor law “control,” as the first Insular Life case tells us, should not merely relate
to the mutually desirable result intended by the contractual relationship; they must have the nature of dictating the
means/methods to be employed in attaining the result, or fixing the methodology and of binding or restricting the
party hired to the use of these means. Result⎼wise, the principal can impose production quotas and can determine
how many agents, with specific territories, ought it be employed to be achieve the company’s objectives. These are
management policy decisions that the labor law element of control cannot reach. All of which do not intrude into the
insurance agents’ means and manner of conducting their sales and only control them as to the desired results and
Insurance Code norms, cannot be used as basis for a finding that the labor law concept of control existed between
Manulife and Tongko.

19. Fuji Television Network Inc. vs. Espiritu, GR No. 204944-45, December 3, 2014

PETITIONER: Fuji Television Network Inc. (Lose) RESPONDENT: Arlene S. Espiritu (Win)

SPECIAL DETAILS: When Arlene signed the non-renewal contract, she affixed U.P to her signature to indicate that the
same was under protest. She filed the complaint with NLRC the day after.

LAW & PRINCIPLES: Regular Employee vs. Contractual (Art.280) ; Illegal Dismissal

FACTS:

● In 2005, Arlene S. Espiritu ("Arlene") was engaged by Fuji Television Network, Inc. ("Fuji") as a news
correspondent/producer. Her employment contract initially provided for a term of one (1) year but was
successively renewed on a yearly basis with salary adjustment upon every renewal.
● In 2009, she was diagnosed with lung cancer. She informed Fuji of her condition. Arlene was informed that
her contract will no longer be renewed since it will be difficult for her to perform her job. She insisted that
she is still fit to work as certified by her doctor.
● The day after Arlene signed the non-renewal contract, she filed a complaint for illegal dismissal and
attorney's fees with the NLRC. She alleged that she was forced to sign the non-renewal contract when Fuji
came to know of her illness and that Fuji withheld her salaries and other benefits.
● Arlene claimed that she was left with no other recourse but to sign the non-renewal contract, and it was
only upon signing that she was given her salaries and bonuses, in addition to separation pay equivalent to
four (4) years.

Page 33 of 67
LABOR 1 | JMM | CASE DIGESTS 2020 – TOPICS 1-3

ISSUES:

1) Whether or not Espiritu is a regular employee or a fixed-term contractual employee; and

2) Whether or not Espiritu was illegally dismissed;

|||

RULING:

1. Arlene was not an independent Contractor. She was a regular employee with a fixed-term contract.

Fuji alleged that Arlene was an independent contractor citing the Sonza case. She was hired because of her skills.
Her salary was higher than the normal rate. She had the power to bargain with her employer. Her contract was for a
fixed term. It also stated that Arlene was not forced to sign the non-renewal agreement, considering that she sent
an email with another version of her non-renewal agreement.

Arlene argued (1) that she was a regular employee because Fuji had control and supervision over her work; (2) that
she based her work on instructions from Fuji; (3) that the successive renewal of her contracts for four years indicated
that her work was necessary and desirable; (4) that the payment of separation pay indicated that she was a regular
employee; (5) that the Sonza case is not applicable because she was a plain reporter for Fuji; (6) that her illness was
not a ground for her dismissal; (7) that she signed the non-renewal agreement because she was not in a position to
reject the same.

2. Arlene was illegally dismissed.

As a regular employee, Arlene was entitled to security of tenure under Article 279 of the Labor Code and could be
dismissed only for just or authorized causes and after observance of due process.

The expiration of the contract does not negate the finding of illegal dismissal. The manner by which Fuji informed
Arlene of non-renewal through email a month after she informed Fuji of her illness is tantamount to constructive
dismissal. Further, Arlene was asked to sign a letter of resignation prepared by Fuji. The existence of a fixed-term
contract should not mean that there can be no illegal dismissal. Due process must still be observed.

Moreoever, disease as a ground for termination under Article 284 of the Labor Code and Book VI, Rule 1,
Section 8 of the Omnibus Rules Implementing the Labor Code require two requirements to be complied
with: (1) the employee’s disease cannot be cured within six months and his continued employment is
prohibited by law or prejudicial to his health as well as to the health of his co-employees; and (2)
certification issued by a competent public health authority that even with proper medical treatment, the
disease cannot be cured within six months. The burden of proving compliance with these requisites is on
the employer. Non-compliance leads to illegal dismissal.

NOTES:

BURDEN OF PROVING INDEPENDENT CONTRACT


It is the burden of the employer to prove that a person whose services it pays for is an independent contractor rather
than a regular employee with or without a fixed term. That a person has a disease does not per se entitle the
employer to terminate his or her services. Termination is the last resort. At the very least, a competent public health
authority must certify that the disease cannot be cured within six (6) months, even with appropriate treatment.

DISTINCTIONS AMONG FIXED-TERM EMPLOYEES, INDEPENDENT CONTRACTORS, AND REGULAR EMPLOYEES


I.Fixed Term Employment
1) The fixed period of employment was knowingly and voluntarily agreed upon by the parties without any force,
duress, or improper pressure being brought to bear upon the employee and absent any other circumstances vitiating
his consent; or

Page 34 of 67
LABOR 1 | JMM | CASE DIGESTS 2020 – TOPICS 1-3

2) It satisfactorily appears that the employer and the employee dealt with each other on more or less equal terms
with no moral dominance exercised by the former or the latter.
These indications, which must be read together, make the Brent doctrine applicable only in a few special cases
wherein the employer and employee are on more or less in equal footing in entering into the contract. The reason
for this is evident: when a prospective employee, on account of special skills or market forces, is in a position to
make demands upon the prospective employer, such prospective employee needs less protection than the ordinary
worker. Lesser limitations on the parties’ freedom of contract are thus required for the protection of the
employee.155 (Citations omitted)

For as long as the guidelines laid down in Brent are satisfied, this court will recognize the validity of the fixed-term
contract. (GMA Network, Inc. vs. Pabriga)

II.Independent Contractor
One who carries on a distinct and independent business and undertakes to perform the job, work, or service on its
own account and under one’s own responsibility according to one’s own manner and method, free from the control
and direction of the principal in all matters connected with the performance of the work except as to the results
thereof.

No employer-employee relationship exists between the independent contractors and their principals.

Art. 106. Contractor or subcontractor. Whenever an employer enters into a contract with another person for the
performance of the former’s work, the employees of the contractor and of the latter’s subcontractor, if any, shall be
paid in accordance with the provisions of this Code.

XXX
The Secretary of Labor and Employment may, by appropriate regulations, restrict or prohibit the contracting-out of
labor to protect the rights of workers established under this Code. In so prohibiting or restricting, he may make
appropriate distinctions between labor-only contracting and job contracting as well as differentiations within these
types of contracting and determine who among the parties involved shall be considered the employer for purposes
of this Code, to prevent any violation or circumvention of any provision of this Code.

There is “labor-only” contracting where the person supplying workers to an employer does not have substantial
capital or investment in the form of tools, equipment, machineries, work premises, among others, and the workers
recruited and placed by such person are performing activities which are directly related to the principal business of
such employer. In such cases, the person or intermediary shall be considered merely as an agent of the employer
who shall be responsible to the workers in the same manner and extent as if the latter were directly employed by
him.

Department Order No. 18-A, Series of 2011, Section 3

© . . . an arrangement whereby a principal agrees to put out or farm out with a contractor the
performance or completion of a specific job, work or service within a definite or predetermined period,
regardless of whether such job, work or service is to be performed or completed within or outside the
premises of the principal.

This department order also states that there is a trilateral relationship in legitimate job contracting and
subcontracting arrangements among the principal, contractor, and employees of the contractor. There
is no employer-employee relationship between the contractor and principal who engages the
contractor’s services, but there is an employer-employee relationship between the contractor and
workers hired to accomplish the work for the principal.162chanRoblesvirtualLawlibrary

Jurisprudence has recognized another kind of independent contractor: individuals with unique skills
and talents that set them apart from ordinary employees. There is no trilateral relationship in this case
because the independent contractor himself or herself performs the work for the principal. In other
words, the relationship is bilateral.

XXX

Page 35 of 67
LABOR 1 | JMM | CASE DIGESTS 2020 – TOPICS 1-3

There are different kinds of independent contractors: those engaged in legitimate job contracting and those who
have unique skills and talents that set them apart from ordinary employees.

Since no employer-employee relationship exists between independent contractors and their principals, their
contracts are governed by the Civil Code provisions on contracts and other applicable laws.

III. Regular Employees


Contracts of employment are different and have a higher level of regulation because they are impressed with public
interest. Article 13, Section 3 of the 1987 Constitution provides full protection to labor.

Apart from the Constitutional guarantee, Article 1700 of the Civil Code states that: The relations between capital and
labor are not merely contractual. They are so impressed with public interest that labor contracts must yield to the
common good. Therefore, such contracts are subject to the special laws on labor unions, collective bargaining,
strikes and lockouts, closed shop, wages, working conditions, hours of labor and similar subjects.

The level of protection to labor should vary from case to caese. When a prospective employee, on account of special
skills or market forces, is in a position to make demands upon the prospective employer, such prospective employee
needs less protection than the ordinary worker.

The level of protection to labor must be determined on the basis of the nature of the work, qualifications of the
employee, and other relevant circumstances such as but not limited to educational attainment and other special
qualifications.

Fuji’s argument that Arlene was an independent contractor under a fixed-term contract is contradictory. Employees
under fixed-term contracts cannot be independent contractors because in fixed-term contracts, an employer-
employee relationship exists. The test in this kind of contract is not the necessity and desirability of the employee’s
activities, “but the day certain agreed upon by the parties for the commencement and termination of the employment
relationship.” For regular employees, the necessity and desirability of their work in the usual course of the employer’s
business are the determining factors. On the other hand, independent contractors do not have employer-employee
relationships with their principals.

To determine the status of employment, the existence of employer-employee relationship must first be settled with
the use of the four-fold test, especially the qualifications for the power to control.

20. Cabaobas et al., vs. Pepsi Cola GR No.176908, March 25, 2015

PETITIONER: Cabaobas et al. (Lose) RESPONDENT:Pepsi Cola (Win)

SPECIAL DETAILS: Of the 47 retrenched employees, 27 of said employees filed the complaint led by Anecito Molon.

LAW & PRINCIPLES: REQUISITES FOR A VALID RETRENCHMENT

FACTS:

Respondent Pepsi-Cola Products Philippines, Inc. (PCPPI) is a domestic corporation engaged in the manufacturing,
bottling and distribution of soft drink products, which operates plants all over the country, one of which is the
Tanauan Plant in Tanauan, Leyte.

Page 36 of 67
LABOR 1 | JMM | CASE DIGESTS 2020 – TOPICS 1-3

1999 - PCPPI’s Tanauan Plant allegedly incurred business losses to over P29, 167, 390. To avert further losses,
PCPPI implemented a company-wide retrenchment program (Corporate-wide Rightsizing Program or CRP) from
1999 to 2000, and retrenched forty-seven (47) employees of its Tanauan Plant.

2000 - petitioners, who are permanent and regular employees of the Tanauan Plant, received their respective letters,
informing them of the cessation of their employment. They filed their respective complaints for illegal dismissal
before the National Labor Relations Commission Regional Branch in Tacloban.

Petitioners alleged that PCPPI was not facing serious financial losses because after their termination, it regularized
four (4) employees and hired replacements for the forty-seven (47) previously dismissed employees. They also
alleged that PCPPI's CRP was just designed to prevent their union, Leyte Pepsi-Cola Employees Union-Associated
Labor Union (LEPCEU-ALU), from becoming the certified bargaining agent of PCPPI's rank-and-file employees.

PCPPI countered that petitioners were dismissed pursuant to its CRP to save the company from total bankruptcy
and collapse; thus, it sent notices of termination to them and to the Department of Labor and Employment.

The NLRC dismissed the complaints for illegal dismissal, and declared the retrenchment program is a valid exercise
of management prerogatives. Petitioners filed a petition for certiorari with the CA. The appellate court affirmed the
decision of the NLRC. Aggrieved, petitioners come before the Court in this petition for review on certiorari.

ISSUE:
Whether or not petitioners’ dismissal pursuant to respondent’s retrenchment program is legal?

RULING:

Yes, as all the requisites for a valid retrenchment are present, the Court finds Pepsi’s rightsizing program and the
consequent dismissal of petitioners in accord with law.

Essentially, the prerogative of an employer to retrench its employees must be exercised only as a last resort,
considering that it will lead to the loss of the employees' livelihood. It is justified only when all other less drastic
means have been tried and found insufficient or inadequate. Corollary thereto, the employer must prove the
requirements for a valid retrenchment by clear and convincing evidence; otherwise, said ground for termination
would be susceptible to abuse by scheming employers who might be merely feigning losses or reverses in their
business ventures in order to ease out employees.

These requirements are:

(1) That retrenchment is reasonably necessary and likely to prevent business losses which, if already incurred,
are not merely de minimis, but substantial, serious, actual and real, or if only expected, are reasonably imminent
as perceived objectively and in good faith by the employer;

(2) That the employer served written notice both to the employees and to the Department of Labor and
Employment at least one month prior to the intended date of retrenchment;

(3) That the employer pays the retrenched employees separation pay equivalent to one (1) month pay or at least
one-half (½) month pay for every year of service, whichever is higher;

(4) That the employer exercises its prerogative to retrench employees in good faith for the advancement of its
interest and not to defeat or circumvent the employees’ right to security of tenure; and

(5) That the employer used fair and reasonable criteria in ascertaining who would be dismissed and who would
be retained among the employees, such as status, efficiency, seniority, physical fitness, age, and financial
hardship for certain workers.

Page 37 of 67
LABOR 1 | JMM | CASE DIGESTS 2020 – TOPICS 1-3

Moreover, in due regard of the abovementioned requisites, the Court observes that Pepsi had validly implemented
its retrenchment program:

● Pepsi complied with the requirements of substantial loss (PEPSI-COLA’s financial statements are
substantial evidence which carry great credibility and reliability viewed in light of the financial crisis that hit
the country which saw multinational corporations closing shops and walking away, or adapting [sic] their
own corporate rightsizing program).
● Pepsi complied with the requirements of due notice to both the DOLE and the workers to be retrenched at
least one (1) month prior to the date of retrenchment.
● Respondents had already been paid the requisite separation pay as evidenced by the September 1999
quitclaims signed by them.
● Pepsi’s Corporate Rightsizing Program was a company-wide program which had already been implemented
in its other plants in Bacolod, Iloilo, Davao, General Santos and Zamboanga. Consequently, given the
general applicability of its retrenchment program, Pepsi could not have intended to decimate LEPCEU-
ALU’s membership, much less impinge upon its right to self-organization, when it employed the same.
● Pepsi’s management exerted conscious efforts to incorporate employee participation during the
implementation of its retrenchment program. Records indicate that Pepsi had initiated sit-downs with its
employees to review the criteria on which the selection of who to be retrenched would be based.
● On the final requirement of fair and reasonable criteria for determining who would or would not be
dismissed, records indicate that Pepsi did proceed to implement its rightsizing program based on fair and
reasonable criteria recommended by the company supervisors.
● Verily, the foregoing incidents clearly negate the claim that the retrenchment was undertaken by Pepsi in
bad faith.

Final Disposition: The petition is denied. The decision and resolution of the Court of Appeals are affirmed.

NOTES: There were other secondary issues discussed in the case. It will be summarized below:

On PCPPI's alleged failure to explain its acts of regularizing four (4) employees and hiring sixty-three (63)
replacements and additional workers

The Court upheld the NLRC Ruling; the replacements were not regular employees and held a different function from
those retrenched. The idea of rightsizing is to reduce the number of workers and related functions and trim down,
streamline, or simplify the structure of the organization to the level of utmost efficiency and productivity in order to
realize profit and survive. After the CRP shall have been implemented, the desired size of the corporation is attained.
Engaging the services of service contractors does not expand the size of the corporate structure. In this sense, the
retrenched workers were not replaced.

On the issue of Union busting

Pepsi Cola, in the selection of workers to be retrenched, did not take into consideration union affiliation because the
unit was supposed to be composed of all members of good standing. epsi Cola, in the selection of workers to be
retrenched, did not take into consideration union affiliation because the It bears stressing that all 47 workers signed
individual release and quitclaims and settled their complaints with respondent Pepsi Cola, apparently with the
assistance of LEPCEU-ALU (a union). It is awkward for LEPCEU-ALU to argue that a serious corporate-wide
rightsizing program cannot be implemented in PEPSI-COLA Tanauan Plant because a nascent unrecognized union
would probably be busted.

21. Begino et al., vs. ABS-CBN Corp., GR No. 199166, April 20, 2015

PETITIONER: Begino et al (Win) RESPONDENT: ABS-CBN Corp (Lose)

SPECIAL DETAILS:

Page 38 of 67
LABOR 1 | JMM | CASE DIGESTS 2020 – TOPICS 1-3

LAW & PRINCIPLES: Article 280 (LC) Employer-Employee Relationship (Four-fold test)

FACTS:
● Respondent ABS-CBN Corporation (formerly ABS-CBN Broadcasting Corporation) is a television and radio
broadcasting corporation which, for its Regional Network Group in Naga City, employed respondent Amalia
Villafuerte as Manager. Thru Villafuerte, ABS-CBN engaged the services of petitioners Begino and Del Valle
sometime in 1996 as Cameramen/Editors for TV Broadcasting while Sumayao and Llorin were engaged as
reporters sometime in 1996 and 2002, respectively.

● Talent Contracts were executed which, though regularly renewed over the years, provided terms ranging
from three (3) months to one (1) year, petitioners were given Project Assignment Forms which detailed,
among other matters, the duration of a particular project as well as the budget and the daily technical
requirements of coverage of news items for subsequent daily airings in respondents’ TV Patrol Bicol
Program.

● While specifically providing that nothing therein shall be deemed or construed to establish an employer-
employee relationship between the parties, the Talent Contracts included provisions on creation and
performance of work in accordance with the ABS-CBN’s professional standards and compliance with its
policies and guidelines covering intellectual property creators, industry codes as well as the rules and
regulations of the KBP and other regulatory agencies, non-engagement in similar work for a person or
entity directly or indirectly in competition with or adverse to the interests of ABS-CBN and non-promotion
of any product or service without prior written consent, and results-oriented nature of the talent’s work
which did not require them to observe normal or fixed working hours.

● Claiming that they were regular employees of ABS-CBN, petitioners filed against respondents the complaint
before the NLRC for regularization, underpayment of overtime pay, holiday pay, 13th month pay, service
incentive leave pay, damages and attorney's fees. Petitioners alleged that they performed functions
necessary and desirable in ABS-CBN's business but were not paid the labor standard benefits the law
extends to regular employees, instead respondents purportedly resorted to the simple expedient of using
said Talent Contracts and/or Project Assignment Forms which denominated petitioners as talents, despite
the fact that they are not actors or TV hosts of special skills.

● Respondent argued that, not having the full manpower complement to produce its own program, the
company had allegedly resorted to engaging independent contractors like actors, directors, artists,
anchormen, reporters, scriptwriters and various production and technical staff, who offered their services
in relation to a particular program. Moreover, company cannot afford to provide regular work for talents
with whom it negotiates specific or determinable professional fees on a per project, weekly or daily basis,
usually depending on the budget allocation for a project.

● NLRC affirmed the earlier ruling in favor of petitioners who, having rendered services necessary and related
to ABS-CBN’s business for more than a year as its regular employees.

● Upon appeal, the CA held that there was no employer-employee relationship. Hence, this petition.

ISSUES: Whether there was an employer-employee relationship between the parties.

RULING:

YES. Court reversed and set aside CA, reinstated NLRC ruling except re-employment. Except for the reinstatement
of Nelson V. Begino, Gener Del Valle, Monina Avila-Llorin and Ma. Cristina Sumayao, the NLRC is accordingly
reinstated.

The Court finds that, notwithstanding the nomenclature of their Talent Contracts and/or Project Assignment Forms
and the terms and condition embodied therein, petitioners are regular employees of ABS-CBN. Time and again, it
has been ruled that the test to determine whether employment is regular or not is the reasonable connection between
the activity performed by the employee in relation to the business or trade of the employer.

Page 39 of 67
LABOR 1 | JMM | CASE DIGESTS 2020 – TOPICS 1-3

As cameramen/editors and reporters, petitioners were undoubtedly performing functions necessary and essential
to ABS-CBN's business of broadcasting television and radio content. It matters little that petitioners' services were
engaged for specified periods for TV Patrol Bicol and that they were paid according to the budget allocated therefor.
Aside from the fact that said program is a regular weekday fare of the ABS-CBN's Regional Network Group in Naga
City, the record shows that, from their initial engagement in the aforesaid capacities, petitioners were continuously
re-hired by respondents over the years. To the mind of the Court, respondents' repeated hiring of petitioners for its
long-running news program positively indicates that the latter were ABS-CBN's regular employees. It also appears
that petitioners were subject to the control and supervision of respondents. The presumption is that when the work
done is an integral part of the regular business of the employer and when the worker, relative to the employer,
does not furnish an independent business or professional service, such work is a regular employment of such
employee and not an independent contractor. |||

NOTES:

Because they are imbued with public interest, labor contracts are subject to the police power of the state and are
placed on a higher plane than ordinary contracts. The recognized supremacy of the law over the nomenclature of
the contract and the stipulations contained therein is aimed at bringing life to the policy enshrined in the Constitution
to afford protection to labor.

If the employee has been performing the job for at least one year, even if the performance is not continuous or
merely intermittent, the law deems the repeated or continuing performance as sufficient evidence of the necessity,
if not indispensability of that activity in the business. When circumstances show that contractually stipulated
periods of employment have been imposed to preclude the acquisition of tenurial security by the employee, Court
has not hesitated in striking down such arrangements as contrary to public policy, morals, good customs or public
order. The nature of the employment depends on the nature of the activities to be performed by the employee,
considering the nature of the employer’s business, the duration and scope to be done, and, in some cases, even
the length of time of the performance and its continued existence.

Determining existence of Employer-Employee Relationship


Case law has consistently applied the four-fold test, to wit: (a) the selection and engagement of the employee; (b)
the payment of wages; (c) the power of dismissal; and (d) the employer's power to control the employee on the
means and methods by which the work is accomplished.

Of these criteria, the so-called "control test" is generally regarded as the most crucial and determinative indicator of
the presence or absence of an employer-employee relationship. Under this test, an employer-employee relationship
is said to exist where the person for whom the services are performed reserves the right to control not only the end
result but also the manner and means utilized to achieve the same.

*See Article 280 of the Labor Code**

22. Social Security System vs. Ubana, GR No. 200114, Aug 25, 2015

PETITIONER: SOCIAL SECURITY SYSTEM (Lost) RESPONDENT: DEBBIE UBAÑA (Won)

Petitioner and its co-defendants SSS Retirees Defendants (SSS) conspired to exploit her and violate
Association and DBP Service Corporation filed their civil service laws and regulations and Civil Code
respective motions to dismiss, arguing that the subject provisions on Human Relations, particularly Articles 19,
matter of the case and respondent's claims arose out of 20, and 21. 8 As a result, she suffered actual losses by
employer-employee relations, which are beyond the way of unrealized income, moral and exemplary
RTC's jurisdiction and properly cognizable by the damages, attorney's fees and litigation expenses.
National Labor Relations Commission (NLRC).
Respondent claimed she was qualified for her position
as Processor, having completed required training and
In a Petition for certiorari, petitioners insist that the trial passed the SSS qualifying examination for Computer
court did not have jurisdiction over respondent's claims Operations Course given by the National Computer

Page 40 of 67
LABOR 1 | JMM | CASE DIGESTS 2020 – TOPICS 1-3

for "unrealized salary income" and other damages, which Institute, U.P. Diliman from May 16 to June 10, 2001, yet
constitute a labor dispute cognizable only by the labor she was not given the proper salary.
tribunals.
Respondent opposed the motions to dismiss, arguing
that pursuant to civil service rules and regulations,
service contracts such as her Service Contract
It maintained that DBP Service Corporation and SSS Agreement with DBP Service Corporation should cover
Retirees Association are legitimate independent job only a) lump sum work or services such as janitorial,
contractors engaged by it to provide manpower services security or consultancy services, and b) piece work or
since 2001, which thus makes respondent an employee intermittent jobs of short duration not exceeding six
of these two entities and not of SSS; and that since it is months on a daily basis. She posited that her service
not the respondent's employer, then there is no cause of contract involved the performance of sensitive work, and
action against it. not merely janitorial, security, consultancy services, or
work of intermittent or short duration. In fact, she was
Petitioners argued that there is a direct causal made to work continuously even after the lapse of her 6-
connection between respondent's claims and her month service contract. Citing Civil Service Commission
employment, which brings the subject matter within the Memorandum Circular No. 40, respondent contended
jurisdiction of the NLRC. Petitioner contends that that the performance of functions outside of the nature
respondent's other claims are intimately intertwined with provided in the appointment and receiving salary way
her claim of actual damages which are cognizable by the below that received by regular SSS employees amount
NLRC. Finally, petitioner avers that the nature and to an abuse of rights; and that her cause of action is
character of the reliefs prayed for by the respondent are anchored on the provisions of the Civil Code on Human
directly within the jurisdiction not of the courts, but of the Relations.
labor tribunals.

SPECIAL DETAILS:

Respondent worked as a Processor with petitioner for six long years, she was paid only P5,038.00 monthly, or
P229.00 daily, while a regular SSS employee with the same designation and who performs identical functions is paid
a monthly salary of P18,622.00, or P846.45 daily wage.

LAW & PRINCIPLES:

Article 217 of the Labor Code as amended vests upon the labor arbiters exclusive original jurisdiction only over the
following:
1. Unfair labor practices;
2. Termination disputes;
3. If accompanied with a claim for reinstatement, those cases that workers may file involving wages, rates
of pay, hours of work and other terms and conditions of employment;
4. Claims for actual, moral, exemplary and other forms of damages arising from employer-employee
relations;
5. Cases arising from any violation of Article 264 of this Code, including questions involving legality of
strikes and lockouts; and
6. Except claims for Employees Compensation, Social Security, Medicare and maternity benefits, all other
claims, arising from employer-employee relations, including those of persons in domestic or household
service, involving an amount exceeding five thousand pesos (P5,000.00) regardless of whether
accompanied with a claim for reinstatement.

Article 19 and 20 of the Labor Code."Every person, must in the exercise of his rights and in the performance of his
duties, act with justice, give everyone his due and observe honesty and good faith (Article 19) and that "Every person
who, contrary to law, willfully or negligently [causes] damages to another, shall indemnify the latter for the same.
(Art. 20)

FACTS:
● December 26, 2002 - Debbie Ubaña filed a civil case for damages against the DBP Service Corporation,
petitioner Social Security System (SSS), and the SSS Retirees Association

Page 41 of 67
LABOR 1 | JMM | CASE DIGESTS 2020 – TOPICS 1-3

● Respondent alleged that in July 1995, she applied for employment with the petitioner. However, after
passing the examinations and accomplishing all the requirements for employment, she was instead
referred to DBP Service Corporation for "transitory employment."
● She took the pre-employment examination given by DBP Service Corporation and passed the same and
went for training to SSS NAga City branch.
● May 28, 1996 - she was made to sign a six-month Service Contract Agreement by DBP Service Corporation,
appointing her as clerk for assignment with SSS Daet branch and was assigned as "Frontliner" of the SSS
Members Assistance Section.
● December 16, 1999 to May 15, 2001 - assigned to the Membership Section as Data Encoder.
● December 16, 2001 to August 26, 2002 (resignation) - transferred to the SSS Retirees Association as
Processor at the Membership Section.
● Her May 28, 1996 Service Contract Agreement with DBP Service Corporation was never renewed, but she
was required to work for SSS continuously under different assignments with a maximum daily salary of
only P229.00; at the same time, she was constantly assured of being absorbed into the SSS plantilla.
● Respondent claimed that she was a qualified Processor but was not given a proper salary.
● Because of the oppressive and prejudicial treatment by SSS, she was forced to resign on August 26, 2002
as she could no longer stand being exploited, the agony of dissatisfaction, anxiety, demoralization, and
injustice. She asserted that she dedicated six years of her precious time faithfully serving SSS, foregoing
more satisfying employment elsewhere, yet she was merely exploited and given empty and false promises.
● Respondent prayed for an award for damages.
● Petitioner argued that the subject matter of the case and respondent's claims arose out of employer-
employee relations, which are beyond the RTC's jurisdiction and properly cognizable by the National Labor
Relations Commission (NLRC).
● Respondent opposed the petitioner's Motion to dismiss arguing that the work she rendered involved the
performance of sensitive work which is beyond the coverage of her Service Contract Agreement with DBP
Service Corporation.
● Citing Civil Service Commission Memorandum Circular No. 40, respondent contended that the
performance of functions outside of the nature provided in the appointment and receiving salary way
below that received by regular SSS employees amount to an abuse of rights; and that her cause of action
is anchored on the provisions of the Civil Code on Human Relations.

RTC RULING

● October 1, 2003- Issued an order dismissing respondent's complaint for lack of jurisdiction, stating that her
claim for damages "has a reasonable causal connection with her employer-employee relations with the
defendants" and "is grounded on the alleged fraudulent and malevolent manner by which the defendants
conspired with each other in exploiting [her], which is a clear case of unfair labor practice," falling under the
jurisdiction of the Labor Arbiter of the NLRC.
● March 6, 2007 - issued another order granting respondent's motion for reconsideration on the ground that
SSS denied the existence of an employer-employee relationship, and the case is one for Damages, it is not
the Civil Service Commission that has jurisdiction to try the case, but the regular courts.

CA RULING

● July 29, 2011- the petition is denied and the RTC’s March 6, 2007’s decision is affirmed. In determining
which body has jurisdiction over a case, the better policy is to consider not only the status or relationship
of the parties but also the nature of the action that is the subject of their controversy.
● Ubaña's claim is rooted on the principle of abuse of right laid in the New Civil Code. She was claiming
damages based on the alleged exploitation [perpetrated] by the defendants depriving her of her rightful
income.
● She invoked not the provisions of the Labor Code or any other labor laws but the provisions on human
relations under the New Civil Code.
● It is the character of the principal relief sought that appears essential in this connection. Where such
principal relief is to be granted under labor legislation or a collective bargaining agreement, the case should
fall within the jurisdiction of the Labor Arbiter and the NLRC, even though a claim for damages might be
asserted as an incident to such claim.

Page 42 of 67
LABOR 1 | JMM | CASE DIGESTS 2020 – TOPICS 1-3

● In the present case, Ubaña's claim for damages is not related to any other claim under Article 217, other
labor statutes, or collective bargaining agreements. All told, it is ineluctable that it is the regular courts that
has [sic] jurisdiction to hear and decide Civil Case No. 7304.

Petitioner filed a Motion for Reconsideration but was denied by the CA. Hence, this petition.

ISSUES:

W/N the trial court has jurisdiction over respondent's claims for "unrealized salary income" and other damages.

RULING:

YES. Since there is no employer-employee relationship between the parties herein, then there
is no labor dispute cognizable by the Labor Arbiters or the NLRC.

"[i]n legitimate job contracting, no employer-employee relation exists between the principal and the job contractor's
employees. The principal is responsible to the job contractor's employees only for the proper payment of wages."

Respondent acknowledges that she is not petitioner's employee, but that precisely she was promised that she would
be absorbed into the SSS plantilla after all her years of service with SSS. In its pleadings, petitioner denied the
existence of an employer-employee relationship between it and respondent. Since both parties admit that there is
no employment relation between them, then there is no dispute cognizable by the NLRC. Thus, respondent's case
is premised on the claim that in paying her only P229.00 daily — or P5,038.00 monthly — as against a monthly salary
of P18,622.00, or P846.45 daily wage, paid to a regular SSS Processor at the time, petitioner exploited her, treated
her unfairly, and unjustly enriched itself at her expense.

For Article 217 of the Labor Code to apply, and in order for the Labor Arbiter to acquire jurisdiction over a dispute,
there must be an employer-employee relation between the parties thereto.

There being no employer-employee relation or any other definite or direct contract between respondent and
petitioner, the latter being responsible to the former only for the proper payment of wages, respondent is thus
justified in filing a case against petitioner, based on Articles 19 and 20 of the Civil Code, to recover the proper salary
due her as SSS Processor.

In this jurisdiction, the "long honored legal truism of 'equal pay for equal work’ has been "impregnably
institutionalized;" "[p]ersons who work with substantially equal qualifications, skill, effort and responsibility, under
similar conditions, should be paid similar salaries." "That public policy abhors inequality and discrimination is
beyond contention. Our Constitution and laws reflect the policy against these evils. The very broad Article 19 of the
Civil Code requires every person, 'in the exercise of his rights and in the performance of his duties, [to] act with
justice, give everyone his due, and observe honesty and good faith'."

NOTES:

Section 2(1), Art. IX-B, 1987 Constitution, expressly provides that "the civil service embraces all branches,
subdivisions, instrumentalities, and agencies of the government, including government-owned or controlled
corporation[s] with original charters." Corporations with original charters are those which have been created by
special law[s] and not through the general corporation law. In contrast, labor law claims against government-owned
and controlled corporations without original charters fall within the jurisdiction of the Department of Labor and
Employment and not the Civil Service Commission. (Light Rail Transit Authority vs. Perfecto Venus, March 24, 2006.)

In Home Development Mutual Fund v. Commission on Audit , it was held that while they performed the work of
regular government employees, DBP Service Corporation personnel are not government personnel, but employees
of DBP Service Corporation acting as an independent contractor. Applying the foregoing pronouncement to the
present case, it can be said that during respondent's stint with petitioner, she never became an SSS employee, as
she remained an employee of DBP Service Corporation and SSS Retirees Association — the two being independent
contractors with legitimate service contracts with SSS.

Page 43 of 67
LABOR 1 | JMM | CASE DIGESTS 2020 – TOPICS 1-3

JARDELEZA, J., dissenting:

Both parties agree that there is no employer-employee relation between them, respondent being an employee of
independent service contractors hired by the SSS. This fact alone, however, does not preclude the controversy
between them from being a labor dispute.
Article 212 (l) of the Labor Code defines a labor dispute to include "any controversy or matter concerning
terms or conditions of employment or the association or representation of persons in negotiating, fixing,
maintaining, changing or arranging the terms and conditions of employment regardless of whether or not
the disputants stand in the proximate relations of employer and employee."

Furthermore, respondent's claims relate to the terms and conditions of her working relationship vis-à-vis the SSS.
While captioned as a suit for damages under Articles 19 and 20 of the Civil Code, respondent's action is really one
to recover from the SSS amounts she would have received had she been employed in petitioner's roster of regular
employees. This is a dispute no different from "regularization cases" usually filed by contractual employees
seeking to be absorbed as regular employees of a company.

The SSS is a government-controlled corporation created by Republic Act (RA) No. 1161. Pursuant to Section 2 (1),
Article IX of the Constitution, a labor dispute involving the SSS is cognizable by the CSC. Thus,
. . . that the action below is for damages under Articles 19, 20 and 21 of the Civil Code would not suffice to
keep the case within the jurisdictional boundaries of regular Courts. That claim for damages is interwoven
with a labor dispute existing between the parties and would have to be ventilated before the
administrative machinery established for the expeditious settlement of those disputes. To allow the
action filed below to prosper would bring about "split jurisdiction" which is obnoxious to the orderly
administration of justice.

23. Century Properties Inc. vs. Babiano, et al., GR No. 220978, July 5, 2016

PETITIONER: CENTURY PROPERTIES, INC. (Won RESPONDENT: EDWIN J. BABIANO (Lost) and EMMA B.
against Babiano, Lost against Conception) CONCEPCION (Won)

CPI’s Contention
● Babiano is merely its agent tasked with selling
its projects. Nonetheless, he was afforded due
process in the termination of his employment
which was based on just causes.
● It also claimed to have validly withheld
Babiano's commissions, considering that they
were deemed forfeited for violating the
"Confidentiality of Documents and Non-
Compete Clause."
● On Concepcion's money claims, NLRC had no
jurisdiction because there was no employer-
employee relations between them. She should
have litigate the same in an ordinary civil action.

SPECIAL DETAILS:

Babiano was remunerated with, inter alia, the following benefits: (a) monthly salary of P70,000.00; (b) allowance of
P50,000.00; and (c) 0.5% override commission for completed sales.

His employment contract also contained a "Confidentiality of Documents and Non-Compete Clause" which, among
others, barred him from disclosing confidential information, and from working in any business enterprise that is in
direct competition with CPI "while [he is] employed and for a period of one year from date of resignation or
termination from [CPI]." Should Babiano breach any of the terms thereof, his "forms of compensation, including
commissions and incentives will be forfeited."

Conception signed an employment agreement, denominated as "Contract of Agency for Project

Page 44 of 67
LABOR 1 | JMM | CASE DIGESTS 2020 – TOPICS 1-3

Director" which provided, among others, that she would directly report to Babiano, and receive a monthly subsidy of
P60,000.00, 0.5% commission, and cash incentives.On March 31, 2008, Concepcion executed a similar contract
anew with CPI in which she would receive a monthly subsidy of P50,000.00, 0.5% commission, and cash incentives
as per company policy. Notably, it was stipulated in both contracts that no employer-employee relationship exists
between Concepcion and CPI.

LAW & PRINCIPLES:

Control Test:

a) the power to hire, i.e., the selection and engagement of the employee;
(b) the payment of wages;
(c) the power of dismissal; and
(d) the employer's power to control the employee's conduct

FACTS:
● October 2, 2002 - Babiano was hired by CPI as Director of Sales, and was eventually appointed as Vice
President for Sales effective September 1, 2007.
● Babiano’s employment contract contained a Confidentiality clause wherein he is barred from disclosing
confidential information and from working for a business enterprise that is a direct competitor of CPI.
● Concepcion was initially hired as Sales Agent by CPI and was eventually promoted as Project Director on
September 1, 2007.
● CPI, through its Executive Vice President for Marketing and Development, Jose Marco R. Antonio (Antonio),
sent Babiano a Notice to Explain on February 23, 2009 directing him to explain his conduct after they
received a report that
○ Babiano provided a competitor with information regarding CPI's marketing strategies,
○ spread false information regarding CPI and its projects
○ recruited CPI's personnel to join the competitor
○ for being absent without official leave (AWOL) for five (5) days
● February 25, 2009 - Babiano tendered his resignation and revealed that he had been accepted as Vice
President of First Global BYO Development Corporation (First Global), a competitor of CPI.
● March 3, 2009 - was served a Notice of Termination for the violations he committed.
● February 23, 2009 - Concepcion resigned as CPI's Project Director
● August 8, 2011 - respondents filed a complaint for non-payment of commissions and damages against
CPI and Antonio before the NLRC claiming that their repeated demands for the payment and release of
their commissions remained unheeded.

LA RULING (March 19, 2012) - ruled in CPI’s favor


● Babiano's acts are blatant violations of the "Confidentiality of Documents and Non-Compete Clause" of his
employment contract, thus, resulting in the forfeiture of his unpaid commissions in accordance with the
same clause.
● It had no jurisdiction over Concepcion's money claim as she was not an employee but a mere agent of CPI,
as clearly stipulated in her engagement contract with the latter.

NLRC RULING (June 25, 2013) - reversed and set aside the LA ruling
● While the NLRC initially concurred with the LA that Babiano's acts constituted just cause which would
warrant the termination of his employment from CPI, it, however, ruled that the forfeiture of all earned
commissions of Babiano under the "Confidentiality of Documents and Non-Compete Clause" is
confiscatory and unreasonable and hence, contrary to law and public policy. In this light, the NLRC held
that CPI could not invoke such clause to avoid the payment of Babiano's commissions since he had
already earned those monetary benefits and, thus, should have been released to him. However, the NLRC
limited the grant of the money claims in light of Article 291 (now Article 306) of the Labor Code which
provides for a prescriptive period of three (3) years.
● Concepcion was CPI's employee, considering that CPI: (a) repeatedly hired and promoted her since 2002;
(b) paid her wages despite referring to it as "subsidy"; and (c) exercised the power of dismissal and control
over her.

Page 45 of 67
LABOR 1 | JMM | CASE DIGESTS 2020 – TOPICS 1-3

CA RULING (April 8, 2015) - affirmed NLRC Ruling


● The CA held that Babiano properly instituted his claim for unpaid commissions before the labor tribunals
as it is a money claim arising from an employer-employee relationship with CPI.
● CPI cannot withhold such commission since the agreement referred to acts done after the cessation of the
employer-employee relationship.
● CA echoed the NLRC's finding that there exists an employer-employee relationship between Concepcion
and CPI. The CA likewise observed the presence of critical factors which were indicative of an employer-
employee relationship with CPI, such as: (a) Concepcion's receipt of a monthly salary from CPI; and (b) that
she performed tasks besides selling CPI properties.

ISSUES:
W/N the CA erred in denying CPI's petition for certiorari, thereby holding it liable for the unpaid commissions of
respondents.

RULING:

Article 1370 of the Civil Code provides that "[i]f the terms of a contract are clear and leave no doubt upon the intention
of the contracting parties, the literal meaning of its stipulations shall control.

In the case at bar, CPI primarily invoked the "Confidentiality of Documents and Non-Compete Clause" found in
Babiano's employment contract to justify the forfeiture of his commissions, viz.:

Confidentiality of Documents and Non-Compete Clause



And in order to ensure strict compliance herewith, you shall not work for whatsoever capacity, either as an
employee, agent or consultant with any person whose business is in direct competition with the company while
you are employed and for a period of one year from date of resignation or termination from the company.

Finally, if undersigned breaches any terms of this contract, forms of compensation including commissions and
incentives will be forfeited.

Here, the contracting parties indisputably wanted the said clause to be effective even during the existence of the
employer-employee relationship between Babiano and CPI, thereby indicating their intention to be bound by such
clause by affixing their respective signatures to the employment contract. More significantly, as CPI's Vice President
for Sales, Babiano held a highly sensitive and confidential managerial position as he "was tasked, among others, to
guarantee the achievement of agreed sales targets for a project and to ensure that his team has a qualified and
competent manpower resources by conducting recruitment activities, training sessions, sales rallies, motivational
activities, and evaluation programs."

Indubitably, obligations arising from contracts, including employment contracts, have the force of law between the
contracting parties and should be complied with in good faith.

Therefore, the CA erred in limiting the "Confidentiality of Documents and Non- Compete Clause" only to acts done
after the cessation of the employer-employee relationship or to the "post-employment" relations of the parties.

In Babiano’s resignation letter, he admitted that he sought employment from First Global, and five (5) days later, was
admitted thereto as vice president. It is evidently clear that he was still
employed by CPI as he has not formally resigned at that time. So he violated the agreement, thus justifying the
forfeiture of his unpaid commissions.

Guided by the control test, the Court finds that Concepcion was an employee of CPI:
(a) CPI continuously hired and promoted Concepcion from October 2002 until her resignation on February 23, 2009
- CPI exercised the power of selection and engagement over her person.
(b) The monthly "subsidy" and cash incentives that Concepcion was receiving from CPI are actually remuneration in
the concept of wages as it was regularly given to her on a monthly basis without any qualification,
(c) CPI had the power to discipline or even dismiss Concepcion - CPI actually exercised such power to dismiss when
it accepted and approved Concepcion's resignation letter.

Page 46 of 67
LABOR 1 | JMM | CASE DIGESTS 2020 – TOPICS 1-3

(d) as aptly pointed out by the CA, CPI possessed the power of control over Concepcion because in the performance
of her duties as Project Director — particularly in the conduct of recruitment activities, training sessions, and skills
development of Sales Directors — she did not exercise independent discretion thereon, but was still subject to the
direct supervision of CPI, acting through Babiano.

Therefore, the CA correctly ruled that since there exists an employer-employee relationship between Concepcion
and CPI, the labor tribunals correctly assumed jurisdiction over her money claims.

In sum, the Court thus holds that the commissions of Babiano were properly forfeited for violating the
"Confidentiality of Documents and Non-Compete Clause." On the other hand, CPI remains liable for the unpaid
commissions of Concepcion in the sum of P591,953.05.

NOTES:

In the case of Insular Life Assurance Co., Ltd. v. NLRC , it was ruled that one's employment status is defined and
prescribed by law, and not by what the parties say it should be, viz.:
It is axiomatic that the existence of an employer-employee relationship cannot be negated by expressly
repudiating it in the management contract and providing therein that the "employee" is an independent
contractor when the terms of the agreement clearly show otherwise. For, the employment status of a
person is defined and prescribed by law and not by what the parties say it should be. In determining the
status of the management contract, the "four-fold test" on employment earlier mentioned has to be applied.
(Emphasis and underscoring supplied)

24. Lu vs. Enopia, GR No. 197899, March 6, 2017

PETITIONER: JOAQUIN LU (Lost) RESPONDENT: TIRSO ENOPIA, ROBERTO BANES,


ALEJANDRE BAGAS, SALVADOR BERNAL, SAMUEL
Lu alleged that there was no employer employee CAHAYAG, ALEJANDRO CAMPUGAN, RUPERTO CERNA,
relationship as its elements were not present, viz.: JR., REYNALDO CERNA, PETER CERVANTES,
● it was the piado who hired petitioners; LEONARDO CONDESTABLE, ROLANDO ESLOPOR,
● they were not paid wages but shares in the ROLLY FERNANDEZ, EDDIE FLORES, ROLANDO FLORES,
catch, which they themselves determine; JUDITO FUDOLIN, LEO GRAPANI, FELIX HUBAHIB,
● they were not subject to his discipline; and JERRY JUAGPAO, MARCIANO LANUTAN, JOVENTINO
● respondent had no control over the day-to-day MATOBATO, ALFREDO MONIVA, VICTORIANO ORTIZ,
fishing operations, although they stayed in JR., RENALDO PIALAN, ALFREDO PRUCIA,PONCIANO
contact through respondent's radio operator or REANDO, HERMENIO REMEGIO, DEMETRIO RUAYA,
checker. EDGARDO RUSIANA, NESTOR SALILI, VICENTE
SASTRELLAS, ROMEO SUMAYANG, and DESIDERIO
Lu also claimed that petitioners should not be TABAY (Won)
reimbursed for their share in the expenses since it was
their joint venture that shouldered these expenses In their Position Paper, petitioners (now herein
respondent) alleged that their refusal to sign the Joint
Venture Fishing Agreement is not a just cause for their
termination. Petitioners also asked for a refund of the
amount of P8,700,407.70 that was taken out of their 50%
income share for the repair and maintenance of boat as
well as the purchase of fishing materials, as Lu should
not benefit from such
deduction.

SPECIAL DETAILS:

Page 47 of 67
LABOR 1 | JMM | CASE DIGESTS 2020 – TOPICS 1-3

LAW & PRINCIPLES:

In determining the existence of an employer-employee relationship, the following elements are considered: We find
all these elements present in this case.
(1) the selection and engagement of the workers;
(2) the power to control the worker's conduct;
(3) the payment of wages by whatever means; and
(4) the power of dismissal.

Under Article 279 of the Labor Code, that the right to security of tenure guarantees the right of employees to continue
in their employment absent a just or authorized cause for termination.

FACTS:
● January 20, 1994 to March 20, 1996 - respondent (herein petitioner Lu) hired petitioners (now herein
respondents) as crew members of his fishing mother boat F/B MG-28.
● Petitioners and Lu had these agreements:
○ income-sharing arrangement wherein 55% goes to Lu, 45% to the crew members, with an
additional 4% as "backing incentive."
○ equally share the expenses for the maintenance and repair of the mother boat,and
○ for the purchase of nets, ropes and payaos
● August 1997 - Lu proposed the signing of a Joint Venture Fishing Agreement between them, was refused
by petitioners as they opposed the one-year term provided in the agreement.
● August 18, 1997 - according to petitioners, Lu terminated their services right there and then because of
their refusal to sign the agreement.
● According to Lu, it was the master fisherman (piado) Ruben Salili that informed him that petitioners still
refused to sign the agreement and have decided to return the vessel F/B MG-28.
● August 25, 1997 - petitioners filed their complaint for illegal dismissal, monetary claims and damages.
● Despite serious efforts made by Labor Arbiter (LA) Arturo P. Aponesto, the case was not amicably settled.
● Petitioner alleged that their refusal to sign the Joint venture is not a just cause for their termination. They
also asked for a refund for the 50% income share taken from them to pay for the repair and maintenance
of the boat as well as the purchase of fishing materials.
● Lu denied having dismissed petitioners, claiming that their relationship was one of joint venture where he
provided the vessel and other fishing paraphernalia, while petitioners, as industrial partners, provided labor
by fishing in the high seas. Lu alleged that there was no employer-employee relationship as its elements
were not present.

LA RULING
● June 30, 1998 - LA rendered a Decision dismissing the case for lack of merit finding that there was no
employer-employee relationship existing between petitioner and the respondents but a joint venture.
● the LA found that:
○ (1) respondents were not hired by petitioner as the hiring was done by the piado or master
fisherman;
○ (2) the earnings of the fishermen from the labor were in the form of wages they earned based on
their respective shares;
○ (3) they were never disciplined nor sanctioned by the petitioner; and
○ (4) the income sharing and expense-splitting was no doubt a working set up in the nature of an
industrial partnership.
● While petitioner issued memos, orders and directions, however, those who were related more on the aspect
of management and supervision of activities after the actual work was already done for purposes of order
in hauling and sorting of fishes, and thus, not in the nature of control as to the means and method by which
the actual fishing operations were conducted as the same was left to the hands of the master fisherman.
● The LA also ruled that the checker and the use of radio were for the purpose of monitoring and supplying
the logistics requirements of the fishermen while in the sea; and that the checkers were also tasked to
monitor the recording of catches and ensure that the proper sharing system was implemented; thus, all
these did not mean supervision on how, when and where to fish.

● Respondents appealed to the NLRC which affirmed the LA’s Decision but was denied.
● Respondents filed a petition for certiorari with the CA.

Page 48 of 67
LABOR 1 | JMM | CASE DIGESTS 2020 – TOPICS 1-3

CA RULING (October 22, 2010) - reversing the NLRC


The CA found that petitioner exercised control over respondents based on the following:
○ (1) respondents were the fishermen crew members of petitioner's fishing vessel, thus, their
services to the latter were so indispensable and necessary that without them, petitioner's deep-
sea fishing industry would not have come to existence much less fruition;
○ (2) he had control over the entire fishing operations undertaken by the respondents through the
master fisherman (piado) and the assistant master fisherman (assistant piado) employed by him;
○ (3) respondents were paid based on a percentage share of the fish catch did not in any way affect
their regular employment status; and
○ (4) petitioner had already invested millions of pesos in its deep-sea fishing industry, hence, it is
highly improbable that he had no control over respondents' fishing operations.

ISSUES:

W/N an employer-employee relationship existed between petitioner and respondents.

RULING:

YES.

In determining the existence of an employer-employee relationship, the following elements are considered: We find
all these elements present in this case.
(1) the selection and engagement of the workers;
(2) the power to control the worker's conduct;
(3) the payment of wages by whatever means; and
(4) the power of dismissal.

In this case, petitioner contends that it was the piado who hired respondents, however, it was shown by the latter's
evidence that the employer stated in their Social Security System (SSS) online inquiry system printouts was MGTR,
which is owned by petitioner. We have gone over these printouts and found that the date of the SSS remitted
contributions coincided with the date of respondents' employment with petitioner. Petitioner failed to rebut such
evidence. Thus, the fact that petitioner had registered the respondents with SSS is proof that they were indeed his
employees. The coverage of the Social Security Law is predicated on the existence of an employer-employee
relationship.

Moreover, the 4% backing incentive fee was paid to respondent’s after deducting their vale or cash advance. If a joint
venture exists, why would these fishermen obtain vale or cash advance from petitioner and not from the piado who
allegedly hired and had control over them.

It was established that petitioner exercised control over respondents. It should be remembered that the control test
merely calls for the existence of the right to control, and not necessarily the exercise thereof. It is not essential
that the employer actually supervises the performance of duties by the employee. It is enough that the former has
a right to wield the power.

Petitioner admitted in his pleadings that he had contact with respondents at sea via the former's radio operator and
their checker. Such communication would establish that he was constantly monitoring or checking the progress of
respondents' fishing operations throughout the duration thereof, which showed their control and supervision over
respondents' activities.

The payment of respondents' wages based on the percentage share of the fish catch would not be sufficient to
negate the employer-employee relationship existing between them

Petitioner wielded the power of dismissal over respondents when he dismissed them after they refused to sign the
joint fishing venture agreement.

Page 49 of 67
LABOR 1 | JMM | CASE DIGESTS 2020 – TOPICS 1-3

The primary standard for determining regular employment is the reasonable connection between the particular
activity performed by the employee in relation to the usual trade or business of the employer. Respondents' jobs as
fishermen-crew members of F/B MG were directly related and necessary to petitioner's deep-sea fishing business
and they had been performing their job for more than one year.

As respondents were petitioner's regular employees, they are entitled to security of tenure under Section 3, 27 Article
XIII of the 1987 Constitution. It is also provided under Article 279 of the Labor Code, that the right to security of
tenure guarantees the right of employees to continue in their employment absent a just or authorized cause for
termination. And respondents' termination based on their refusal to sign the same, not being shown to be one of
those just causes for termination under Article 282, is, therefore, illegal.

The CA correctly found that respondents are entitled to the payment of


● backwages from the time they were dismissed until the finality of this decision.
● exemplary damages to each respondent
● attorney's fees in the amount of 10% of the total monetary award
● Legal interest at 6%

NOTES:

At the outset, We reiterate the doctrine that the existence of an employer-employee relationship is ultimately a
question of fact. Generally, We do not review errors that raise factual questions. However, when there is a conflict
among the factual findings of the antecedent deciding bodies like the LA, the NLRC and the CA, it is proper, in the
exercise of Our equity jurisdiction, to review and re-evaluate the factual issues and to look into the records of the
case and re-examine the questioned findings. In dealing with factual issues in labor cases, substantial evidence or
that amount of relevant evidence which a reasonable mind might accept as adequate to justify a conclusion is
sufficient.

It is settled that no particular form of evidence is required to prove the existence of an employer-employee
relationship. Any competent and relevant evidence to prove the relationship may be admitted.

As held in Ruga v. NLRC:


x x x [I]t must be noted that petitioners received compensation on a percentage commission based on the
gross sale of the fish-catch, i.e., 13% of the proceeds of the sale if the total proceeds exceeded the cost of
the crude oil consumed during the fishing trip, otherwise, only 10% of the proceeds of the sale. Such
compensation falls within the scope and meaning of the term "wage" as defined under Article 97(f) of the
Labor Code, thus:
(f) "Wage" paid to any employee shall mean the remuneration or earnings, however designated,
capable of being expressed in terms of money, whether fixed or ascertained on a time, task, piece
or commission basis, or other method of calculating the same, which is payable by an employer
to an employee under a written or unwritten contract of employment for work done or to be done,
or for services rendered or to be rendered, and included the fair and reasonable value, as
determined by the Secretary of Labor, of board, lodging, or other facilities customarily furnished
by the employer to the employee. x x x

25. Apelanio vs. Arcanys, Inc. GR No. 227098, November 14, 2018

PETITIONER: Julius Apelanio (LOST) RESPONDENT: Arcanys, INC and CEO Alan Debonneville
- averred that when his probationary contract - stated that they hired petitioner as a web designer
was terminated, he was immediately offered and was made aware that he would be placed on
a retainership agreement lasting from probationary status, and that his failure to meet
Oct.10- 24, 2012. the stringent requirements and standards set
- Was told he did not meet the reasonable forth would terminate his employment contract.
standards of satisfactory performance, but - alleged that at the time petitioner's probationary
was still offered the retainership agreement - employment ended, respondent corporation

Page 50 of 67
LABOR 1 | JMM | CASE DIGESTS 2020 – TOPICS 1-3

did not include security of tenure, with lesser experienced several hacking incidents that were
pay, and no labor standard benefits. reported to the police authorities
- alleged that respondents found him qualified - claimed that petitioner took advantage of their
for the position, but opted to hire his services predicament when he approached respondent
on a per project basis, justifying the lesser corporation's CEO and represented that he had
pay and the lack of security of tenure and information about the hacking perpetrator
labor standard benefits. - deny that they violated petitioner's right to
- maintains that the retainership agreements organize; and that the charge of unfair labor
offered to him contained GM Bantug's practice is baseless, misleading, and irrelevant
signature, and that said signature signified
the validity of the subject agreements

SPECIAL DETAILS:

Evaluation Grade of Petitioner


2nd month - rating of 3.06
4th month- rating of 2.99
6th month - rating of 2.77

1st Retainership Agreement:


- Oct. 10-24, 2012
- Same project requirements, no security of tenure, reduced pay, and no labor standard benefits
2nd Retainership Agreement:
- Oct.5- Nov. 12, 2012
- Reduced daily rate of P 875.14 from the daily rate of P 1, 257.15 from the initial agreement

LAW & PRINCIPLES:

Article 281 of the Labor Code

Art. 281. Probationary employment.


Probationary employment shall not exceed six (6) months from the date the employee started working, unless it is
covered by an apprenticeship agreement stipulating a longer period. The services of an employee who has been
engaged on a probationary basis may be terminated for a just cause or when he fails to qualify as a regular employee
in accordance with reasonable standards made known by the employer to the employee at the time of his
engagement. An employee who is allowed to work after a probationary period shall be considered a regular
employee.

FACTS:
● Petitioner was hired by respondents as a Usability/Web Design Expert. He was placed on a "probationary
status" for a period of six months.
● Respondent corporation evaluated his performance in terms of his dependability, efficiency, initiative,
cooperation, client responsiveness, judgment, punctuality, quality and quantity of work, professionalism,
and attitude towards customers, colleagues, and respondent corporation as a whole.
● Although petitioner was aware that he was undergoing evaluation, he was allegedly not informed of what
the passing grade was or what constituted as "reasonable standards of satisfactory performance.”
● After the evaluation, petitioner was given a letter informing him that they were not converting his status
into a regular employee since his performance fell short of the stringent requirements and standards set
by respondent corporation. Petitioner was given his final pay and he signed a Waiver, Release and
Quitclaim.
● Despite not reaching the reasonable standard, petitioner was offered a retainership agreement. Petitioner
was confused with the arrangement but agreed since he had to support his family.
● Once the retainership period lapsed, petitioner was offered another retainership agreement with a reduced
daily rate.
● Petitioner consulted a lawyer and refused to sign the 2nd retainership agreement.

Respondents’ Side:
● Before the petitioner’s probationary contract ended, respondents requested the petitioner to sign a
termination notice. Respondents released and paid petitioner his final pay on Oct. 23.

Page 51 of 67
LABOR 1 | JMM | CASE DIGESTS 2020 – TOPICS 1-3

● Petitioner also executed a Waiver, Release, and Quitclaim dated Nov. 16, in favor of respondents, indicating
that he had no further claim whatsoever against the company and that he had received his full pay.
● Respondents allege that after the petitioner’s probationary period several hacking incidents occurred which
resulted to severe losses and damage.
● Petitioner was said to have taken advantage of the situation. Respondents claim that the petitioner
allegedly dangled the information to respondents in exchange for a retainership contract, and respondents
were lured in.
● Respondents learned that the said petitioner did not have any information about the incident and stopped
transacting with the petitioner.
● Petitioner then sued them before the Labor Arbiter for unfair labor practice, illegal dismissal, and damages.

Labor Arbiter- in favor of respondent


The dismissal was an exercise of an employer's management prerogative to retain only those it deems fit. In
addition, petitioner was aware that he failed to qualify when he knowingly signed a quitclaim and waiver in favor of
respondents after he received his final pay.

NLRC- in favor of petitioner


The NLRC rejected respondents' argument that the retainership agreements were mere drafts that did not even
contain petitioner's signature. On the contrary, the NLRC agreed with petitioner that it is normal for an employee not
to sign his own copy of the agreement.

CA - reversed the NLRC decision and reinstated the Labor Arbiter’s decision

ISSUES: Whether or not Apelanio was illegally dismissed

RULING:

NO
● Jurisprudence is replete with circumstances stating that an employer may unilaterally prepare an
employment contract, stating the terms and conditions required of a potential employee, and that a
potential employee had only to adhere to it by signing it. Such contract is known as a contract of adhesion
(check notes for definition), which is allowed by law albeit construed in favor of the employee in case of
ambiguity.
● In this case, however, it cannot be denied that in the retainership agreements provided by petitioner, his
signature or "adherence" is notably absent. As a result, said retainership agreements remain ineffectual
and cannot be used as evidence against respondents.
● CA pointed out the significance of petitioner’s failure to sign:

First, the draft of the second agreement, which petitioner claimed to be another extension of the first,
indicated that such agreement was entered into, and supposed to be signed by the parties on the 10th of
October 2012.

Second, the Skype conversation between petitioner and respondents’ representative on October 24, 2012
showed that they were discussing possible compensation at P18,000.00, which was the remuneration
indicated in the first agreement. If the first agreement got finalized and was already implemented, then why
would the draft of the second one still indicate the 10th of October 2012 as the date of execution and
signing of the first agreement?

Although it may be argued that the dates were merely clerical errors or unreplaced entries resulting to
oversight, the Skype conversation between petitioner and respondents' representative on October 24, 2012,
confirmed the non-conclusion of the first agreement; for it would be illogical for the parties to still discuss
the remuneration indicated in the first agreement if the same had already been implemented, and, in fact,
was about to end on the day that the conversation took place.

● A review of the retainership agreements indicates that petitioner was merely engaged as a
consultant, in relation to the hacking incidents endured by respondents. Petitioner merely alleged
that he was hired as an employee under said retainership agreements, but has yet to provide
evidence to support such claim. "It is a basic rule in evidence that each party must prove his

Page 52 of 67
LABOR 1 | JMM | CASE DIGESTS 2020 – TOPICS 1-3

affirmative allegations." Therefore, Article 281 of the Labor Code finds no application in this
case, absent any evidence to prove that petitioner worked beyond his probationary employment.

NOTES:

Philippine Commercial international Bank vs. CA

A contract of adhesion is defined as one in which one of the parties imposes a ready-made form of contract, which
the other party may accept or reject, but which the latter cannot modify. One party prepares the stipulation in the
contract, while the other party merely affixes his signature or his "adhesion" thereto, giving no room for negotiation
and depriving the latter of the opportunity to bargain on equal footing. Nevertheless, these types of contracts have
been declared as binding as ordinary contracts, the reason being that the party who adheres to the contract is free
to reject it entirely.

In contracts of adhesion, "one party prepares the stipulation in the contract, while the other party merely affixes
his signature or his 'adhesion' thereto." Besides, "the one who adheres to the contract is in reality free to reject it
entirely; if he adheres, he gives his consent."

26. Dr. Loreche-Amit vs. Cagayan De Oro Medical Center, GR No. 216635, June 3, 2019

PETITIONER: Dr. Mary Jean Loreche-Amit RESPONDENT: Cagayan De Oro Medical Center, Dr.
- Contended that she was dismissed without Francisco Oh, Dr. Hermano Emano
just cause and due process - averred that petitioner was not hired by them as
- argues that she is not a corporate officer she merely assisted Dr. Gaerlan
because her position as Pathologist is not - Respondents maintained that petitioner worked
among those included in the by-laws of CDMC at the same time as pathologist in other
hospitals and was not prohibited to do so

SPECIAL DETAILS:
● Petitioner is a member of the Board of Governors accredited by the PRC.
● Petitioner was appointed as a Pathologist with a term of 5 years from May 2006-2011.

LAW & PRINCIPLES:

Corporation Code
Corporate officers are given such character either by the Corporation Code or by the corporation's by-laws. Under
Section 25 of the Corporation Code, the corporate officers are the president, secretary, treasurer and such other
officers as may be provided in the by-laws. Other officers are sometimes created by the charter or by-laws of a
corporation, or the board of directors may be empowered under the by-laws of a corporation to create additional
offices as may be necessary.

Four-fold test:
1) the selection and engagement of the employees
2) the payment of wages
3) the power of dismissal
4) the power to control the employee's conduct, must be applied to determine the existence of an employer-
employee relationship.

The power to control the work of the employee is considered the most significant determinant of the existence of
an employer-employee relationship. This test is premised on whether the person for whom the services are
performed reserves the right to control both the end achieved and the manner and means used to achieve that end.

Economic Reality Test


Under this test, the economic realities prevailing within the activity or between the parties are examined, taking into
consideration the totality of circumstances surrounding the true nature of the relationship between the parties. In

Page 53 of 67
LABOR 1 | JMM | CASE DIGESTS 2020 – TOPICS 1-3

our jurisdiction, the benchmark of economic reality in analyzing possible employment relationships for purposes of
applying the Labor Code ought to be the economic dependence of the worker on his employer.

FACTS:
● Petitioner worked at CDMC, when she was engaged to Dr. Gaerlan as Associate Pathologist. When Dr.
Gaerlan passed away, CDMC formally appointed petitioner as Chief Pathologist for five years.
● CDMC’s Board of Directors passed a resolution recalling the petitioner’s appointment. Thus, petitioner filed
a complaint for illegal dismissal.
● According to petitioner, Dr. Emano asked her to help his daughter to qualify as a pathologist since petitioner
is one of the members of Board of Governors by the PRC. Petitioner refused to assist Dr. Emano because
the latter failed to qualify in the clinical pathology examination. This started Dr. Emano’s subtle attempt to
oust her from her job.
● Dr. Oh issued an Inter-Office Memorandum addressing all laboratory personnel working in and out of the
building without proper permission to be considered as absent without official leave and that the payment
for printing duplicate copies not endorsed to the hospital will be considered as a form of stealing.
● Petitioner then slammed the memorandum against the wall and called Dr. Oh and irrational man. She then
received a memorandum from Dr. Oh for alleged conduct. unbecoming/insubordination, and to explain why
her appointment should not be revoked due to such behavior. Her appointment was then recalled.
● According to the respondents, petitioner was not hired by them and merely assisted Dr. Gaerlan.
Respondents maintain that petitioner worked at the same time as pathologist in Capitol College Hospital
and J.R. Borja Memorial Hospital as she was not prohibited to do so.

Labor Arbiter
The complaint was dismissed for lack of jurisdiction. However, the Labor Arbiter found that petitioner is a corporate
officer of the hospital because of her appointment by the Board of Directors through a resolution. Matters relating
to the propriety of her dismissal is under the jurisdiction of the Regional Trial Court under Section 5.2 of Republic
Act (R.A.) No. 8799 (The Securities Regulation Code of the Philippines).
NLRC
The NLRC affirmed the ruling of the Labor Arbiter stating that there is no employer-employee relationship between
CDMC and the petitioner.

ISSUES:
1. Whether or not there was an employee-employer relationship between the petitioner and respondent
2. Whether or not the labor tribunals have jurisdiction over the complaint for illegal dismissal filed by petitioner

RULING:
1. YES, there exists an employee-employer relationship. The Court agrees with the contention of petitioner
that she is not a corporate officer. To be considered as a corporate officer, the designation must be either
provided by the Corporation Code or the by-laws of the corporation.

In this case, nowhere in the records could the by-laws of CDMC be found. An appointment through the
issuance of a resolution by the Board of Directors does not make the appointee a corporate officer. It is
necessary that the position is provided in the Corporation Code or in the by-laws.In the absence of the by-
laws of CDMC, there is no reason to conclude that petitioner, as Pathologist, is considered as a corporate
officer.

However, this is not an automatic declaration that petitioner is an employee of CDMC. The four-fold test,
to wit: 1) the selection and engagement of the employees; 2) the payment of wages; 3) the power of
dismissal; and 4) the power to control the employee's conduct, must be applied to determine the existence
of an employer-employee relationship.

CDMC, through the Board of Directors, exercised the power to select and supervise petitioner as the
Pathologist. It must be emphasized that petitioner was appointed as Pathologist with a term of five years
and was paid compensation which is at 4% of the gross receipts of the Clinical Section of the laboratory.
However, based on the records, CDMC does not exercise the power of control over petitioner.

As the Labor Arbiter, NLRC, and the CA aptly observed, petitioner was working for two other hospitals aside
from CDMC, not to mention those other hospitals which she caters to when her services are needed. Such

Page 54 of 67
LABOR 1 | JMM | CASE DIGESTS 2020 – TOPICS 1-3

fact evinces that petitioner controls her working hours. Relevant is the economic reality test which this
Court has adopted in determining the existence of employer-employee relationship. Under this test, the
economic realities prevailing within the activity or between the parties are examined, taking into
consideration the totality of circumstances surrounding the true nature of the relationship between the
parties.

The fact that petitioner continued to work for other hospitals strengthens the proposition that petitioner
was not wholly dependent on CDMC. Petitioner admitted that she received in full her 4% share in the Clinical
Section of the hospital regardless of the number of hours she worked therein. Petitioner manages her
method and hours of work.

The rule is that where a person who works for another performs his job more or less at his own pleasure,
in the manner he sees fit, not subject to definite hours or conditions of work, and is compensated
according to the result of his efforts and not the amount thereof, no employer-employee relationship
exists.

The Memorandum, pertaining to petitioner's behavior does not sufficiently establish the element of control.
It is administrative in character which does not, in any way, pertains to the manner and method of
petitioner's work.

2. YES, since the RTC does not have jurisdiction over the case. There was no intra-corporate controversy, the
latter being operative in vesting jurisdiction upon Regional Trial Courts over all controversies in the election
or appointment of directors, trustees, officers or managers of corporations, partnerships or associations.

NOTES:

WPP Marketing Communications, Inc. v. Galera and Marc II Marketing, Inc. v. Joson
This Court declared that respondents are not corporate officers because neither the Corporation Code nor the by-
laws of the respective corporations provided so. In the latter case, this Court treated as employee the respondent
whose position was not expressly mentioned in the Corporation Code or the by-laws.

27. Fernandez vs. Kalookan SlaughterHouse Inc., GR No. 225075, June 19, 2019

PETITIONER: Arnulfo Fernandez RESPONDENT: Kalookan Slaughterhouse, INC/ Ernesto Cuanan


(WON) - asserted that petitioner is an independent butcher working under
- Filed a complaint for illegal its Operation Supervisor, Cirilo Tablit alleged that petitioner's
dismissal before the Labor salaries were paid by Tablit
Arbiter - claims that petitioner was not dismissed but was only barred from
- Was informed he could no entering as he failed to comply with the "No I.D., No Entry" Policy
longer work due to his old and the "No Uniform, No Entry" Policy.
age

SPECIAL DETAILS:
● Petitioner, he was hired in 1994 as a butcher by Kalookan Slaughterhouse, Inc., a single proprietorship
owned by respondent Ernesto Cunanan.

LAW & PRINCIPLES:

To determine the existence of an employer-employee relationship, four elements generally need to be considered,
namely:
(1) the selection and engagement of the employee;
(2) the payment of wages;
(3) the power of dismissal; and
(4) the power to control the employee's conduct.
These elements or indicators comprise the so-called 'four-fold' test of employment relationship."

Page 55 of 67
LABOR 1 | JMM | CASE DIGESTS 2020 – TOPICS 1-3

FACTS:
● Petitioner worked as a butcher by Kalookan Slaughterhouse, a single proprietorship owned by respondent.
● Petitioner worked from Mon.-Sun., from 6:30PM-7:30PM, with a daily wage of P700, which was later
reduced to P500. Respondent claimed that he met an accident while driving the company’s truck and that
deductions were made from his wages. Petitioner also claimed that he had a headache and was not able
to report to work. The next day, he only received P200 due to his previous undertime. He was also informed
that could no longer report to work due to his old age.
● Respondent asserted that petitioner is an independent butcher working under its Operation Supervisor,
Cirilo Tablit. The payment was based on the no. of hogs he butchered and was only required to be in the
slaughterhouse when customers brought hogs to be slaughtered.
● Respondent alleged that it imposed policies on the entry to the premises, which applied to employees,
dealers, independent butchers, hog and meat dealers and trainees. According to respondent, petitioner
violated the policies and he misconstrued the disallowance to enter the slaughterhouse as an act of
dismissal.

Labor Arbiter
LA ruled that petitioner was illegally dismissed. The requisites of an employer-employee relationship were
established as follows: petitioner was hired by Kalookan Slaughterhouse through Tablit and petitioner was paid his
daily wage for his butchering services. Respondent had authority to discipline petitioner regarding his work activities
and exercised control over the conduct of petitioner in the performance of his work and implemented policies
regulating his rendition of services.

Respondent failed to prove that petitioner was not its employee. LA found that petitioner was illegally dismissed.
Old age was not a valid or just cause to terminate petitioner’s employment and was arbitrary and whimsical.

NLRC Decision
Reversed the ruling of the LA. Although there was a semblance of an employer-employee relationship, the facts and
circumstances in this case showed that there was no employer-employee relationship.Petitioner was an
independent contractor and not an employee because there was no regular payroll showing his name and the legal
deductions made from his salary. The NLRC found that the Sinumpaang Salaysay of Tablit tends to show that there
was no employer-employee relationship. Petitioner failed to prove any dismissal as he was only barred from entering
the premises for his failure to follow the slaughterhouse's policies. Ruled that there was just cause to dismiss
petitioner as he was found sleeping on duty.

CA
Petitioner’s claim is not supported by evidence as he failed to submit salary vouchers, pay slips, daily work schedule
and even a certificate of withholding tax on compensation income. Petitioner also failed to disprove the Sinumpaang
Salaysay of Tablit that petitioner was one of the butchers that Tablit personally hired and paid when there were too
many hogs to be butchered at the slaughterhouse.

ISSUES:
1. WoN the CA erred in affirming the NLRC decision which failed to recognize that there was an employee-
employer relationship (YES)
2. WoN the CA failed to recognize that there was an illegal dismissal

RULING:
1. NLRC and the CA committed a grave error and agrees with the LA. In Masonic Contractor, Inc. v. Madjos,
the court ruled that the fact that the company provided identification cards and uniforms and the vague
affidavit of the purported employer were sufficient evidence to prove the existence of employer-employee
relationship.

The totality of petitioner's evidence and the admissions of Kalookan Slaughterhouse convinces the Court
that petitioner was indeed an employee of Kalookan Slaughterhouse. Petitioner was able to present an
I.D., gate passes, log sheets, and a trip ticket. Kalookan Slaughterhouse even admitted through De Guzman
that uniforms were given to all personnel, including petitioner. The CA, however, disregarded the gate
passes, as it claimed that the gate pass had a note that such did not qualify the holder as an employee.
This is an error as this only applied to one of the gate passes and the other gate passes did not have this
notation.

Page 56 of 67
LABOR 1 | JMM | CASE DIGESTS 2020 – TOPICS 1-3

Petitioner was able to submit an I.D. in addition to the gate passes. The trip ticket and the log sheets also
showed that Kalookan Slaughterhouse engaged petitioner. These are sufficient to prove that petitioner
was engaged by Kalookan Slaughterhouse. Other than Tablit's Sinumpaang Salaysay, no document was
presented to show that he paid petitioner's salaries.

By denying that petitioner was its employee but alleging that he rendered services as Tablit's employee,
Kalookan Slaughterhouse effectively admitted the substantial fact that petitioner has been rendering
butchering services for 20 years. (refer to case #3 in notes)

2. Petitioner was illegally dismissed and entitled to his money claims. The Court finds that the LA was correct
in ruling that petitioner was illegally dismissed. Kalookan Slaughterhouse failed to specifically deny that on
July 22, 2014, petitioner was informed that he could no longer report for work. De Guzman only alleged that
he merely barred petitioner from entering the slaughterhouse because of his failure to wear his I.D. and
uniform but he failed to state that this was done on July 22, 2014. De Guzman's silence on this matter is
deemed as an admission that petitioner was indeed dismissed on July 22, 2014.

The LA's award of service incentive leave pay, night shift differential pay, and 13th month pay is also proper
as respondent failed to prove that it had paid petitioner such benefits under the law. Such award should be
limited to three years prior to the filing of the complaint in August 5, 2014 in accordance with Article 306
of the Labor Code.

NOTES:

1. Cariño v. Maine Marine Phils., Inc


As a rule, "in appeals by certiorari under Rule 45 of the Rules of Court, the task of the Court is generally to review
only errors of law since it is not a trier of facts, a rule which definitely applies to labor cases." As the Court ruled in
Scanmar Maritime Services, Inc. v. Conag: "But while the NLRC and the LA are imbued with expertise and authority
to resolve factual issues, the Court has in exceptional cases delved into them where there is insufficient evidence to
support their findings, or too much is deduced from the bare facts submitted by the parties, or the LA and the NLRC
came up with conflicting findings.

2. Masonic Contractor v. Madjos


It is common practice for companies to provide identification cards to individuals not only as a security measure,
but more importantly to identify the bearers thereof as bona fide employees of the firm or institution that issued
them. The provision of company-issued identification cards and uniforms to respondents, aside from their inclusion
in MCI's summary payroll, indubitably constitutes substantial evidence sufficient to support only one conclusion:
that respondents were indeed employees of MCI.

By their silence, petitioners are deemed to have admitted the same. Section 11 of Rule 8 of the Rules of Court, which
supplements the NLRC Rules, provides that an allegation not specifically denied is deemed admitted.

3. Pamplona Plantation Company v. Acosta


By setting forth these defenses, petitioner, in effect, admitted that respondents worked for it, albeit in different
capacities. Such allegations are negative pregnants — denials pregnant with the admission of the substantial facts
in the pleading responded to which are not squarely denied, and amounts to an acknowledgement that respondents
were indeed employed by petitioner.

TOPIC 3: HIRING OF EMPLOYEE


1. PT&T vs. NLRC, 272 SCRA 596 [1997]

PETITIONER: Philippine Telegraph and Telephone RESPONDENT:


Company (Lost)

Page 57 of 67
LABOR 1 | JMM | CASE DIGESTS 2020 – TOPICS 1-3

SPECIAL DETAILS:
• de Guzman contracted marriage on May 26, 1991
• Fixed periods of her employment as reliever on both instances were from November 21, 1990-April 20, 1991
and July 19, 1991-August 8, 1991
• complaint was filed before the Regional Arbitration Branch of NLRC in Baguio City

LAW & PRINCIPLES:


Art 136. Prohibition Against Stipulation of Marriage
It shall be unlawful for an employer to require as a condition of employment or continuation of employment that a
woman employee shall not get married or to stipulate expressly or tacitly that upon getting married, a woman
employee shall be deemed separated, or to actually dismiss, discharge, discriminate or otherwise prejudice a woman
employee merely by reason of her marriage

FACTS:
• In present case, petitioner invokes the alleged concealment of civil status and defalcation of company
funds as grounds to terminate services of an employee. Private respondent (de Guzman), contrarily argues
that what motivated respondent to terminate her services was her having contracted marriage during her
employment, which is prohibited by petitioner in its company policies
• Grace de Guzman was hired twice by petitioner as a reliever for a fixed period of time. In both instances,
she indicated that she was single although she had contracted marriage a few months before.
• De Guzman was then asked to join petitioner company as a probationary employee, the probationary period
to cover 150 days. In her application form, she again indicated that she was single even though she was
already married.
• When petitioner learned about de Guzman’s civil status, she was sent a memorandum asking her to explain
the discrepancy and was reminded about the company’s policy of not accepting married women for
employment. De Guzman replied stating that she was not aware of PT&T’s policy regarding married women,
and that she did not deliberately hide her civil status.
• Petitioner remained unconvinced by her explanation and dismissed her from the company
• De Guzman initiated a complaint for illegal dismissal, with a claim for non-payment of cost of living
allowances (COLA)
• Labor Arbiter handed down a decision declaring that de Guzman was illegally dismissed by petitioner, which
was later upheld by the NLRC on appeal

ISSUES:
Whether or not the alleged concealment of civil status can be a ground to terminate employee’s services

RULING:
No.
• Article 136 of the Labor Code prohibits discrimination by reason of marriage of a female employee.
• An employer is free to regulate, according to his best discretion and best business judgement, all aspects
of employment, “from hiring to firing,” except in cases of unlawful discrimination or those which may be
provided by law.
• In the case at bar, petitioner's policy of not accepting any woman worker who contracts marriage is afoul
of the right against discrimination, afforded all women workers by our labor laws and by no less than the
Constitution. Records disclose clearly that her ties with the company were dissolved principally because of
the company's policy that married women are not quali􏰀ed for employment in PT&T, and not merely
because of her supposed acts of dishonesty.
• The policy of PT&T goes against Article 136 of the Labor Code on the right of a woman to be free from any
kind of stipulation against marriage in connection with her employment.
• Petition was dismissed for lack of merit

2. Duncan Asso. Of Detailman-PTGWO vs. Glaxo Wellcome Phils., G.R. No. 162994, Sept. 17, 2004

PETITIONER: Pedro A. Tecson RESPONDENT: Glaxo Wellcome Philippines, Inc. (Glaxo)

Page 58 of 67
LABOR 1 | JMM | CASE DIGESTS 2020 – TOPICS 1-3

Tecson’s contentions: Glaxo’s contentions:


a. The policy violates the equal protection clause a. There is a genuine interest in avoiding a conflict
of the Constitution because it restricts the of interest because the 2 company’s products
employee’s right to marry. were in direct competition.
b. He was constructively dismissed because: (1) b. Tecson was given time to remedy the situation
he was transferred; (2) his pay was diminished; and was encouraged not to resign but instead
he was excluded from attending seminars and let his wife resign.
training sessions; and (4) he was prohibited c. He signed the contract making him fully aware
from promoting respondent’s products. of such stipulations.
d. The relocation was assumed favorable
because he was from Agusan and his wife from
Butuan.
e. His exclusion from seminars was because the
products were in direct competition.
f. The delay in his sales paraphernalia was
because he was presumed to have already
transferred, they were sent to his new location.

SPECIAL DETAILS:

LAW & PRINCIPLES: The stipulations in the contract signed by Tecson were not contrary to law. Therefore, his
signature binds him to follow suit to the stipulations therein.

FACTS:
● Tecson was hired by Glaxo Wellcome Philippines, Inc. (Glaxo) as a medical representative. He was
assigned to market the products in the Camarines Sur-Camarines Norte area.
● He signed an employment contract containing the following stipulation: “ to disclose…any existing or future
relationship by consanguinity or affinity with co-employees or employees of competing drug companies
and should management find that such relationship poses a possible conflict of interest, to resign from the
company.”
● The Employee Code of Conduct of Glaxo states that if management perceives a conflict of interest or a
potential conflict between such relationship and the employee’s employment with the company, the
management and the employee will explore the possibility of a “transfer to another department in a non-
counterchecking position” or preparation for employment outside the company after six months.
● He entered in to romantic relationship with Bettsy, Astra’s (competitor) Branch coordinator in Albay
● He received reminders from his District Manager, regarding the conflict of interest, but still married Bettsy.
● He was informed that his marriage gave rise to a conflict of interest and that either himself or his wife
would have to resign from their jobs. Glaxo expressed that they wanted to keep him.
● He requested time for compliance because Astra was planning to merge with Zeneca. Bettsy would then
avail of the redundancy package as well as the separation package, should this merger happen.
● Tecson requested a transfer to the milk division to eliminate the potential of a conflict of interest because
Astra did not have a milk division.
● Tecson was transferred to the Butuan City-Surigao City-Agusan del Sur sales area but defied such order.
● During lis pendens he was paid his salary but was not issued samples of products which competed with
Astra. He was also not included in product conferences.
● Tecson, with his issue failing to be resolved, was offered P50,000 in separation pay which he declined.
● Tecson’s employment was terminated.

ISSUES:
1. WON the Court of Appeals erred in making its decision.
2. WON Tecson was constructively dismissed.

RULING:
1. No. There is a conflict of interest and Tecson was given several chances to eliminate this. Glaxo has a right
to guard its trade secrets, manufacturing formulas, marketing strategies and other confidential programs
and information from competitors, especially so that it and Astra are rival companies in the highly
competitive pharmaceutical industry. The relationship in question might compromise the interests of the
company.

Page 59 of 67
LABOR 1 | JMM | CASE DIGESTS 2020 – TOPICS 1-3

2. No. Constructive dismissal is defined as a quitting, an involuntary resignation resorted to when continued
employment becomes impossible, unreasonable, or unlikely; when there is a demotion in rank or diminution
in pay; or when a clear discrimination, insensibility or disdain by an employer becomes unbearable to the
employee. None of these conditions are present in the instant case.

NOTES: NCMB did not err in making its decision because it is a valid exercise of its management prerogatives.

1. Conflict of Interest

Employees are expected:

a. To avoid having personal or family interest, financial or otherwise, in any competitor supplier or other
businesses which may consciously or unconsciously influence their actions or decisions and thus deprive
Glaxo Wellcome of legitimate profit.
b. To refrain from using their position in Glaxo Wellcome or knowledge of Company plans to advance their
outside personal interests, that of their relatives, friends and other businesses.
c. To avoid outside employment or other interests for income which would impair their effective job
performance.
d. To consult with Management on such activities or relationships that may lead to conflict of interest.

2. Employee Relationships

Employees with existing or future relationships either by consanguinity or affinity with co-employees of competing
drug companies are expected to disclose such relationships to the Management. If management perceives a conflict
or potential conflict of interest, every effort shall be made, together by management and the employee, to arrive at
a solution within six (6) months, either by transfer to another department in a non-counter checking position, or by
career preparation toward outside employment after Glaxo Wellcome. Employees must be prepared for possible
resignation within six (6) months, if no other solution is feasible.

3. Star Paper Corp., vs. Simbol, G.R. No. 164774, April 12, 2006

PETITIONERS: Star Paper Corporation, Josephine RESPONDENTS: Ronaldo D. Simbol (Simbol), Wilfreda
Ongsitco and Sebastian Chua N. Comia (Comia) and Lorna E. Estrella (Estrella)

Josephine Ongsitco as Manager of the Personnel and They were all regular employees of the company.
Administration Department
Respondent’s Contentions:
Sebastian Chua as Managing Director The policy of the company is contract to the provisions
listed in the LAW & PRINCIPLES section.
Company’s Contentions:
The company contends that employee spouses have the
right to decide which one of them will resign. It is neither
discriminating against women nor an employee’s marital
status because they are free to marry just not co-
workers.

SPECIAL DETAILS:

LAW & PRINCIPLES:

Protection of Labor Under the 1987 Constitution:


● Article II, Section 18. The State affirms labor as a primary social economic force. It shall protect the rights
of workers and promote their welfare.

Page 60 of 67
LABOR 1 | JMM | CASE DIGESTS 2020 – TOPICS 1-3

● Article XIII, Section 3. The State shall afford full protection to labor, local and overseas, organized and
unorganized, and promote full employment and equality of employment opportunities for all.

It shall guarantee the rights of all workers to self-organization, collective bargaining and negotiations, and
peaceful concerted activities, including the right to strike in accordance with law. They shall be entitled to
security of tenure, humane conditions of work, and a living wage. They shall also participate in policy and
decision-making processes affecting their rights and benefits as may be provided by law.

The State shall promote the principle of shared responsibility between workers and employers, recognizing
the right of labor to its just share in the fruits of production and the right of enterprises to reasonable returns
on investments, and to expansion and growth.

Protection of Labor Under the Civil Code:

● Art. 1700. The relation between capital and labor are not merely contractual. They are so impressed with
public interest that labor contracts must yield to the common good. Therefore, such contracts are subject
to the special laws on labor unions, collective bargaining, strikes and lockouts, closed shop, wages, working
conditions, hours of labor and similar subjects.

● Art. 1702. In case of doubt, all labor legislation and all labor contracts shall be construed in favor of the
safety and decent living for the laborer.

Protection of Labor Under the Labor Code:

Art. 136. It shall be unlawful for an employer to require as a condition of employment or continuation of employment
that a woman employee shall not get married, or to stipulate expressly or tacitly that upon getting married a woman
employee shall be deemed resigned or separated, or to actually dismiss, discharge, discriminate or otherwise
prejudice a woman employee merely by reason of her marriage.

FACTS:
● Simbol met and married his wife, Alma Dayrit, while they were both employed in said company.
● Ongsitco advised the couple that should they decide to get married, one of them should resign pursuant to
a company policy to which Simbol resigned.
● Comia met and married her husband, Howard Comia, while they were both employed in said company.
Comia resigned
● Estrella was impregnated by Zuñiga, an already married co-worker of hers. She resigned before she could
be terminated for immorality.
● The respondents each signed a Release and Confirmation Agreement stating that that they have no money
and property accountabilities in the company and that they release the latter of any claim or demand of
whatever nature.
● Respondent’s versions:
a. Simbol and Comia: They did not resign voluntarily. They were compelled to resign in view of an
illegal company policy.
b. Estrella: She was tricked into having conjugal relations with a man who misrepresented himself
as SEPARATED. It was only after she became pregnant that she found out that he was not
separated. She severed her relationship to avoid dismissal. She had an accident then returned to
work after 21 days but was refused entry because she was already dismissed for immoral
conduct. She was allowed to explain herself but submitted her letter of resignation to claim her
13th month pay.
● Respondents later filed a complaint for unfair labor practice, constructive dismissal, separation pay and
attorney's fees. They claim that the company policy is illegal. They were dismissed due to their union
membership.
● Complaint dismissed for lack of merit. CA reversed the decision.

ISSUES:
1. WON there is a reasonable necessity in the case-at-bar.
2. WON Estrella’s resignation letter was valid.

Page 61 of 67
LABOR 1 | JMM | CASE DIGESTS 2020 – TOPICS 1-3

RULING:
1. No. Petitioners' sole contention that "the company did not just want to have two (2) or more of its
employees related between the third degree by affinity and/or consanguinity" is lame.

The respondents were hired after they were found fit for the job, but were asked to resign when they married
a co-employee. Petitioners failed to show how the marriage of Simbol, then a Sheeting Machine Operator,
to Alma Dayrit, then an employee of the Repacking Section, could be detrimental to its business operations.

They also failed to explain the detriment to the case of Wilfreda Comia, then a Production Helper in the
Selecting Department, who married Howard Comia, then a helper in the cutter-machine. The policy is
premised on the mere fear that employees married to each other will be less efficient. If we uphold the
questioned rule without valid justification, the employer can create policies based on an unproven
presumption of a perceived danger at the expense of an employee's right to security of tenure.

2. Yes. Her resignation letter was written in her own handwriting making it voluntary and thus valid.

Her contention that she was pressured to resign because she got impregnated by a married man and she
could not stand being looked upon or talked about as immoral is incredulous. If she really wanted to avoid
embarrassment and humiliation, she would not have gone back to work at all. Nor would she have filed a
suit for illegal dismissal and pleaded for reinstatement.

NOTES:
● ANTI-NEPOTISM EMPLOYMENT POLICY- to prevent the hiring of unqualified persons based on their status
as a relative, rather than upon their ability.

● DISPARATE TREATMENT ANALYSIS- the plaintiff must prove that an employment policy is discriminatory
on its face. No-spouse employment policies requiring an employee of a particular sex to either quit, transfer,
or be fired are facially discriminatory. For example, an employment policy prohibiting the employer from
hiring wives of male employees, but not husbands of female employees, is discriminatory on its face.

● DISPARATE IMPACT- the complainants must prove that a facially neutral policy has a disproportionate
effect on a particular class. For example, although most employment policies do not expressly indicate
which spouse will be required to transfer or leave the company, the policy often disproportionately affects
one sex.

4. Del Monte Phils vs. Velasco, G.R. No. 153477, March 6, 2007

PETITIONER: Del Monte RESPONDENT: Velasco, seasonal employee who was then regularized of Del Monte
Philippines working as a field laborer. Formerly pregnant. (See contention facts section)

SPECIAL DETAILS:
• Supervisor name – Prima Ybanez.
• Complainant incurred absences exceeding six (6) days as she actually failed to report for work from August
15-18, 23-26, 29-31, September 1-3, 5-10, 12-17, 21-24, 26-30, and October 1-3, 1994

LAW & PRINCIPLES:


Article 137 of the Labor Code
Art. 137. Prohibited acts. – It shall be unlawful for any employer:
(1) To deny any woman employee the benefits provided for in this Chapter or to discharge any woman
employed by him for the purpose of preventing her from enjoying any of the benefits provided under this
Code;
(2) To discharge such woman on account of her pregnancy, while on leave or in confinement due to her
pregnancy; or
(3) To discharge or refuse the admission of such woman upon returning to her work for fear that she may
again be pregnant.

Page 62 of 67
LABOR 1 | JMM | CASE DIGESTS 2020 – TOPICS 1-3

FACTS:
• Velasco was warned in writing due to her absences. Thru a letter, Velasco, was again warned in writing by
Del Monte about her absences without permission and a forfeiture of her vacation leave entitlement for the
year 1990-1991.
• Year 1991-1992 another warning letter was sent to Velasco regarding her absences without permission.
Her vacation entitlement for the said employment year affected was consequently forfeited.
• In view of the following, a notice of hearing was sent for charges against Velasco for violating the Absence
Without Official Leave Rule, which she failed to appear. Another notice of hearing was sent. After hearing,
Del Monte terminated the services of Velasco.
• Feeling aggrieved, Velasco filed for a petition for illegal dismissal.
Respondent’s contention: Dismissal was illegal because...
• She was suffering from urinary tract infection, a pregnancy-borne, at the time she committed the alleged
absences.
• Her absence from work on august 15, 16, 17 & 18, 1994 she had sent an application for leave to her
supervisor.
• She went to the company hospital for check-up and was advised accordingly to rest in quarters for four (4)
days or on August 27 to 30, 1994. Still not feeling well, she failed to work on September 1, 1994 and was
again advised two days of rest in quarters on September 2-3, 1994.
• she went to see an outside doctor and ordered her to rest for another five (5) consecutive days, or from
September 5 to 9, 1994. She declared she did not file the adequate leave of absence because a medical
certificate was already sufficient per company policy.
• September 1994 - she failed to report to work but sent an application for leave of absence to her supervisor
which was not anymore accepted.
Labor Arbiter:
• Velasco was an incorrigible absentee;
• She failed to file leaves of absence;
• Her absences in 1986 and 1987 were without permission;
• The Del Monte gave the Velasco several chances to reform herself;
• The Velasco did not justify her failure to appear during the scheduled hearings and failed to explain her
absences.
Velasco appealed to the NLRC.
NLRC:
1. Under the company rules, the employee may make a subsequent justification of her absenteeism, which
she was able to do
2. Not disputed that the Velasco incurred absences exceeding six (6) days within one employment year – a
ground for dismissal under the company rules – Del Monte admitted the fact that she had been pregnant,
hence, negating Del Monte’s assertion that the Velasco failed to give any explanation of her absences;
3. Records bear the admission of Del Monte’s officer of the receipt of the hospital record showing the cause
of her absences ("RIQ advice" or "rest-in-quarters") which could already serve as reference
4. Del Monte admitted that the Velasco was under "RIQ advice" on September 2-3, 1994 and yet insisted in
including these dates among Velasco’s 16 purported unexplained absences; it is sufficient for Velasco "a
plain laborer" with "unsophisticated judgment," to send word to her employer through a co-worker on
August 15 to 16, 1994 that she was frequently vomiting;
5. Sheer distance between Velasco’s home and her workplace made it difficult to send formal notice even
sending her child to inform her supervisor about her absence on September 5, 1994 due to stomach ache,
but her child failed to approach the officer because her child felt ashamed, if not mortified;
6. Her narration that she had to bear pains during her absences on September 21 to 27, 1994 is credible;
7. She dared not venture through the roads for fear of forest creatures or predators;
8. Del Monte is guilty of unlawfully discharging Velasco on account of her pregnancy under Article 137(2) of
the Labor Code;
9. Del Monte’s reference to the previous absenteeism of Velasco is misplaced because the latter had already
been penalized
Motion for Reconsideration denied. Del Monte then appealed to the CA. Petition dismissed, NLRC decision affirmed.
CA Decision:
1. Absences due to a justified cause cannot be a ground for dismissal;
2. Undisputed that the Velasco was pregnant at the time she incurred the absences in question;
3. Certification issued by a private doctor duly established this fact;

Page 63 of 67
LABOR 1 | JMM | CASE DIGESTS 2020 – TOPICS 1-3

4. No less than Del Monte’s company doctor who advised the Velasco to have rest-in-quarters for four days
on account of a pregnancy- related sickness;
5. Duly established that Velasco filed leaves of absence though the last had been refused by the company
supervisor;
6. The dismissal of an employee due to prolonged absence with leave by reason of illness duly established
by the presentation of a medical certificate is not justified;
7. It is undisputed that Velasco’s sickness was pregnancy-related;
8. Under article 137(2) of the labor code, the Del Monte committed a prohibited act in discharging a woman
on account of her pregnancy.
CA denied Del Monte’s Motion for Reconsideration. Hence.

ISSUES:
1. Whether the employment of respondent had been validly terminated on the ground of excessive absences
without permission.
2. Whether the petitioner discharged the respondent on account of pregnancy, a prohibited act.

RULING: CA and NLRC Decision AFFIRMED.

1. NO. Filflex is not applicable to the case because of the nature of the illness, which is pregnancy and related
illnesses. However, it is unreasonable to isolate the condition strictly to the dates stated in the medical
certificates. It can be safely assumed that, the absences not covered, are due to the continuing condition
of pregnancy and related illnesses, and, hence, are justified absences. SC adopt NLRC decision (No. 2).
Undisputed that complainant incurred absences exceeding 6 days. However, while it is not disputed that
complainant incurred absences exceeding six (6) days, her being pregnant at the time these absences
were incurred is not questioned and is even admitted by respondent. Court is puzzled why respondent
asserts complainant failed to explain satisfactorily her absences on August 15-18, 29-31, September 1-3
and 5-10, 1994, yet reconsidered the rest of her absences for being covered with "rest-in-quarters" (RIQ)
advice from its hospital personnel when this advice was unquestionably issued in consideration of the
physiological and emotional changes complainant, a conceiving mother, naturally developed.
2. YES. Court is convinced that the petitioner terminated the services of respondent on account of her
pregnancy which justified her absences and, thus, committed a prohibited act rendering the dismissal
illegal. Court agrees with the CA Decision No. 8. SC finds no cogent reason to disturb the findings of the
NLRC and the CA that the respondent was able to subsequently justify her absences in accordance with
company rules and policy; that the respondent was pregnant at the time she incurred the absences; that
this fact of pregnancy and its related illnesses had been duly proven through substantial evidence; that the
respondent attempted to file leaves of absence but the petitioner’s supervisor refused to receive them; that
she could not have filed prior leaves due to her continuing condition; and that the petitioner, in the last
analysis, dismissed the respondent on account of her pregnancy, a prohibited act. Petitioner puts much
emphasis on respondent’s "long history" of unauthorized absences committed several years beforehand.
However, petitioner cannot use these previous infractions to lay down a pattern of absenteeism or habitual
disregard of company rules to justify the dismissal of respondent. The undeniable fact is that during her
complained absences in 1994, respondent was pregnant and suffered related illnesses. Again, it must be
stressed that respondent’s discharge by reason of absences caused by her pregnancy is covered by the
prohibition under the Labor Code. Since her last string of absences is justifiable and had been subsequently
explained, the petitioner had no legal basis in considering these absences together with her prior infractions
as gross and habitual neglect.

NOTES:
-No jurisprudences, only law is used.

Page 64 of 67
LABOR 1 | JMM | CASE DIGESTS 2020 – TOPICS 1-3

5. Yrasuegui vs. Phil Air Lines, G.R. No. 168081, October 17, 2008

PETITIONER: Armando Yrasuegui RESPONDENT: Phil. Airlines Inc.


● Due to Yrasuegui inability to attain his ideal
weight, "and considering the utmost leniency"
extended to him "which spanned a period
covering a total of almost five (5) years", his
services were considered terminated "effective
immediately."

SPECIAL DETAILS:
● Yrasuegui weight problem of dates back to 1984, when PAL advised him to go on an extended vacation
leave from December 29, 1984 to March 4, 1985 to address his weight concerns.
● A month prior to his removal of flight attendant duties (Apr. 26, 1989), Yrasuegui weighed 209 pounds, 43
pounds over his ideal weight.
● Yrasuegui’s weight problem recurred so he again went on leave without pay from October 17, 1988 to
February 1989.
● He was removed from flight duty effective May 6, 1989 to July 3, 1989, and was formally requested to trim
down to his ideal weight and report for weight checks on several dates. He was advised that he may avail
of the services of the company physician should he wish to do so.
● On Feb 5, 1989, Yrasuegui was found overweight at 215 pounds, which is 49 pounds beyond the limit.
Consequently, his off-duty status was retained and on October 17, 1989, upon PAL Line Administrator Gloria
Dizon’s personally visit was found to have weighed 217 pounds,
● Despite, Yrasuegui’s letter and assurance to lose weight, he still remained overweight.

LAW & PRINCIPLES:


● Gross and habitual neglect, a recognized just cause, is considered voluntary although it lacks the element
of intent found in Article 282 (a), (c), and (d)."
● The burden of evidence lies with the party who asserts an affirmative allegation, petitioner has to prove
his allegation with particularity.

FACTS:
● Armando Yrasuegui is international flight steward of Philippine Airlines, Inc. (PAL). He stands five feet and
eight inches (5'8") with a large body frame. The proper weight for a man of his height and body structure is
from 147-166 pounds, the ideal weight being 166 pounds, as mandated by the Cabin and Crew
Administration Manual of PAL.
● PAL advised Yrasuegui to go on an extended vacation leave from December 29, 1984 to March 4, 1985 to
address his weight concerns.
● Yrasuegui failed to meet the company’s required weight standards so he prompted to take another leave
without pay from March 5, 1985 to November 1985.
● Yrasuegui met the required weight and was allowed to return to work.
● Yrasuegui weight problem occurred again so he was forced to take leave without pay again and trim down
his weight.
● Yrasuegui continued to become overweight.
● Yrasuegui reduced some weight but he was still overweight.
● PAL finally served Yrasuegui a Notice of Administrative Charge for violation of company standards on
weight requirements on November 13, 1992, .
● On June 15, 1993, Yrasuegui was formally informed by PAL
● Labor Arbiter (Valentin C. Reyes) ruled that the weight standards of PAL are reasonable in view of the nature
of the job, however, the weight standards need not be complied with under pain of dismissal since his
weight did not hamper the performance of his duties..
● NLRC affirmed Labor Arbiter’s decision.
● CA reversed the NLRC’s decision and denied the motion for reconsideration.

ISSUES:
Whether or not a person’s obesity can be a ground for dismissal under par (e) of Art. 282 of the Labor Code of the
Phils.

Page 65 of 67
LABOR 1 | JMM | CASE DIGESTS 2020 – TOPICS 1-3

RULING:
The SC affirmed the CA’s decision with a modification that Yrasuegui is entitled to a separation free.

On obesity of petitioner as a ground for dismissal under Article 282 (e) 44 of the Labor Code:
● The SC noted that while reducing weight is costing him "a lot of expenses,” Yrasuegui has only himself to
blame since he could have easily availed the assistance of the company physician, per the advice of PAL.
He chose to ignore the suggestion and he repeatedly failed to report when required to undergo weight
checks, without offering a valid explanation. Thus, his fluctuating weight indicates absence of willpower
rather than an illness.
● Furthermore, the Court noted that obesity may not be unintended, but is nonetheless voluntary. As the CA
correctly puts it, "[v]oluntariness basically means that the just cause is solely attributable to the employee
without any external force influencing or controlling his actions. This element runs through all just causes
under Article 282, whether they be in the nature of a wrongful action or omission. Gross and habitual
neglect, a recognized just cause, is considered voluntary although it lacks the element of intent found in
Article 282 (a), (c), and (d)."

Burden of evidence lies with the party who asserts discrimination


● The burden of evidence lies with the party who asserts an affirmative allegation, petitioner has to prove
his allegation with particularity. There is nothing on the records which could support the finding of
discriminatory treatment. Yrasuegui cannot establish discrimination by simply naming the supposed cabin
attendants who are allegedly similarly situated with him. Substantial proof must be shown as to how and
why they are similarly situated and the differential treatment petitioner got from PAL despite the similarity
of his situation with other employees.

On claims that reinstatement and wages are moot:


● Contrary to the allegation of Yrasuegui that PAL "did everything under the sun" to
● frustrate his "immediate return to his previous position", the SC found that there is evidence that PAL opted
to physically reinstate him to a substantially equivalent position in accordance
● with the order of the Labor Arbiter. Yrasuegui duly received the return to work notice on February 23, 2001.
● He failed to prove that he complied with the return to work order of PAL. Neither does it appear on record
that he actually rendered services for PAL from the moment he was dismissed, in order to insist on the
payment of his full backwages.
● In insisting that he be reinstated to his actual position despite being overweight, Yrasuegui in effect wants
to render the issues in the present case moot. He asks PAL to comply with the impossible.

On being entitled to a separation pay:


● Normally, a legally dismissed employee is not entitled to separation pay based on Art. 279 of the Labor
Code. However, the SC noted that there are possible exemptions granted to a legally dismissed employee
as an act "social justice", or based on "equity". In both instances, it is required that the dismissal
● (1) was not for serious misconduct; and
● (2) does not re􏰀ect on the moral character of the employee.

NOTES:
● The dismissal of petitioner can be predicated on the bona fide occupational qualification defense.
Employment in particular jobs may not be limited to persons of a particular sex, religion, or national origin
unless the employer can show that sex, religion, or national origin is an actual qualification for performing
the job. The qualification is called a bona fide occupational qualification (BFOQ). In short, the test of
reasonableness of the company policy is used because it is parallel to BFOQ. BFOQ is valid “provided it
reflects an inherent quality reasonably necessary for satisfactory job performance.”
● The business of PAL is air transportation. As such, it has committed itself to safely transport its
passengers. In order to achieve this, it must necessarily rely on its employees, most particularly the cabin
flight deck crew who are on board the aircraft. The weight standards of PAL should be viewed as imposing
strict norms of discipline upon its employees.
● The primary objective of PAL in the imposition of the weight standards for cabin crew is flight safety.

Page 66 of 67
LABOR 1 | JMM | CASE DIGESTS 2020 – TOPICS 1-3

● Separation pay, however, should be awarded in favor of the employee as an act of social justice or based
on equity. This is so because his dismissal is not for serious misconduct. Neither is it reflective of his moral
character.

Page 67 of 67

You might also like